Aaos 2001

You might also like

Download as pdf or txt
Download as pdf or txt
You are on page 1of 140

American Academy of Orthopaedic Surgeons

2001 Sports Medicine Self-Assessment Examination by Dr.Dhahirortho

1
1
2  American Academy of Orthopaedic Surgeons

2
2
American Academy of Orthopaedic Surgeons
2001 Sports Medicine Self-Assessment Examination by Dr.Dhahirortho

1.An 18-year-old high school football player sustains a thigh injury that results in the findings
shown in Figure 1. Initial management should consist of

1- gentle passive stretching.


2- pulsed therapeutic ultrasonography.
3- interferential electrical stimulation.
4- cross-fiber friction massage.
5- resting the muscle group.

DISCUSSION: The radiograph shows myositis ossificans within the


quadriceps muscle. This condition occurs as a complication of
muscle injury. Initial treatment should include rest, ice, compression,
and elevation. While gentle active range of motion is encouraged in
the functional recovery from this injury, passive stretching is
contraindicated as it can enhance hemorrhage and accentuate the
development of myositis ossificans. Ultrasound is similarly
contraindicated because it can enhance the development of myositis
ossificans and has no proven efficacy in this patient; electrical stimulation also has no proven
benefits. Massage is contraindicated in the initial management of this injury because of its
influence on increasing local blood flow. PREFERRED RESPONSE: 5

2. What is the function of the rotator cuff during throwing?

1- Limits humeral head translation in the transverse plane but not in the sagittal plane
2- Limits superior migration but not anterior and posterior translation
3- Limits superior migration and anterior and posterior translation
4- Provides little control of superior anterior and posterior translation
5- Creates inferior migration with maximal contraction during acceleration

DISCUSSION: The coupled action of the rotator cuff prevents superior migration and controls
anterior and posterior translation by depressing the humeral head. PREFERRED RESPONSE: 3

3. A 24-year-old female soccer player has had lateral joint line pain and a recurrent
effusion in the left knee after sustaining a twisting injury 6 weeks ago. She reports that
symptoms worsen with athletic activities. MRI scans are shown in Figures 2a through 2c.
What is the most likely diagnosis?

1- Osteochondral fracture of the lateral femoral condyle


2- Trabecular injury of the lateral tibial plateau
3- Lateral meniscal tear with a parameniscal cyst
4- Fibular collateral ligament tear
5- Discoid lateral meniscal tear
3
3
4  American Academy of Orthopaedic Surgeons

DISCUSSION: The MRI scans show the typical findings of a torn discoid lateral meniscus. The
average transverse diameter of the lateral meniscus is 11 or 12 mm. A discoid lateral meniscus is
suggested when three or more contiguous 5-mm sagittal sections on the MRI scan show continuity
of the menicus between the anterior and posterior horns, or when two adjacent peripheral sagittal 5-
mm sections show equal meniscal height. Normally the black “bow tie” would be seen on two
contiguous sagittal sections. The presence of a discoid meniscus can be further confirmed if
coronal views reveal increased width. PREFERR RESPONSE: 5

4. A 29-year-old woman who underwent an anterior cruciate ligament (ACL)


reconstruction 6 months ago now reports difficulty achieving full knee extension, and physical
therapy fails to provide relief. The knee is stable on ligament testing. Figure 3 shows the
findings at a repeat arthroscopy. Treatment should now include

1- revision of the failing ACL


reconstruction.
2- arthroscopic lysis of adhesions and
manipulation of the knee.
3- surgical removal of hypertrophic
fibrous tissue.
4- excision of the torn medial
meniscus.
5- continued aggressive physical
therapy.

DISCUSSION: The patient has a cyclops lesion.


This is a nodule of fibroproliferative tissue that originates from either drilling debris from the tibial
tunnel or remnants of the ACL stump; more rarely it is the result of broken graft fibers. The
treatment of choice is excision of the nodule and, if needed, additional notchplasty. Marked
improvements in function and symptoms have been noted after removal of the extension block and
resumption of a rehabilitation program.

PREFERRED RESPONSE: 3

5. The major blood supply to the cruciate ligaments arises from which of the
following structures?

1- Superior genicular artery


2- Middle genicular artery
3- Inferior genicular artery
4- Infrapatellar fat pad
5- Intramedullary vessels

DISCUSSION: The major blood supply to the cruciate ligaments arises from the ligamentous
branches of the middle genicular artery. Few terminal branches of the inferior genicular artery
contribute to the blood supply. The synovial plexus and sheath covering the cruciate ligaments are
also supplied by branches of the middle genicular artery. The blood supply to the cruciate
ligaments is predominately of soft-tissue origin. There is no significant osseous vascular
contribution to the ligaments.

PREFERRED RESPONSE: 2
4
4
American Academy of Orthopaedic Surgeons
2001 Sports Medicine Self-Assessment Examination by Dr.Dhahirortho

6.In the anterior cruciate ligament (ACL)-deficient knee, which of the following variables has
the highest correlation with the development of arthritis?

1- Duration of time since the injury


2- Patient age
3- Additional ligament injury
4- Degree of laxity
5- Meniscal integrity

DISCUSSION: Ample evidence supports an increased rate of degenerative arthritis in the ACL-
deficient knee. Several variables play a role in the development of the arthritis, but the integrity of
the meniscus has been shown to be the single most important factor. PREFERRED RESPONSE: 5

7. A 20-year-old football player has immediate pain in the midfoot and is unable to bear
weight after an opposing player lands on the back of his plantar flexed foot. AP and lateral
radiographs are shown in Figures 4a and 4b. Management should consist of

1- closed reduction and a non-weight-bearing cast.


2- closed reduction and a weight-bearing cast.
3- closed reduction and percutaneous pinning.
4- open reduction and casting.
5- open reduction and internal fixation.

DISCUSSION: The history and radiographs indicate a Lisfranc fracture-dislocation of the foot. The
radiographs show the classic “fleck sign,” which is an avulsion of the Lisfranc ligament from the
base of the second metatarsal. Most authors recommend open reduction and internal fixation of this
injury. Closed reduction can be attempted, but anatomic reduction is unlikely because of the
interposed bone fragments and soft tissues. Standard radiographs are not reliable in identifying 1 to
2 mm of subluxation of the tarsometatarsal joint. The tarsometatarsal joint has a poor tolerance to
even mild subluxation, and the resulting decrease in joint contact area increases the likelihood of
posttraumatic arthritis. Open reduction with the joint visible allows more anatomic reduction and
internal fixation of larger osteochondral fragments or excision of smaller interposed fragments.

PREFERRED RESPONSE: 5

8. What effect does deep freezing have on allograft tissue?

1- Causes no deleterious clinical effect on ligamentous grafts


2- Causes a less deleterious effect on cartilage than on ligamentous grafts
3- Causes degradation of the extracellular matrix
4- Allows for preservation of cells with tissue
5 5- Eliminates the chance of human immunodeficiency virus (HIV) transmission
5
6  American Academy of Orthopaedic Surgeons

DISCUSSION: Deep freezing is the simplest and most widely used method of ligament allograft
storage. All cells in the tissue are destroyed with the freezing. However, for this reason, it is not a
preferred storage method for menisci or cartilage allografts. Although this method may enhance
success because it removes potential antigens located on the cells, it cannot guarantee elimination of
HIV transmission. The advantage of cryopreservation storage is that a significant number of cells
will survive the process, a factor important in meniscal allograft survival after implantation. No
deleterious effects are noted clinically because of the acellularity of the tissue.

PREFERRED RESPONSE: 1

9. A 32-year-old man who works as a laborer has had left trapezius wasting and lateral
scapular winging after injuring his shoulder when a cargo box fell onto his neck 8 months ago.
He now reports posterior shoulder pain and fatigue, and he has difficulty shrugging his
shoulder. Examination reveals marked scapular winging, impingement signs, and an
asymmetrical appearance when the patient attempts a shoulder shrug. Primary scapular-
trapezius winging is the result of damage to the

1- spinal accessory nerve, causing shoulder depression with the scapula translated
laterally and the inferior angle rotated laterally.
2- spinal accessory nerve, causing shoulder elevation with the scapula translated
medially and the inferior angle rotated medially.
3- long thoracic nerve, causing shoulder elevation with the scapula translated medially
and the inferior angle rotated medially.
4- long thoracic nerve, causing shoulder depression with the scapula translated laterally
and the inferior angle rotated laterally.
5- thoracodorsal nerve, causing shoulder depression with the scapula translated laterally
and the inferior angle rotated laterally.

DISCUSSION: The patient has primary scapular-trapezius winging. This condition can be caused
by blunt trauma to the relatively superficial spinal accessory nerve that is located in the floor of the
posterior cervical triangle in the subcutaneous tissue. Other causes of injury include penetrating
trauma, traction, or surgical injury. With trapezius winging, the shoulder appears depressed and
laterally translated because of an unopposed serratus anterior. This contrasts with primary serratus
anterior winging, which is caused by injury to the long thoracic nerve. In this condition, the scapula
assumes a position of superior elevation and medial translation, and the inferior angle is rotated
medially. The thoracodorsal nerve supplies the latissimus dorsi and is not involved in primary
scapular winging.

PREFERRED RESPONSE: 1

6
6
American Academy of Orthopaedic Surgeons
2001 Sports Medicine Self-Assessment Examination by Dr.Dhahirortho

10. A 32-year-old football coach has had a 4-month history of increasing right wrist pain,
particularly during blocking exercises, and he reports significant pain with range of motion
and gripping activities. He denies any history of trauma. Examination reveals dorsal wrist
tenderness and boggy fullness over the dorsum of the wrist. No erythema is noted. Grip
strength is 60% compared with the opposite side. Radiographs are shown in Figures 5a and
5b. What is the most likely diagnosis?

1- Scapholunate dissociation
2- Triangular fibrocartilage tear
3- Scaphoid fracture
4- Perilunate dislocation
5- Kienbock’s disease

DISCUSSION: The patient has Kienbock’s


disease (osteonecrosis of the lunate), which
presents with boggy synovitis of the wrist,
decreased range of motion, and often normal
radiographs. The patient’s radiographs
reveal small fragments from the lunate, with
increased density in the lunate body. While
a traumatic event may precede the patient’s
pain, often an insidious increase in pain is
found. Repetitive trauma has been suggested as a possible cause. This disease process is classically
associated with an ulnar-negative variant. An MRI scan, revealing a low-intensity signal in the
lunate, is the best diagnostic tool for early Kienbock’s disease.

PREFERRED RESPONSE: 5

11. Which of the following properties apply to the human meniscus when compared with
articular cartilage?

1- Less elastic and less permeable


2- Less elastic and more permeable
3- Of the same elasticity and permeability
4- More elastic and more permeable
5- More elastic and less permeable

DISCUSSION: The meniscal cartilage, like articular cartilage, possesses viscoelastic properties.
The extracellular matrix is a biphasic structure composed of a solid phase (collagen, proteoglycan)
that acts as a fiber-reinforced porous-permeable composite, and a fluid phase that may be forced
through the solid matrix by a hydraulic pressure gradient. Although these properties are shared with
articular cartilage, the meniscus is more elastic and less permeable than articular cartilage.

PREFERRED RESPONSE: 5

7
7
8  American Academy of Orthopaedic Surgeons

12. An 18-year-old football player lands on a flexed knee and ankle after being tackled.
Examination reveals increased external rotation and posterior translation and varus at 30° of
flexion, which decreases as the knee is flexed to 90°. What is the most likely diagnosis?

1- Torn posterolateral corner


2- Torn posterior cruciate ligament (PCL) and posterolateral corner
3- Torn PCL
4- Rupture of the quadriceps tendon
5- Rupture of the lateral collateral ligament

DISCUSSION: The flexed knee and ankle mechanism of injury can result in a PCL and/or
posterolateral corner injury. The examination reveals an isolated injury to the posterolateral corner
(arcuate, popliteus, posterolateral capsule). This results in increased posterior translation and
external rotation, as well as varus that is most notable at 30° of flexion and decreases as the knee is
further flexed to 90°. Combined PCL and posterolateral corner injuries are characterized by
increasing instability as the knee is flexed to 90° from 30°, while isolated PCL tears show the
greatest degree of instability at 90° of flexion. A rupture of the quadriceps tendon would not affect
anterior or posterior stability, whereas an isolated rupture of the lateral collateral ligament, which is
a rare injury, is characterized by varus instability at 30° of knee flexion without posterior
translation.
PREFERRED RESPONSE: 1

13. Figure 6 shows the radiograph of a 14-year-old baseball player who felt a pop and had
an immediate onset of pain in his elbow after a hard throw from the outfield. The best course
of action should be to

1- obtain stress radiographs of the elbow.


2- obtain an MRI scan of the elbow.
3- apply a splint and initiate early range-of-motion
exercises.
4- apply a cast in 90° of flexion for 4 weeks.
5- perform open reduction and internal fixation.

DISCUSSION: The valgus stress at the elbow caused by throwing


strains the medial collateral ligament. The medial epicondyle, on
which the ligament inserts, is the last ossification center to fuse to
the distal humerus, and acute avulsion of the medial epicondyle
can occur in adolescents. If the elbow is allowed to heal in a
displaced position, valgus instability and loss of elbow extension
may result. Valgus instability is especially problematic for the throwing athlete. Surgical treatment
with rigid internal fixation is the treatment of choice for displaced medial epicondyle avulsion
fractures. Valgus instability is prevented, and the rigid fixation allows for early range of motion.
PREFERRED RESPONSE: 5

14. Osteophyte formation at the posteromedial olecranon and olecranon articulation in


high-caliber throwing athletes is most often the result of underlying

1- anterior capsular tears.


2- forearm pronator and flexor muscle weakness.
3- biceps or brachialis muscle weakness.
4- ulnar collateral ligament insufficiency.
8 5- radial collateral ligament insufficiency.

8
American Academy of Orthopaedic Surgeons
2001 Sports Medicine Self-Assessment Examination by Dr.Dhahirortho

DISCUSSION: During the late acceleration phase of throwing, the triceps forcibly contracts,
extending the elbow as the ball is released. Normally, this force is absorbed by the anterior capsule
and the brachialis and biceps muscles. However, if the ulnar collateral ligament is insufficient, the
elbow will be in a subluxated position during extension and cause impaction of the olecranon and
the olecranon fossa posteromedially. Over time, osteophyte formation is likely to occur.
PREFERRED RESPONSE: 4

15. Sudden cardiac death in the young athlete is most frequently caused by

1- hypertrophic cardiomyopathy.
2- active myocarditis.
3- mitral valve prolapse.
4- aortic rupture.
5- coronary artery disease.

DISCUSSION: Hypertrophic cardiomyopathy is the leading cause of sudden cardiac death in


athletes, accounting for 40% of reported cases. Most athletes have no previous symptoms, and
sudden death may be the first clinical manifestation. The prevalence of hypertrophic
cardiomyopathy in the general population is 1 in 500, with a mortality rate of 2% to 4% in young
adults. Athletes with active myocarditis should not engage in sports for up to 6 months, and
although they may be at risk for the development of chronic cardiomyopathy, it is rarely a cause of
sudden cardiac death. Mitral valve prolapse with an accompanying systolic murmur is common in
the general population, but infrequently a cause of sudden cardiac death. Weakening of the aortic
wall associated with Marfan syndrome can result in abrupt rupture of the aorta. This accounts for
3% of sudden cardiac deaths in young athletes. Marfan syndrome usually can be detected on
preparticipation screenings by its skeletal and ocular manifestations. Atherosclerotic coronary
artery disease is the most common cause of sudden cardiac death in older athletes, accounting for
75% of reported cases. However, it is much less common in the young competitive athlete.

PREFERRED RESPONSE: 1

16. A 14-year-old football player has had right knee pain for the past 2 months; however,
he denies any history of trauma. Examination shows an abductor lurch and increased
external rotation of the right lower extremity. The best course of action should be to

1- apply a knee sleeve during sports.


2- withdraw from football for 2 weeks.
3- obtain AP and frog-lateral radiographs of the pelvis.
4- obtain an MRI scan of the right knee.
5- initiate physical therapy.

DISCUSSION: Slipped capital femoral epiphysis is the most common pathology involving the hip
in adolescents. While patients with acute slips may report severe pain and are unable to ambulate,
those with chronic slips often have pain during ambulation, a limp, and increased external rotation
of the hip. While 60% of the patients specifically report hip pain, the remainder have pain in the
thigh or knee. The initial diagnostic study of choice is AP and frog-lateral radiographs of the
pelvis; bilateral involvement is frequently seen.

PREFERRED RESPONSE: 3

9
9
10  American Academy of Orthopaedic Surgeons

17.Which of the following is considered the appropriate initial management protocol for an
unconscious football player without spontaneous respirations?

1- Log roll to a supine position, remove the helmet, and begin assisted breathing
2- Stabilize the head and neck, log roll to a supine position, remove the helmet, and
begin assisted breathing
3- Log roll to a supine position, stabilize the head and neck, remove the face mask, and
begin cardiopulmonary respiration (CPR)
4- Log roll onto a spine board, stabilize the head and neck, remove the face mask, and
begin CPR
5- Stabilize the head and neck, log roll to a supine position, remove the face mask, and
begin assisted breathing

DISCUSSION: The on-field evaluation and management of the seriously injured athlete requires
advance preparation and planning. It is imperative that the health care team have a game plan in
place and the proper equipment readily available. The initial step consists of stabilizing the head
and neck by manually holding the head and neck in a neutral position. Then, in the following order,
check for breathing, pulses, and level of consciousness. If the athlete is breathing, simply remove
the mouth guard and maintain the airway. If the athlete is not breathing, the face mask must be
removed and the chin strap left in place. An open airway must be established, followed by assisted
breathing. CPR is only instituted when breathing and circulation are compromised. If the athlete is
unconcious or has a suspected cervical spine injury, the helmet must not be removed until the
athlete has been transported to an appropriate facility and the cervical spine has been completely
evaluated.

PREFERRED RESPONSE: 5

18. Figure 7 shows the radiograph of an 18-year-old hockey player who sustained a
shoulder injury during a fall into the side boards. Examination reveals a significant
prominence at the acromioclavicular joint. Management should consist of

1- a figure-of-8 clavicle strap.


2- a sling for comfort, followed by early range-
of-motion and strengthening exercises.
3- open reduction and stabilization.
4- immobilization in a spica cast.
5- resection of the distal clavicle.

DISCUSSION: The radiograph shows a type V


acromioclavicular separation with greater than 100%
superior elevation of the clavicle. This finding
implies detachment of the deltoid and trapezius from
the distal clavicle. Because of severe compromise of
function and potential compromise to the overlying
skin, surgery is the treatment of choice for type V
acromioclavicular separations. During reduction and
repair, meticulous repair of the deltotrapezial fascia will also aid in securing the repair.

PREFERRED RESPONSE: 3

10
10
American Academy of Orthopaedic Surgeons
2001 Sports Medicine Self-Assessment Examination by Dr.Dhahirortho

19. A 22-year-old professional ballet dancer reports a 3-month history of posterior ankle
pain that occurs when she changes from a flat foot to pointe (hyperplantar flexed position).
Examination does not elicit the pain with forced passive plantar flexion. A radiograph is
shown in Figure 8. What is the most likely cause of the pain?

1- Mild subtalar arthritis


2- Posterior tibialis tendinitis
3- Os trigonum entrapment syndrome
4- Flexor hallucis longus tenosynovitis
5- Retrocalcaneal bursitis

DISCUSSION: The most common causes of posterior ankle pain in


ballet dancers are flexor hallucis longus tenosynovitis and os
trigonum syndrome. Flexor hallucis longus tenosynovitis differs
from a symptomatic os trigonum by the absence of pain with forced
plantar flexion and the presence of pain with resisted plantar flexion
of the great toe. The pain is often felt in the posterior ankle and can
be associated with a snapping or triggering sensation. Os trigonum
syndrome commonly occurs in ballet dancers who perform in a position of extreme plantar flexion.
The pain occurs from entrapment of the os trigonum between the posterior portion of the talus and
calcaneus. PREFERRED RESPONSE: 4

20. An 18-year-old man recently underwent an uncomplicated arthroscopic partial medial


meniscectomy that was complicated by reflex sympathetic dystrophy (RSD), also termed
“sympathetically maintained pain” (SMP). What is the most common finding of
this condition?
1- Joint stiffness
2- Cold intolerance
3- Decreased sweating
4- Osteopenia
5- Disproportionate pain

DISCUSSION: The hallmark for RSD or SMP is the presence of pain that is out of proportion to
that expected for the degree of the injury. SMP often extends well beyond the involved area and is
present in a nonanatomic distribution. The pain is frequently described as a burning sensation, with
extreme sensitivity to light touch. Joint stiffness can be present but is a nonspecific finding. There
may be cold intolerance, but this is not a cardinal symptom. Sweating actually may be increased.
Osteopenia, if present, is a late finding. PREFERRED RESPONSE: 5

21. What is the main function of collagen found within articular cartilage?

1- Compressive properties
2- Tensile properties
3- Proteoglycan synthesis
4- Cartilage metabolism
5- Joint lubrication

DISCUSSION: The main function of collagen in articular cartilage is to provide the tissue’s tensile
strength. It also immobilizes proteoglycans within the extracellular matrix. Compressive properties
are maintained by proteoglycans. Cartilage metabolism is maintained by the indwelling
chondrocytes. The flow of water through the tissue promotes transport of nutrients and provides a
11 source of lubricant for the joint. PREFERRED RESPONSE: 2
11
12  American Academy of Orthopaedic Surgeons

22. A 15-year-old girl who competes in gymnastics has immediate pain and giving way of
the left elbow after falling from the uneven parallel bars and landing on her outstretched
arms. Examination reveals swelling and tenderness about the elbow, especially over the
medial side. Measurement of elbow motion shows 0° to 125° of flexion, and valgus stress at
the elbow is painful. AP, lateral, and stress radiographs are shown in Figures 9a through 9c.
Management should consist of

1- arthroscopic repair of the ulnar collateral ligament.


2- direct surgical repair of the ulnar collateral ligament.
3- reconstruction of the ulnar collateral ligament with a palmaris longus tendon
autograft.
4- a hinged elbow brace to allow early protected range of motion.
5- immobilization of the elbow to allow healing of the ulnar collateral ligament.

DISCUSSION: While many low-demand patients with injuries to the ulnar collateral ligament can
be treated nonsurgically, Jobe and associates described two situations in which ulnar collateral
ligament reconstruction is indicated: (1) an acute complete rupture in a competitive athlete who uses
the upper extremities extensively and who wishes to remain active; and (2) chronic pain or
instability that does not improve after at least 3 months of nonsurgical management. Rarely is
direct surgical repair of the ligament possible or able to withstand the valgus stresses applied to the
elbow. Most authors recommend surgical reconstruction of the ulnar collateral ligament using a
palmaris longus, plantaris, or fourth toe extensor tendon from the fourth autograft.PR RESP: 3

23. A 15-year-old boy who participates in track reports acute pain along the left iliac crest
during a sprint. Examination reveals that the anterior superior iliac spine is nontender. The
most likely diagnosis is an injury to the

1- epiphysis.
2- apophysis.
3- enthesis.
4- tendon.
5- muscle.

DISCUSSION: The patient has iliac apophysitis. The radiographic findings are easily overlooked
but usually reveal slight asymmetric widening of the iliac crest apophysis. The apophysis is the
most vulnerable structure, as it is three to five times weaker than the tendon. This is not an
epiphyseal site, and injury to the muscle or the tendinous insertion to bone (enthesis) is unlikely.
12 PREFERRED RESPONSE: 2

12
American Academy of Orthopaedic Surgeons
2001 Sports Medicine Self-Assessment Examination by Dr.Dhahirortho

24. A 40-year-old woman who is an avid tennis player reports the insidious onset of
progressive left shoulder pain for the past 2 months. Examination reveals full range of
motion with a positive impingement sign. Strength in the supraspinatus and infraspinatus
muscles is normal, although stress testing is painful. An earlier subacromial cortisone
injection provided good, but only temporary relief. An AP radiograph of the left shoulder is
shown in Figure 10. Management should now consist of

1- a rotator cuff exercise program and anti-inflammatory drugs.


2- repeat subacromial cortisone injections as necessary.
3- open subacromial decompression.
4- arthroscopic evacuation of calcium deposits.
5- open rotator cuff repair.

DISCUSSION: The radiograph shows calcific deposits within the


substance of the supraspinatus tendon. Patients with this condition
are prone to recurrent bouts of acute inflammation in the shoulder.
While the response to cortisone injection is often dramatic, repeated
injections are not recommended because of injury to the collagen
fibers. Good results have been obtained with arthroscopic
evacuation of the calcium deposits. In one study, the addition of a
subacromial decompression did not improve the results.

PREFERRED RESPONSE: 4

25. Which of the following nerves is susceptible to entrapment near the calcaneal
attachment site of the plantar fascia and can mimic or co-exist with plantar fasciitis?

1- First branch of the lateral plantar nerve


2- Dorsal cutaneous branch of the superficial peroneal nerve
3- Medial calcaneal branch of the posterior tibial nerve
4- Lateral branch of the medial plantar nerve
5- Communicating branch of the fourth common digital nerve

DISCUSSION: The first branch of the lateral plantar nerve is susceptible to entrapment beneath the
deep fascia of the adductor hallucis muscle adjacent to the calcaneal attachment of the plantar
fascia. This can be a cause of chronic heel pain. Additionally, the nerve is vulnerable to injury by a
blind dissection in releasing the plantar fascia. The dorsal cutaneous branch of the superficial
peroneal nerve supplies sensation to the dorsum of the foot. The medial calcaneal branch of the
posterior tibial nerve lies in the subcutaneous tissues and innervates the skin of the heel. It is
vulnerable to injury from skin incisions on the medial side of the heel. The lateral branch of the
medial plantar nerve forms the second and third common digital nerves. Entrapment of the proper
medial plantar nerve can occur at the master knot of Henry. This is well distal to the calcaneal
attachment of the plantar fascia, and the pain usually radiates more distally in the arch, separate
from heel pain. The communicating branch of the fourth common digital nerve crosses to the third
common digital nerve. Therefore, the third common digital nerve receives supply from both the
lateral and medial plantar nerves. This dual supply has been implicated in the increased incidence
of digital neuroma of the third common digital nerve.PREFERRED RESPONSE: 1
13
13
14  American Academy of Orthopaedic Surgeons

26. Figure 11 shows the radiograph of an 18-year-old soccer player who reports recurrent
lateral foot pain after sustaining an inversion injury. History reveals that 6 months ago he
had been treated in a non-weight-bearing cast for a fifth metatarsal fracture. Management
should consist of

1- intermedullary fixation.
2- a brace or taping to limit inversion stress.
3- a short leg walking cast or a fracture walker.
4- a non-weight-bearing short leg cast.
5- a rigid orthotic insole, with early motion exercises.

DISCUSSION: Fractures in this area of the fifth metatarsal have a


high incidence of delayed union, nonunion, and recurrence with
nonsurgical management. In an acute fracture, prolonged casting in a
non-weight-bearing cast may allow for healing; however, in the
presence of prolonged symptoms, recurrent fracture, and
intermedullary sclerosis, surgical treatment is preferred. Surgery
most commonly consists of intermedullary fixation or medullary
curettage and bone grafting, followed by application of a non-weight-
bearing cast. PREFERRED RESPONSE: 1

27. Which of the following types of exercise used to increase flexibility is considered most
beneficial in increasing joint range of motion?

1- Ballistic stretching
2- Static stretching
3- Proprioceptive neuromuscular facilitation (PNF)
4- Isokinetic
5- Eccentric

DISCUSSION: Evidence has shown that PNF is the treatment of choice to increase joint range of
motion and flexibility. PNF has the advantage of pushing the patient to stretch a little further when
the muscle tendon unit is relaxed by a partner. While isokinetic and eccentric exercises can
improve flexibility, and therefore increase range of motion, their main purpose is to increase
strength and endurance. Ballistic stretching involves a large load applied rapidly; however,
evidence has shown that static stretching, where a low load is applied for a long duration, offers a
more significant benefit. PREFERRED RESPONSE: 3

28. The view from an anterosuperior portal of the right shoulder shown in Figure 12
reveals which of the following findings?

1-Rupture of the subscapularis tendon


2-Tear of the rotator interval
3-Humeral avulsion of the glenohumeral ligament (HAGL)
lesion
4-Anterior ligamentous periosteal sleeve avulsion (ALPSA)
lesion
5-Bankart lesion

14
14
American Academy of Orthopaedic Surgeons
2001 Sports Medicine Self-Assessment Examination by Dr.Dhahirortho

DISCUSSION: The arthroscopic view shows a HAGL lesion. With the arthroscope directed
anteroinferiorly, muscular striations of the subscapularis can be visualized through the avulsion site.
In vitro strain studies indicate that glenohumeral ligament failure on the humeral side occurs in
approximately 25% of patients, while clinically this lesion has been reported in approximately 9%
of patients with shoulder instability. Failure to recognize and treat this lesion leads to persistent
anterior instability. An ALPSA lesion, a Bankart variant, occurs on the glenoid side and is
characterized by a sleeve-like medial retraction and inferior rotation. A Bankart lesion is the classic
avulsion of the glenohumeral ligament from the glenoid rim. The subscapularis tendon and the
rotator interval are not shown in the figure. PREFERRED RESPONSE: 3

29. An 18-year-old football player has intense pain and is unable to bear weight on the
right knee after being tackled from the front. A posterior knee dislocation is reduced on the
field. Because the game took place in a remote location, the patient is not examined in the
emergency department until 5 hours after the injury. Examination now shows a grossly
swollen knee with moderate ischemia in the lower leg. Posterior tibial and dorsalis pedis
pulses are diminished. The best course of action should be to

1- obtain an emergent arteriogram.


2- obtain an emergent MRI scan.
3- perform a thorough examination of the knee ligaments.
4- perform surgical repair or bypass of the injured popliteal vessels.
5- perform surgical repair or bypass of the injured popliteal vessels and ligament
reconstruction.

DISCUSSION: Vascular injuries occur in approximately 20% to 35% of knee dislocations, of


which one third are posterior. Recognition of the vascular injury is essential. Normal pulses or
normal capillary refill do not preclude an arterial injury, and arteriography should be considered in
all knee dislocations. If the leg is ischemic, the arteriogram should be circumvented and the patient
taken directly to the operating room. The risk of muscle fibrosis, contracture, or vascular
insufficiency, and the need for amputation increase significantly when ischemia exceeds 6 hours.
This patient has ischemia and is considered a vascular emergency. As such, delays for a thorough
examination of the ligament, MRI scans, and even an arteriogram are unwarranted. Concurrent
ligamentous repair and reconstruction should be deferred until vascular stability has been achieved.
PREFERRED RESPONSE: 4

30. A 17-year-old football player is unable to flex the distal interphalangeal (DIP) joint of
his ring finger. He states that he injured the finger 6 weeks ago while attempting to tackle
another player who pulled free from his grip, but he did not inform his coach at the time of
the injury. Current radiographs show an observable fleck of bone volar to the base of the
proximal phalanx. Treatment should consist of

1- fusion of the DIP joint with no reconstruction of the tendon.


2- advancement and repair of the tendon to the base of the distal phalanx.
3- two-stage reconstruction of the profundus tendon.
4- Z-plasty advancement of the profundus tendon.
5- tenodesis of the distal tendon remnant with the flexor digitorum sublimis.

DISCUSSION: Flexor digitorum profundus ruptures are classified into three types. In type I, the
tendon retracts into the palm. In type II, the tendon retracts to the level of the proximal phalanx, the
vinculum remains intact, and the blood supply is preserved to the tendon. A small fleck of bony
fragment observed at the A2 pulley is pathognomonic for a type II rupture. Successful primary
15 repair of the type II rupture has been reported as late as 2 months after the injury. Type III injuries
15
16  American Academy of Orthopaedic Surgeons

have large fragments of the distal phalanx attached and are caught distally by the A1 pulley. Type
III ruptures can be repaired up to several months after the injury.

PREFERRED RESPONSE: 2

31. A 48-year-old ski instructor dislocates his nondominant shoulder in a fall.


Management consisting of application of a sling for 1 week results in improvement in his pain.
Follow-up examination 6 weeks after the injury reveals that the patient continues to have
difficulty with shoulder elevation. Management should now include

1- use of the sling for an additional 3 weeks.


2- physical therapy.
3- a corticosteroid injection.
4- an MRI scan of the rotator cuff.
5- arthroscopic labral repair.

DISCUSSION: Patients who are older than age 45 years and have initial dislocations are at greater
risk for tearing the rotator cuff. Patients who are unable to lift the upper extremity or who have
continued pain should undergo further evaluation for potential rotator cuff tears; early diagnosis is
preferred. Physical therapy or continued use of a sling will be of little benefit. A corticosteroid
injection might delay the diagnosis and compromise subsequent rotator cuff repair. Repairing the
labrum generally is not necessary in a patient of this age who has an initial dislocation.

PREFERRED RESPONSE: 4

32. Figure 13 shows the MRI scan of a 29-year-old rock climber who reports increasing
shoulder pain and weakness. Based on these findings, atrophy will most likely occur in which
of the following muscles?

1- Infraspinatus and supraspinatus


2- Infraspinatus
3- Supraspinatus
4- Teres minor
5- Deltoid

DISCUSSION: The MRI scan shows a cyst at the spinoglenoid notch. These cysts are often
associated with a labral injury, such as a superior labrum anterior and posterior (SLAP) lesion. The
suprascapular nerve passes through the suprascapular notch and sends motor branches to the
supraspinatus and sensory branches to the capsule. At the spinoglenoid notch, the infraspinatus
branch of the suprascapular nerve is compressed by the cyst, leading to isolated infraspinatus
atrophy. The teres minor and the deltoid are innervated by the axillary nerve.

PREFERRED RESPONSE: 2

16
16
American Academy of Orthopaedic Surgeons
2001 Sports Medicine Self-Assessment Examination by Dr.Dhahirortho

33. A 46-year-old man has acute tenderness along the ulnar aspect of the wrist after falling
on his outstretched hand while playing basketball. Examination reveals tenderness and mild
swelling along the volar ulnar aspect of the wrist. Radiogaphs are shown in Figures 14a
through 14c. Management should consist of

1- immobilization.
2- closed reduction.
3- open reduction and internal fixation.
4- early range of motion.
5- excision.

DISCUSSION: The PA view of the wrist shows a pisiform fracture. Pisiform fractures constitute
1% to 3% of all carpal bone fractures. This fracture can be further evaluated with a carpal tunnel
view or a supination oblique view of the wrist. Initial management should consist of
immobilization with a short arm cast. If nonsurgical measures fail, bony excision is warranted.
PREFERRED RESPONSE: 1

34. A 32-year-old powerlifter who was performing a dead lift 3 days ago noted a sharp
pain in the front of his dominant right arm just after beginning to lower the weight. He now
reports pain in the anterior aspect of the arm that worsens when he opens a door.
Examination reveals moderate ecchymosis and swelling of the forearm and tenderness in the
antecubital fossa. The MRI scans are shown in Figures 15a and 15b. If the injury is left
unrepaired, the greatest functional deficit will most likely be the loss

1- elbow extension motion.


2- elbow flexion strength.
3- forearm supination motion.
4- forearm pronation strength.
5- forearm supination strength.

DISCUSSION: A complete tear of the distal biceps brachii most often occurs from a large, rapid
eccentric elbow extension load. A pop or tearing sensation usually occurs, and a palpable defect in
the antecubital fossa is often present on examination. The treatment of choice is a direct primary
17 repair by a two-incision technique. If left unrepaired, the most disabling consequence is the loss of
17
18  American Academy of Orthopaedic Surgeons

forearm supination strength. It is unlikely that significant elbow or forearm motion will be lost if
the rupture is left unrepaired and early motion exercises are initiated. Elbow flexion strength tends
to return with time, but the loss of forearm supination strength remains problematic. PR RESP: 5

35. Figure 16 shows the lateral radiograph of a patient who is scheduled to undergo an
anterior cruciate ligament (ACL) reconstruction. If the graft is tensioned at 20° of flexion and
the femoral tunnel is created by passing a reamer over the guide wire marked “A,” the
resulting ligament reconstruction will excessively

1-tighten as the knee extends past 10° of flexion.


2-tighten as the knee flexes past 90°.
3-loosen as the knee extends past 10° of flexion.
4-loosen as the knee flexes past 30°.
5-loosen as the knee flexes past 90°.

DISCUSSION: If the femoral tunnel is created using guide wire A, it will be too far anterior in the
intercondylar notch. The distance between a central tibial insertion for the ACL and an anterior
femoral tunnel will progressively increase as the knee is flexed. Therefore, if the graft is tensioned
near extension, the ligament will excessively tighten as the knee flexes past 90°. This will result in
restricted knee flexion or failure of the graft as full flexion is gained. There will be little effect on
the ligament as it extends from 20° to 0° of flexion. If the graft is tensioned in significant flexion
(greater than 60°), it will be excessively loose as the knee fully extends. PREFERR RESP: 2

36. Which of the following nerves is most commonly injured during revision surgery
following a Bristow procedure?
1- Dorsal scapular
2- Suprascapular
3- Axillary
4- Musculocutaneous
5- Ulnar

DISCUSSION: Because of the previously transferred bone block of coracoid and short arm flexors,
the musculocutaneous nerve often scars along the anteroinferior glenohumeral capsule.
Mobilization of this tissue places the nerve at greatest risk. The axillary nerve is also potentially at
risk, but this is nonspecific to prior surgery, particularly the Bristow procedure. PREF RESPONS: 4

37. A 17-year-old high school soccer player sustains an anterior cruciate ligament (ACL)
tear at the beginning of the season. An MRI scan confirms a complete ACL tear with no
meniscal injuries. The patient plans an early return to play and would like to avoid surgery.
Therefore, the patient and family should be advised that nonsurgical management consisting
of rehabilitative exercises and the use of a functional knee brace will most likely result in

1- recurrent buckling with a probable meniscal tear.


2- limitation of motion with a delay in recovery.
3- a full return to activity with no limitations.
4- an improvement in overall performance.
18 5- an uneventful completion of the soccer season.

18
American Academy of Orthopaedic Surgeons
2001 Sports Medicine Self-Assessment Examination by Dr.Dhahirortho

DISCUSSION: While there are athletes who can function at a full level with an ACL tear, they are
in the minority. As yet, there is no reliable way to predict the patients who will be able to
compensate for the loss of the ACL. Studies have confirmed the risk of recurrent instability and
meniscal injury in athletes with an ACL-deficient knee who participate in cutting sports. One study
showed that only 12 of 43 patients who attempted rehabilitation and bracing were able to return
successfully for the season. Another study showed that 17 of 31 athletes who were able to return to
their sport sustained 23 meniscal tears because of recurrent instability. PREFERR RESPONSE: 1

38. A patient underwent anterior stabilization of the shoulder 6 months ago, and
examination now reveals lack of external rotation beyond 0°. The patient has a normal
apprehension sign and normal strength, and the radiographs are normal. Based on these
findings, the patient is at greater risk for the development of
1- recurring instability.
2- osteoarthritis.
3- osteonecrosis.
4- a tear of the rotator cuff.
5- internal impingement.

DISCUSSION: Because the patient’s shoulders are overtensioned anteriorly, premature


osteoarthritis may develop. This may create obligate translation posteriorly and increase the
interarticular pressure of the humeral head against the glenoid. Patients should achieve 20° to 30°
of external rotation with the elbow at the side. Late degenerative arthritis following a Putti-Platt
procedure is associated with significant restriction of external rotation. This patient’s shoulder has
a reduced risk of anterior instability, rotator cuff tear, and internal impingement because of the
limitation of motion. PREFERRED RESPONSE: 2

39. A 13-year-old girl who competes in gymnastics reports the insidious onset of lateral left
elbow pain over the past 6 months. She also notes occasional catching episodes in the elbow;
however, she denies any history of trauma. Examination reveals tenderness over the lateral
epicondyle and extensor muscle origin. The elbow is stable and has full flexion, but lacks 10°
of full extension. An AP plain radiograph and an MRI scan are shown in Figures 17a and
17b. Management of the elbow should consist of

1- open excision of the radial head.


2- a cortisone injection into the extensor muscle origin.
3- a tennis elbow release.
4- arthroscopic removal of loose bodies and microfracture of the crater.
5- rest, physical therapy, pulsed electromagnetic therapy, and no further gymnastic activities.
19
19
20  American Academy of Orthopaedic Surgeons

DISCUSSION: The radiograph and MRI scan show osteochondritis dissecans of the capitellum, and
the patient’s history suggests a loose body. The treatment of choice is arthroscopic removal of the
loose body and microfracture of the crater. Excision of the radial head, a cortisone injection, or
tennis elbow release does not treat the pathology in the capitellum. Nonsurgical treatment would
not relieve the mechanical symptoms of the loose body or promote healing in the crater.
PREFERRED RESPONSE: 4

40. A 25-year-old man injures his shoulder while skiing. Examination reveals increased
passive external rotation, pain in the cocked position, and a positive lift-off test. What is the
most likely diagnosis?

1- Ruptured biceps tendon


2- Subscapularis tear
3- Anterior subluxation
4- Internal impingement syndrome
5- Locked posterior dislocation

DISCUSSION: A positive lift-off test and increased passive external rotation are diagnostic of a
subscapularis tear or detachment. Although a similar injury could produce anterior instability, this
will test the integrity of the subscapularis. A locked dislocation has limited passive movement. A
ruptured biceps tendon will most likely produce ecchymosis and findings similar to supraspinatus
trauma. Internal impingement is not associated with subscapularis weakness. PREFE RESPON: 2

41. A college basketball player is struck in the eye by a player’s hand while driving to the
basket. Fluorescein evaluation reveals the injury shown in Figure 18. Management should
consist of

1- administration of ophthalmic corticosteroids and antibiotics with application of an eye patch.


2- evaluation of intact visual fields and pupillary responses prior to a return to play.
3- consultation with an ophthalmologist prior to emergent repair of the damaged structure.
4- measurement of ocular pressure and fundoscopic examination in a properly lit examination
room.
5- strict bed rest with the head elevated, minimizing head motion during the healing process.

DISCUSSION: The athlete has a corneal abrasion. Fluorescein staining identifies the break in the
epithelium when examined with ultraviolet light. Topical antibiotics are used as prophylaxis
against secondary bacterial infection, and the patch, applied with the lid closed, is used for comfort
and to promote epithelial healing. The accompanying symptoms, including pain, tearing, and
photophobia, are usually too intense to allow a return to play. Surgery is reserved for a corneal
laceration with associated loss of the anterior chamber. While a proper fundoscopic examination
may be a consideration, increased intraocular pressure is not typically associated with this injury.
20
20
American Academy of Orthopaedic Surgeons
2001 Sports Medicine Self-Assessment Examination by Dr.Dhahirortho

Traumatic hemorrhage in the anterior chamber (hyphema) necessitates strict bed rest during the
early phases of healing; examination will most likely reveal the red fluid level of blood settling
inferiorly in the anterior chamber. It is often associated with increased intraocular pressure.
PREFERRED RESPONSE: 1

42. In patient selection for meniscal allograft transplantation, which of the following
variables has the greatest influence on outcome?

1- Grade of chondromalacia
2- Limb alignment
3- Patient age
4- Patient weight
5- Postoperative level of activity

DISCUSSION: Many clinical studies to date show that the extent of arthritis is the most common
variable that has the greatest influence on outcome. The success rate of allograft transplantation is
significantly diminished in patients who have grade IV chondromalacia of the knee or notable
flattening and general joint incongruity.PREFERRED RESPONSE: 1

43. A 10-year-old boy sustained an injury to the left knee. The radiographic findings
shown in Figure 19 are most commonly associated with injury to which of the following
structures?

1- Anterior cruciate ligament (ACL)


2- Posterior cruciate ligament (PCL)
3- Patellar tendon
4- Lateral capsule
5- Pes anserinus

DISCUSSION: The radiograph shows a bony avulsion of the ACL attachment site on the tibial
spine in this skeletally immature patient. In this age group, injury often results in failure of the
bony attachment site rather than the substance of the ligament. Avulsion of the patellar tendon
insertion site can occur, but this structure is located at the apophysis of the tibial tubercle. The
attachment site of the PCL is much more posterior. In adults, bony avulsion is more commonly
associated with PCL injuries than with ACL injuries. When a small bony avulsion of the lateral
capsule from the lateral tibial plateau is seen on the AP view, this finding is considered
pathognomonic of an ACL injury (Segond sign) in adults. The area of the pes anserinus is anterior
and distal; avulsion would be unusual.

PREFERRED RESPONSE: 1
21
21
22  American Academy of Orthopaedic Surgeons

44. What is the single most important nutritional factor affecting athletic performance?

1- Maximum precompetition carbohydrate stores


2- Adequate carbohydrate consumption during competition
3- Maintenance of adequate serum sodium
4- Maintenance of adequate serum potassium
5- Maintenance of adequate hydration

DISCUSSION: Maintenance of adequate hydration is the single most important factor affecting
athletic performance. While carbohydrate loading may be beneficial for some endurance athletes,
the consumption of carbohydrates during exercise does not appear to be beneficial for athletes
engaged in events that last less than 1 hour. In general, athletes consuming a balanced diet do not
need electrolyte supplementation. PREFERRED RESPONSE: 5

45. A right-handed 20-year-old college baseball pitcher has had a 6-month history of
vague right elbow pain while pitching. Examination reveals full flexion of the elbow and a
loss of only a few degrees of full extension. The elbow is stable, but palpation reveals
tenderness over the olecranon. Plain radiographs are inconclusive. MRI and CT scans are
shown in Figures 20a and 20b. Management should consist of

1- repair of a triceps tendon avulsion.


2- arthroscopy of the elbow for removal of loose bodies.
3- arthroscopic removal of a posteromedial olecranon osteophyte.
4- internal fixation of an olecranon stress fracture.
5- rest, rehabilitation, and resumption of pitching when the fracture is healed.

DISCUSSION: The patient has a stress fracture of the olecranon that occurs with repetitive
throwing motions. If the fracture is not displaced, the initial treatment of choice is rest and
rehabilitation to maintain elbow motion, followed by aggressive strengthening at 6 to 8 weeks. A
light throwing program generally can begin at 8 to 12 weeks. Complete recovery may require 3 to 6
months. If the fracture is displaced or if nonsurgical management fails, surgery is indicated for
internal fixation of the stress fracture. PREFERRED RESPONSE: 5

46. What is the most common associated pathology in patients who have suprascapular
nerve entrapment secondary to ganglion cysts?

1- Glenohumeral arthritis
2- Fracture of the clavicle
3- Tear of the rotator cuff
4- Rupture of the long head of the biceps tendon
5- Superior labrum anterior and posterior (SLAP) lesion
22
22
American Academy of Orthopaedic Surgeons
2001 Sports Medicine Self-Assessment Examination by Dr.Dhahirortho

DISCUSSION: It is well known that suprascapular nerve entrapment can be secondary to many
entities, and its association with ganglion cysts and SLAP lesions has been well documented.
Because of a superior labral tear, synovial fluid will leak out of the joint underneath the labrum,
causing the cyst and secondary compression of the nerve. PREFERRED RESPONSE: 5

47. A 27-year-old runner training for his first marathon reports lateral knee pain after an
unusually long training run. He states that the most significant pain occurs while running
downhill. Examination of the patient while he is laying on the unaffected side reveals
increased pain when manual pressure is applied to the lateral femoral epicondylar area
during knee range of motion of 30° to 45°. What is the most likely diagnosis?

1- Popliteal tendinitis
2- Iliotibial band friction syndrome
3- Excessive lateral pressure syndrome
4- Lateral meniscal tear
5- Stress fracture

DISCUSSION: Iliotibial band friction syndrome is one of the most common causes of lateral knee
pain in runners. It is caused by increased friction between the iliotibial band and the lateral femoral
condyle because of increased tension on the lateral structures. It may be caused by a prominence of
the lateral epicondyle or a malalignment of the lower extremity in the runner, including genu
varum, tibia vara, heel varus and forefoot supination, or compensating pronation. These structural
characteristics can couple with relative muscle imbalance and lead to an altered running gait,
enhancing friction between the lateral femoral condyle and the iliotibial band. Management is
usually nonsurgical, including stretching of the iliotibial band and strengthening of the hip abductor
muscles, with occasional use of cortisone injections or iontophoresis. PREFERRED RESPONSE: 2

48. A 30-year-old woman who runs approximately 30 miles a week has had right hip and
groin pain for the past 3 weeks. Examination reveals an antalgic gait, limited motion of the
right hip, and pain, especially with internal and external rotation. Plain radiographs are
normal, and an MRI scan is shown in Figure 21. Management should consist of

1-immediate internal fixation of the right femoral neck stress fracture.


2-non-weight-bearing crutch ambulation until symptoms resolve, followed by a gradual resumption
of activities.
3-ultrasound therapy to promote fracture healing.
4-a metabolic work-up.
5-a bone scan to look for other stress fractures.

DISCUSSION: A stress fracture of the hip is a


relatively common problem in endurance sports. These
fractures are classified as compression-side, tension-
side, and displaced femoral neck fractures. The MRI
scan shows a compression-side stress fracture.
Compression-side fractures usually occur in the inferior
or calcar area of the proximal femur, and non-weight-
bearing crutch ambulation for 6 to 7 weeks will most
likely result in healing. Once the patient is walking
without pain or a limp, activities can be slowly
increased. Because tension-side fractures have a high
risk of displacement, treatment should consist of
23 immediate internal fixation. PREFERR RESPONSE: 2
23
24  American Academy of Orthopaedic Surgeons

49. Which of the following primary prognostic factors best predicts the outcome of the
knee lesion shown in Figure 22?

1- Location
2- Size
3- Knee stability
4- Patient age
5- Degree of pain

DISCUSSION: The patient has osteochondritis


dissecans. While location, size, and knee stability
are all relevant to the overall prognosis, studies
have shown that younger patients with open
growth plates have a better prognosis of healing
when compared with patients who have closed
growth plates. The degree of pain is also relevant
to treatment, but it is subjective rather than objective and is not as reliable of a prognostic indicator
as age.PREFERRED RESPONSE: 4

50. Figures 23a and 23b show the AP and lateral radiographs of the elbow of a 30-year-old
professional pitcher. The pathology shown in these studies is most consistent with which of
the following conditions?

1- Insertional triceps tendinitis


2- Valgus extension overload
3- Medial epicondylitis
4- Stress fracture of the olecranon
5- Chronic olecranon bursitis

DISCUSSION: The radiographs show the osteophytic build-up of the posteromedial corner of the
elbow that occurs with valgus extension overload in the pitching elbow. This is the result of
excessive valgus forces during the acceleration and deceleration phases of throwing. These forces,
coupled with medial elbow stresses, cause a wedging of the olecranon into the medial wall of the
olecranon fossa. Valgus instability of the elbow may further stimulate osteophyte formation.
Repetitive impact of a spur within the olecranon fossa may cause fragmentation and eventual
24 formation of loose bodies. PREFERRED RESPONSE: 2

24
American Academy of Orthopaedic Surgeons
2001 Sports Medicine Self-Assessment Examination by Dr.Dhahirortho

51. Figure 24 shows the radiograph of a 10-year-old boy who sustained a valgus injury to
the knee. Examination reveals grade III medial laxity. Initial management should consist of

1- an MRI scan.
2- stress radiographs of the knee.
3- activities as tolerated.
4- a hinged range-of-motion brace.
5- a knee immobilizer.

DISCUSSION: Based on the mechanism of injury and findings of


medial laxity, the most likely diagnosis is injury to either the
growth plate or the medial collateral ligament. With the open
physeal plate, this area of injury is presumed present until proven
otherwise; therefore, stress radiographs should be obtained before
implementing any treatment or ordering more extensive and
expensive tests. PREFERRED RESPONSE: 2

52. A right-handed 14-year-old pitcher has had a 3-month history of shoulder pain while
pitching. Examination reveals full range of motion, a mildly positive impingement sign, pain
with rotational movement, and no instability. Plain AP radiographs of both shoulders are
shown in Figures 25a and 25b. Management should consist of

1- referral to a pitching coach to improve throwing mechanics.


2- a weight-training program that concentrates on rotator cuff strengthening.
3- rest until symptoms have resolved, followed by a gradual return to pitching.
4- a metabolic work-up.
5- cessation of pitching until the physis is closed.

DISCUSSION: The patient has the classic signs of Little Leaguer’s shoulder, with findings that
include pain localized to the proximal humerus during the act of throwing and radiographic
evidence of widening of the proximal humeral physis. Examination usually reveals tenderness to
palpation over the proximal humerus, but the presence of any swelling, weakness, atrophy, or loss
of motion is unlikely. The treatment of choice is rest from throwing for at least 3 months, followed
by a gradual return to pitching once the shoulder is asymptomatic. PREFERRED RESPONSE: 3
25
25
26  American Academy of Orthopaedic Surgeons

53. A 38-year-old man sustains a complete avulsion with retraction of the ischial
attachment of the hamstring muscles in a fall while water skiing. He indicates that he is an
aggressive athlete who participates regularly in multiple running and cutting-type sports, and
he strongly desires to continue his athletic competition. Management should
consist of

1- ultrasound, iontophoresis, and stretching, with an early return to sports.


2- a local corticosteroid injection and strengthening, with a delayed return to sports.
3- immobilization and rehabilitation, with a delayed return to sports.
4- early surgical repair, prolonged rehabilitation, and a return to sports.
5- rehabilitation, with delayed surgical repair if the patient is unable to return to sports.

DISCUSSION: Several studies have identified a complete proximal avulsion of the hamstring
muscles as an injury that leads to significant long-term disability, with a high percentage of athletes
who must permanently restrict their activities following nonsurgical management. Early surgical
repair and prolonged rehabilitation have yielded consistently better results than nonsurgical
management. PREFERRED RESPONSE: 4

54. What mechanism contributes to strength gains during conditioning of the


preadolescent athlete?

1- Enhanced neurogenic adaptations


2- Advanced myogenic adaptations
3- Increased contractile proteins
4- Increased short-term energy sources
5- Thickening of the connective tissue

DISCUSSION: Prepubescent athletes gain strength through neurogenic adaptations, including


recruitment of motor units, reduced inhibition, and learned motor skills. Myogenic adaptations
(muscle hypertrophy) occur after puberty and include increased contractile proteins, thickening of
the connective tissue, and increased short-term energy sources such as creatine phosphate.
PREFERRED RESPONSE: 1

55. Following an episode of transient quadriplegia in contact sports, an athlete’s return to


play is absolutely contraindicated when

1- the spinal canal to vertebral body ratio (Torg ratio) is less than or equal to 0.8.
2- electromyelographic studies are abnormal.
3- MRI scans or contrast-enhanced CT scans show severe spinal stenosis.
4- unilateral burning pain persists.
5- the episode of quadriplegia lasts 5 minutes.

DISCUSSION: Return to play decisions after traumatic spinal or spinal cord injury are not always
clear-cut and often must be made on a patient-by-patient basis. The Torg ratio has been found to
have low sensitivity in patients with large vertebral bodies. Abnormal electromyographic studies
can persist in the face of normal function and do not define spinal injury. Duration of quadriplegia
is not related to anatomic pathology. Findings on MRI scans or contrast-enhanced CT scans
consistent with stenosis include lack of a significant cerebrospinal fluid signal around the cord,
bony or ligament hypertrophy, or disk encroachment. Based on these findings, return to play should
be avoided. PREFERRED RESPONSE: 3

26
26
American Academy of Orthopaedic Surgeons
2001 Sports Medicine Self-Assessment Examination by Dr.Dhahirortho

56. A 16-year-old snowboarder has significant pain and is still unable to bear weight after
sustaining a lateral ankle injury in a fall 1 week ago. Examination reveals swelling and
tenderness in the sinus tarsi. AP, lateral, and mortise radiographs of the ankle are
unremarkable. Management should consist of

1- an elastic bandage, cold packs, and weight bearing as tolerated.


2- non-weight-bearing and a CT scan of the talus.
3- cast immobilization for 10 days, followed by progressive rehabilitation.
4- cast immobilization for 6 weeks, followed by progressive rehabilitation.
5- stirrup splinting, cold packs, and aggressive rehabilitation.

DISCUSSION: Because there is a significant possibility that the patient may have a fracture of the
lateral process of the talus, there is some disagreement as to the best radiographic study to identify
this injury. A CT scan is an appropriate diagnostic tool to visualize the fracture and identify any
displacement. Displaced lateral process fractures are best treated surgically. PREFER RESP: 2

57. A 24-year-old man who plays golf noted the immediate onset of pain on the ulnar side
of his hand and has been unable to swing a club for the past 6 weeks after striking a tree root
with his club during his golf swing. Examination reveals full motion of the wrist, diminished
grip strength, and tenderness over the hypothenar region. A CT scan of the hand and wrist is
shown in Figure 26. Management should consist of

1-immobilization of the wrist until the fracture heals.


2-excision of the hook of the hamate.
3-internal fixation of the fractured hook of the hamate.
4-ultrasound therapy to promote fracture healing.
5-limited intercarpal arthrodesis.

DISCUSSION: Fractures of the hook of the hamate


frequently are not identified in the acute phase.
Because the fracture can be difficult to see on plain
radiographs, the lack of findings can lead to a painful
nonunion. A carpal tunnel view may show the fracture,
but a CT scan will best detect the injury.
Immobilization is the treatment of choice and will result
in union in most patients unless the diagnosis is delayed. However, excision of the fragment may
be necessary for patients who have nonunion, persistent pain, or ulnar nerve palsy.

PREFERRED RESPONSE: 1

58. An 18-year-old football player sustains a contact injury to his right lower leg, and
radiographs show a closed transverse fracture of the middle third of the tibia. Based on the
clinical examination, a compartment syndrome is suspected. When measuring compartment
pressures, the highest tissue pressure is recorded how many centimeters proximal or distal to
the fracture site?

1- 0 cm to 5 cm
2- 5 cm to 10 cm
3- 10 cm to 15 cm
4- 15 cm to 20 cm
5- Greater than 20 cm
27
27
28  American Academy of Orthopaedic Surgeons

DISCUSSION: Measurements of compartment pressures in patients with tibial fractures and


compartment syndrome reveal that the highest tissue pressures are recorded at the level of the
fracture or within 5 cm of the fracture. Tissue pressures show a statistically significant decrease
when they are recorded at increasing distances proximal and distal to the site of the highest pressure
recorded. To reliably determine the location of the highest tissue pressure in patients with tibial
fractures, measurements should be obtained, at a minimum, in both the anterior and deep posterior
compartments at the level of the fracture, as well as at locations proximal and distal. The highest
tissue pressure recorded should serve as a basis for determining the need for fasciotomy.
PREFERRED RESPONSE: 1

59. A 50-year-old patient who plays tennis sustained the deformity shown in Figure 27
following a high volley. Further diagnostic work-up should include

1-an electromyogram (EMG) of the upper extremity.


2-an ultrasound of the short head of the biceps.
3-an MRI scan of the rotator cuff.
4-a CT scan with contrast of the anterior labrum.
5-a subclavian venogram.

DISCUSSION: The patient has a rupture of the long head of the biceps; however, patients older
than age 45 years are at greater risk of having an associated rotator cuff tear. An MRI scan should
be ordered to avoid missing concomitant rotator cuff pathology. While patients may report pain
radiating down the arm at the time of the tendon rupture, an EMG is not indicated. The short head
of the biceps is intact and needs no further work-up, even though the muscle descends in most
cases. The anterior labrum can be injured but is not associated with this deformity. PR RE: 3

60. A 16-year-old ice hockey player is struck on the chest by the puck. He skates a few
strides and then collapses. What is the most likely diagnosis?

1- Acute aortic dissection


2- Pulmonary contusion
3- Commotio cordis
4- Acute cardiac tamponade
5- Splenic rupture

DISCUSSION: Sudden cardiac arrest following a blow to the chest in young athletes has been
termed “commotio cordis.” It is most common in Little League and other youth projectile sports
(eg, ice hockey, lacrosse). The cause, although not completely determined, is most likely an
arrhythmia related to the impact in a vulnerable time in the cardiac cycle. Resuscitation has proven
to be exceedingly difficult, resulting in a high mortality rate.

PREFERRED RESPONSE: 3

28
28
American Academy of Orthopaedic Surgeons
2001 Sports Medicine Self-Assessment Examination by Dr.Dhahirortho

61.A 24-year-old dancer sustains the injury shown in Figure 28. Management should
consist of

1- closed reduction and application of a well-molded cast.


2- open reduction and percutaneous pin fixation.
3- open reduction and internal fixation with a mini
fragment plate and screws.
4- intramedullary screw fixation.
5- brief immobilization and symptomatic treatment.

DISCUSSION: The patient has a moderately displaced distal


diaphyseal fracture of the fifth metatarsal, and the most appropriate
treatment is brief immobilization and symptomatic management.
Attempts at closed reduction are unlikely to appreciably alter the
position of the fracture. Surgical techniques for either reduction of the
fracture or fixation have not been shown to result in improved
functional outcomes. PREFERRED RESPONSE: 5

62. A 22-year-old volleyball player has atrophy of the infraspinatus muscle. This deficit is
the result of entrapment of what nerve?

1- Axillary nerve in the posterolateral space


2- Dorsal scapular nerve at the medial border of the scapula
3- Suprascapular nerve in the scapular notch
4- Suprascapular nerve in the spinoglenoid notch
5- Subscapular nerve at the rotator interval

DISCUSSION: Suprascapular deficits, as the result of repetitive forceful internal rotation with
overhead ball striking, occur in the spinoglenoid notch. Compression interferes with distal
suprascapular nerve innervation to the infraspinatus, while allowing the supraspinatus to function
normally. A scapular notch entrapment of this nerve would involve both the supraspinatus and the
infraspinatus. The axillary, dorsal scapular, and subscapular nerves do not affect the infraspinatus.
PREFERRED RESPONSE: 4

63. Figure 29 shows the radiograph of a 25-year-old woman who has had a 3-month
history of ankle pain after sustaining an inversion injury to the ankle. She reports occasional
catching, but no sense of instability. Examination reveals ligament stability. Management
should consist of

1-a non-weight-bearing short leg cast.


2-open reduction and internal fixation.
3-no weight bearing with motor exercises for 8 weeks.
4-debridement, curettage, and drilling.
5-an ankle brace or taping when participating in athletic activity.

29
29
30  American Academy of Orthopaedic Surgeons

DISCUSSION: Osteochondral lesions of the talar dome can have a traumatic or nontraumatic
etiology. Most authors site a probable traumatic etiology for lateral lesions. Stage I and II lesions,
which are composed of compressed subchondral bone or a partial detached osteochondral fragment,
can be treated initially in a non-weight-bearing short leg cast for 6 weeks. Stage III medial lesions
can also be treated in the same manner. If symptoms persist, the treament of choice is debridement
of the fracture, curettage of the lesion, and drilling of the subchondral bone. This treatment also
applies to lateral stage III and all stage IV lesions. If the fragment is at least one third of the size of
the talar dome, management should consist of open reduction and internal fixation. In patients with
more chronic lesions (4 to 6 months of persistent pain), the threshold to proceed with surgery is
lower, even in a stage II lesion. PREFERRED RESPONSE: 4

64. A 19-year-old college cross-country runner is amenorrheic and has recurrent stress
fractures. Long-term management should consist of

1- cross training with swimming and cycling.


2- a complete cessation of running.
3- vitamin D and calcium supplements.
4- increased caloric intake.
5- oral contraceptives, vitamin D, and calcium supplements.

DISCUSSION: The triad of menstrual dysfunction, disordered eating, and stress fracture is well
recognized in women who participate in endurance sports. The best treatment remains to be
determined, but at present, the combination of oral contraceptives to regulate menses, an increased
intake of calcium and vitamin D, as well as nutritional counseling, is the recommended treatment
for decreased bone mass related to exercise-induced amenorrhea. PREFERRED RESPONSE: 5

65. A 47-year-old male tennis player has pain in his nondominant shoulder that has failed
to respond to 4 months of nonsurgical management. Examination reveals acromial
tenderness and pain at the supraspinatus tendon insertion. He has a positive impingement
sign, pain on forward elevation, and minimal cuff weakness. The MRI scans are shown in
Figures 30a and 30b. To completely resolve his symptoms, treatment should consist of

1- rigid open reduction and internal fixation of the os acromiale with autologous bone graft.
2- arthroscopic repair of the rotator cuff and acromioplasty.
3- arthroscopic excision of the os acromiale.
4- arthroscopic decompression of the supraglenoid cyst.
5- open distal clavicle excision (Mumford procedure).
30
30
American Academy of Orthopaedic Surgeons
2001 Sports Medicine Self-Assessment Examination by Dr.Dhahirortho

DISCUSSION: The MRI scans show a mesoacromion with tendonopathy of the supraspinatus. The
history and physical findings indicate that the patient has a symptomatic os acromiale. Simple
excision of the unstable os acromiale has not yielded consistently good results. Meticulous internal
fixation using tension banding with cannulated screws and autologous bone grafting has shown
good results for this problem.PREFERRED RESPONSE: 1

66. A 39-year-old competitive cyclist sustains an injury to her left hip in a fall.
Gadolinium arthrography, with an accompanying MRI scan, is shown in Figure 31. A cleft,
or defect, identified by the arrow, indicates a detachment of the

1- acetabular labrum.
2- zona orbicularis.
3- iliofemoral ligament.
4- acetabular pulvinar.
5- retinacular vessels.

DISCUSSION: The area indicated by the arrow represents gadolinium contrast extending into a
separation between the lateral labrum and its acetabular attachment. This can be a traumatic
detachment, but occasionally a cleft may be present as a normal variant of the labral morphology.
The capsular attachment of the iliofemoral ligament is peripheral to the labrum. The pulvinar is the
common name applied to the fat and overlying synovium contained within the acetabular fossa
above the ligamentum teres. The zona orbicularis is a circumferential thickening of the capsule
around the femoral neck, and the retinacular vessels travel within the capsular synovium up the
femoral neck to supply the femoral head.

PREFERRED RESPONSE: 1

67. A cortisone injection in the subacromial space will most likely result in

1- elevated blood glucose levels in patients with diabetes.


2- increased instability in multidirectional patients.
3- accelerated rupture of the long head of the biceps.
4- accelerated osteoporosis of the tuberosity.
5- altered proprioception of the glenohumeral joint.

DISCUSSION: A cortisone injection in the subacromial space will most likely result in elevated
blood glucose levels in patients with type I diabetes mellitus. Patients should be warned of this
potential complication. Cortisone does not have an effect on instability or proprioception, and a
single injection would not affect osteoporosis. Repetitive injections or injection into the tendon
itself could accelerate rupture of the biceps tendon.

PREFERRED RESPONSE: 1

31
31
32  American Academy of Orthopaedic Surgeons

68. A high school athlete reports the sudden onset of low back pain while performing a
dead lift. Examination reveals a lumbar paraspinal spasm and a positive straight leg raising
test. The deep tendon reflexes, motor strength, and sensation in the lower extremeties are
normal. The radiographs are normal. If symptoms persist for more than a few weeks,
management should consist of

1- an electromyogram and nerve conduction velocity studies.


2- an MRI scan.
3- a bone scan.
4- physical therapy.
5- bed rest.

DISCUSSION: In adolescents, a lumbar herniated disk is characterized by a paucity of clinical


findings; a positive straight leg raising test may be the only consistent positive finding. This may
result in a long period of nonsurgical management that fails to provide relief. Activities that place a
significant shear load on the lumbar spine, such as the dead lift, are associated with an increased
risk of central disk herniation. When an adolescent who lifts weights has a history of low back pain
that fails to respond to a short period of active rest, an MRI scan is the study of choice to evaluate
for a lumbar herniated disk. PREFERRED RESPONSE: 2

69. A 22-year-old skier reports painful range of motion in the left thumb after falling
forward on his outstretched hand while holding his ski pole. Examination of the left thumb
reveals increased AP laxity and 45° of valgus laxity at the metacarpophalangeal (MCP) joint.
Examination of the right thumb shows 25° of valgus laxity at the MCP joint. Radiographs are
normal. Management should consist of

1- primary repair of the ulnar collateral ligament.


2- volar plate arthroplasty.
3- pinning of the MCP joint for 6 weeks.
4- a thumb spica cast.
5- a hand-based thumb spica splint.

DISCUSSION: The patient has a complete tear of the ulnar collateral ligament as defined by MCP
joint laxity of greater than 30° (or 15° greater laxity compared with the opposite side). Primary
repair is the treatment of choice because displacement of the ligament superficial to the adductor
aponeurosis (Stener lesion) must be corrected. Any volar plate injury can be addressed during
repair of the ulnar collateral ligament. PREFERRED RESPONSE: 1

70. Which of the following structures is most commonly involved in lateral epicondylitis?

1- Anconeus
2- Extensor digitorum communis
3- Extensor carpi radialis longus
4- Extensor carpi radialis brevis
5- Extensor carpi ulnaris

DISCUSSION: The most common specific site of involvement is the origin of the extensor carpi
radialis brevis. It is usually caused by overuse activities, such as the eccentric overload exhibited
during a backhand in tennis. In most patients, the characteristic friable, grayish tissue described as
angiofibroblastic hyperplasia or hyaline degeneration is seen at the extensor carpi radialis brevis
origin. PREFERRED RESPONSE: 4
32
32
American Academy of Orthopaedic Surgeons
2001 Sports Medicine Self-Assessment Examination by Dr.Dhahirortho

71. When comparing surgical and nonsurgical extremities in patients who underwent
anterior cruciate ligament (ACL) reconstruction using patellar tendon or hamstrings
autografts, isokinetic strength measurements obtained 6 months after the surgery would most
likely reveal

1- significant quadricep weakness in the pateller tendon compared with the hamstring.
2- significant quadricep weakness in the hamstring compared with the pateller tendon.
3- significant weakness in the hamstring compared with the patellar tendon.
4- significant hamstring weakness in the pateller tendon compared with the hamstring.
5- no significant difference between the hamstring and the pateller tendon.

DISCUSSION: Follow-up examination at 6 months revealed no statistically significant differences


in quadricep or hamstring strength when comparing surgical versus nonsurgical extremities
isokinetically. Therefore, the selection of autogenous hamstring or patellar tendon for ACL
reconstruction should not be based solely on the assumption of the graft tissue source altering the
recovery of quadricep and/or hamstring strength.

PREFERRED RESPONSE: 5

72. A quarterback sustains a rough tackle after which he appears confused, has a dazed
look on his face and an unsteady gait on standing. He denies loss of consciousness.
Reexamination within 10 minutes is normal, the patient is lucid, and he wants to return to
play. The coach and the player should be advised that he may

1- return to play immediately.


2- return to play in 1 week, if asymptomatic.
3- return to play in 1 month, if asymptomatic.
4- return only after a screening CT scan.
5- not return to play for the season.

DISCUSSION: The patient has a grade I (mild) concussion that can result in confusion and
disorientation, without loss of consciousness. This concussion syndrome is completely reversible,
with no long-term sequelae. Athletes who sustain a grade I concussion may return to play after 15
minutes if there are no lingering symptoms, such as headache or vertigo. A grade II concussion is
characterized by loss of consciousness of less than 5 minutes. With this type of injury, the athlete
can return to play in 1 week, if asymptomatic. If a grade III (severe) concussion is sustained, the
athlete should avoid contact for a minimum of 1 month before considering a return to competition.
A grade III concussion is characterized by a loss of consciousness of greater than 5 minutes or
posttraumatic amnesia of greater than 24 hours. A CT scan is not indicated in a grade I injury. An
athlete who sustains three grade I or grade II concussions, or two grade III concussions may not
return to play for the season..

33 PREFERRED RESPONSE: 1
33
34  American Academy of Orthopaedic Surgeons

73. The bone avulsion shown in Figure 32 has a high correlation with tearing of the

1- iliotibial band.
2- anterior cruciate ligament.
3- posterior cruciate ligament.
4- lateral collateral ligament.
5- biceps femoris tendon.

DISCUSSION: As described by Segond in 1987, an avulsion fracture of the lateral tibial plateau is
commonly referred to as a Segond fracture. Subsequent to 1987, several authors have also found
that the lateral capsular sign represents, but is not limited to, a disruption of the middle third of the
lateral capsule and a tear of the anterior cruciate ligament. PREFERRED RESPONSE: 2

74. A 21-year-old college defensive lineman sustains a minimally displaced (less than 1
mm) midthird scaphoid fracture during the first game of the season. Management should
consist of

1- cast immobilization and a return to play as symptoms allow.


2- cast immobilization and a return to play when union is achieved.
3- open reduction and internal fixation, followed by early range of motion with a return to play
when union is achieved.
4- open reduction and internal fixation, followed by a return to play with protective casting.
5- symptomatic treatment, with definitive treatment at the end of the season.

DISCUSSION: The union rate for minimally displaced midthird scaphoid fractures is quite high
with cast immobilization while allowing a return to sports. Inadequate immobilization results in a
much higher nonunion rate. Early fixation and rehabilitation have been proposed for sports or
positions that are not amenable to cast immobilization. While immobilization of a nondisplaced
fracture results in an acceptably high union rate, there is no advantage to fixation in conjunction
with immobilization in the course of healing. With adequate immobilization and protection, play
restrictions until healing has occurred are unnecessary.

PREFERRED RESPONSE: 1

34
34
American Academy of Orthopaedic Surgeons
2001 Sports Medicine Self-Assessment Examination by Dr.Dhahirortho

75. A 16-year-old football player sustains a direct blow to the anterior aspect of his flexed
right knee. Examination reveals a contusion over the anterior tibial tubercle and a
small effusion. MRI scans are shown in Figures 33a through 33c. What is the most likely
diagnosis?

1- Partial tear of the patellar tendon


2- Osteochondral fracture of the femur
3- Anterior cruciate ligament (ACL) tear
4- Posterior cruciate ligament (PCL) tear
5- Patella fracture

DISCUSSION: The MRI scans show disruption of the fibers of the PCL. Patients sustaining an
isolated acute PCL injury can present with only minimal discomfort and have full range of motion.
When examination reveals a contusion over the tibial tubercle and discomfort with the posterior
drawer examination, with or without instability, a possible injury to the PCL should be considered.
In acute injuries, the reported accuracy of MRI imaging for diagnosing PCL tears ranges from 96%
to 100%. PREFERRED RESPONSE: 4

76. A 15-year-old diver has had persistent, activity-related low back pain for the past 2
months. He denies any history of trauma. Examination reveals that the pain is localized to
the lumbosacral junction, and there are no radicular symptoms. The pain is worse with back
extension. Neurologic examination is normal, as are AP, lateral, and oblique radiographs of
the lumbosacral spine. Further evaluation should include

1- flexion and extension radiographs of the lumbosacral spine.


2- diskography.
3- an MRI scan of the lumbosacral spine.
4- a bone scan with single proton emission computed tomography (SPECT).
5- a renal ultrasound.

DISCUSSION: Spondylolysis may develop as a stress fracture resulting from repetitive


hyperextension during athletic activities. In young people, the pars interarticularis is thin, the
35
35
36  American Academy of Orthopaedic Surgeons

neural arch has not yet reached maximum strength, and the intravertebral disk is less resistant to
shear. While clinical symptoms may lead to the suspicion of spondylolysis, radiographic
confirmation may be difficult in early cases. Plain radiographs may be negative initially, and the
plain MRI scan may not offer good visualization of the pars. A bone scan with SPECT is very
sensitive initially. CT scans with regular axial and reverse-gantry angled cuts may help determine
the type of fracture and the course of treatment.

PREFERRED RESPONSE: 4

77. A 23-year-old college basketball player reports persistent lateral ankle pain after
sustaining an inversion injury 6 months ago. Examination reveals pain over the anterolateral
ankle, absence of swelling, and no clinical instability. Management consisting of vigorous
physical therapy fails to provide relief, and a intra-articular corticosteroid injection provides
only temporary relief. Radiographs obtained at the time of injury and subsequent AP and
varus stress views are normal. A recent MRI scan fails to show any abnormalities.
Management should now include

1- cast immobilization.
2- arthroscopy.
3- continued physical therapy.
4- a repeat corticosteroid injection.
5- a short course of oral steroids.

DISCUSSION: Because the patient has failed to respond to appropriate nonsurgical management
and imaging studies are normal, the use of arthroscopy not only aids in the diagnosis of chronic
ankle pain, but is also helpful in its treatment. In patients with this condition, typical findings
include synovitis in the lateral gutter and fibrosis along the talofibular articulation; syndesmosis
chondromalacia of the talus and ankle also may be found. In patients with anterior soft-tissue
impingement, approximately 84% who have a poor response to nonsurgical management will have
a good to excellent response after arthroscopic synovectomy and debridement.

PREFERRED RESPONSE: 2

78. Which of the following tissues used for anterior cruciate ligament (ACL)
reconstruction has the highest maximum load to failure?

1- Allograft ACL
2- Bone-patellar tendon-bone with a width of 10 mm
3- Fascia lata with a width of 15 mm
4- Central quadriceps tendon with a width of 15 mm
5- Quadruple semitendinosus and gracilis tendons

DISCUSSION: While the patellar tendon ligament is considered by many to be the tissue of choice
for ACL reconstruction, more recent studies have shown that the quadruple semitendinosus and
gracilis tendon graft has the greatest stiffness and offers the highest maximum load to failure.

PREFERRED RESPONSE: 5

36
36
American Academy of Orthopaedic Surgeons
2001 Sports Medicine Self-Assessment Examination by Dr.Dhahirortho

79. Creatine is currently being used by athletes as a dietary supplement in an attempt to


enhance performance. What is the physiologic basis for its use?

1-Assists in carbohydrate metabolism and glycogen synthesis by producing adenosine diphosphate


(ADP) to enhance aerobic activities
2-Converts to phosphocreatine (PCr), which acts as an energy reservoir for adenosine triphosphate
(ATP) in muscle tissue
3-Converts to PCr, which enhances the production of ADP and promotes the metabolism of
triglycerides as an energy source
4-Converts to PCr, which enhances the production of ADP and promotes the metabolism of proteins
as an energy source
5-Converts to ADP by creatine kinase (CK) providing an energy reservoir for the production of
ATP

DISCUSSION: Creatine is currently used as a nutritional supplement in an attempt to enhance


athletic performance. The physiologic basis for its use is based on its conversion by CK to PCr,
which acts as an energy reservoir in muscle cells for the production of ATP. A number of studies
that examined the effect of creatine supplementation on performance concluded that while creatine
does not increase peak force production, it can increase the amount of work done in the first few
anaerobic short duration, maximal effort trials. The mechanism for this enhancement of work is
unknown, but it is most likely secondary to the increase in the available PCr pool. PREF RESPO: 2

80. A 16-year-old high school football player sustains an injury to the left hip. The avulsed
fragment identified by the arrow in Figure 34 represents the origin of which of the following
structures?

1- Ischiofemoral ligament
2- Pubofemoral ligament
3- Rectus femoris
4- Sartorius
5- Gluteus minimus

DISCUSSION: The avulsed fragment represents the origin of the rectus femoris from the anterior
inferior iliac spine and the brim of the acetabulum. Avulsion of the anterior inferior iliac spine is
much less common than avulsion of the anterior superior iliac spine with its origin of the sartorius.
The origin of the gluteus minimus is from the outer cortex of the iliac wing and has not been
reported as a source of bony avulsion. The hip capsule is composed of the ischiofemoral and
pubofemoral ligaments, in addition to the iliofemoral ligament. The pelvic attachment of the
ischiofemoral ligament is from the ischial part of the acetabulum posteriorly, while the pubofemoral
ligament attaches to the pubic portion inferiorly. Technically, ligaments do not have origins and
insertions as muscle tendon groups do, but have attachment sites.

PREFERRED RESPONSE: 3

37
37
38  American Academy of Orthopaedic Surgeons

81. Which of the following methods of meniscal repair has the highest load to
failure strength?
1- Horizontal suture
2- Vertical suture
3- Mulberry knot
4- T-fix suture
5- Meniscal arrow

DISCUSSION: Numerous experimental studies have shown that vertical suture techniques are
superior to all of the other noted methods. In fact, vertical sutures have been shown to be twice as
strong as several of these techniques. PREFERRED RESPONSE: 2

82. Figure 35 shows the lateral radiograph of a 15-year-old basketball player who felt a
dramatic pop in his knee when landing after a lay-up. The patient reports that he cannot
bear weight on the injured extremity. Management should consist of

1-closed reduction and casting in extension.


2-open reduction with suture fixation of the proximal fragment.
3-closed reduction, followed by functional bracing.
4-open reduction and internal fixation with screws and complete proximal
tibial epiphysiodesis.
5-open reduction and internal fixation with screws.

DISCUSSION: Tibial tubercle avulsion is an injury of the adolescent knee


that most often occurs just before the end of growth. The fracture usually
occurs with jumping, either at push-off or landing. This patient has a type
III injury. In type III injuries, the articular surface is disrupted, and
meniscal injury and compartment syndrome can occur. Open reduction is
the treatment of choice, and anterior fasciotomy should be considered
prophylactically at the time of surgery. Although the fracture heals with
an anterior epiphysiodesis of the proximal tibia, little growth remains in
this patient and no special handling of the physis is warranted.PREFERRED RESPONSE: 5

83. A 52-year-old man has pain in the sternal area after landing on his right shoulder in a
fall from his bicycle. In addition, he reports that he had difficulty swallowing and breathing
immediately after the fall, but the symptoms resolved. A CT scan reveals a posterior
sternoclavicular dislocation. Initial management should include

1- a snug figure-of-8 splint and observation for spontaneous reduction.


2- closed reduction under general anesthesia.
3- closed reduction under general anesthesia and percutaneous pinning.
4- open reduction and capsuloligamentous repair.
5- open reduction and wire stabilization of the joint.

DISCUSSION: Posterior sternoclavicular dislocations require rapid treatment because of the


proximity of major neurovascular structures and the airway. Initial management should consist of
closed reduction under general anesthesia in the operating room with a chest surgeon available. A
successful closed reduction is usually stable. Internal fixation of sternoclavicular injuries should be
avoided because of the likelihood of hardware migration and possible injury to the mediastinal
structures. If closed reduction is unsuccessful, open reduction is indicated. Treatment following
reduction of the sternoclavicular joint includes the application of a figure-of-8 splint and a sling for
38 6 weeks, followed by stretching and strengthening exercises. PREFERRED RESPONSE: 2

38
American Academy of Orthopaedic Surgeons
2001 Sports Medicine Self-Assessment Examination by Dr.Dhahirortho

84. What nerve is at greatest risk of harm from the portal shown in Figure 36?

1- Radial
2- Ulnar
3- Median
4- Lateral antebrachial cutaneous
5- Posterior antebrachial cutaneous

DISCUSSION: The figure shows the anterolateral portal for elbow arthroscopy, and injury to the
radial nerve has been reported in conjunction with this portal site. Studies have shown that closer
proximity to the radial nerve is associated with more distal portal sites. The lateral and posterior
antebrachial cutaneous nerves are both at less risk of injury. The ulnar and median nerves are both
fairly remote to this location. PREFERRED RESPONSE: 1

85. In the majority of patients with chronic anterior cruciate ligament (ACL)-deficient
knees, analysis of the gait pattern during level walking will most likely reveal which of the
following changes?

1-No significant differences in gait from the contralateral knee


2-No change in knee flexion-extension moment with balancing of quadriceps and hamstring activity
3-A change in knee flexion-extension moment with decreased hamstring activity
4-A change in knee flexion-extension moment with decreased demand on the quadriceps and a net
increase in hamstring activity
5-A change in knee flexion-extension moment with increased demand on the quadriceps

DISCUSSION: Patients with chronic ACL-deficient knees typically have lower than normal net
quadriceps activity during the middle portion of the stance phase; the net moment about the knee
reverses from one that demands quadriceps activity to one that demands increased hamstring
activity. This type of gait is termed “quadriceps avoidance.” This avoidance is believed to be a
functional adaptation to reduce anterior tibial translation, and it is most prevalent as the knee moves
from 45° of flexion toward full extension, the arc of motion through which the ACL is most
responsible for stability. PREFERRED RESPONSE: 4

86. Glenohumeral inferior stability in the adducted shoulder position is primarily a


function of the
1- rotator cuff.
2- posterior glenohumeral ligament.
3- long head of the biceps tendon.
4- inferior glenohumeral ligament complex.
5- superior glenohumeral ligament.

DISCUSSION: When the arm is adducted, the superior structures, including the superior
glenohumeral ligament, are responsible in limiting the inferior translation. With the arm abducted,
the inferior glenohumeral ligament complex is responsible for limiting inferior subluxation. Rotator
cuff activity can actually depress the humeral head and does not play a role in preventing inferior
subluxation. The long head of the biceps and the posterior glenohumeral ligament do not play a
39 role in protecting the shoulder from inferior instability. PREFERRED RESPONSE: 5
39
40  American Academy of Orthopaedic Surgeons

87. A 20-year-old football player has repeated episodes of heat cramps during summer
training sessions. A deficiency of what electrolyte is most responsible for heat cramps?

1- Potassium
2- Magnesium
3- Chloride
4- Sodium
5- Iron

DISCUSSION: Sodium deficiency is the cause of heat cramps. It is the principle electrolyte of
sweat and is readily lost during training, especially in warmer temperatures. The condition can be
avoided by adding extra table salt to food and maintaining good hydration before and after sports
activities. Salt tablets are to be avoided when a patient has heat cramps because the high soluble
load will cause gastric irritation. PREFERRED RESPONSE: 4

88. Figure 37 shows the radiograph of a 23-year-old football player who sustained a blow
to the anterior aspect of his shoulder. Examination reveals pain and limited rotation. He is
unable to flex the arm above the shoulder. Management should include which of the following
studies?

1- Axillary radiograph
2- Arthrogram
3- Electromyogram
4- Bone scan
5- Arteriogram

DISCUSSION: The patient has a posterior dislocation. The radiograph reveals marked internal
rotation, but fails to show whether the humeral head is posteriorly displaced. Therefore, an axillary
radiograph should be obtained to help confirm the diagnosis. Transverse view CT or MRI scans
also may be useful. The other studies will not help confirm the diagnosis. In addition to a direct
posterior blow, a shoulder dislocation may be caused by a seizure disorder or
electrocution.PREFERRED RESPONSE: 1

89. A 21-year-old football player had severe pain and immediate swelling in the left
anteromedial chest wall while bench pressing near maximal weights several days ago.
Examination at the time of injury revealed a mass on the anteromedial chest wall. Follow-up
examination now reveals decreased swelling, and axillary webbing is observed. The patient
has weakness to adduction and forward flexion. The injured muscle originates from the

1- proximal clavicle and sternocostal margin.


2- proximal humerus.
3- coracoid process.
4- distal clavicle and acromion.
5- anterior scapula.

DISCUSSION: The patient has a pectoralis major rupture, an injury that occurs most commonly
during weight lifting. Grade III injuries represent complete tears of either the musculotendinous
40 junction or an avulsion of the tendon from the humerus, the most common injury site. Examination

40
American Academy of Orthopaedic Surgeons
2001 Sports Medicine Self-Assessment Examination by Dr.Dhahirortho

will most likely reveal ecchymoses and swelling in the proximal arm and axilla, and strength testing
will show weakness with internal rotation and in adduction and forward flexion. Axillary webbing,
caused by a more defined inferior margin of the anterior deltoid as the result of rupture of the
pectoralis, can be seen as the swelling diminishes. Surgical repair is the treatment of choice for
complete ruptures. Nonsurgical treatment is associated with significant losses in adduction, flexion,
internal rotation, strength, and peak torque. The pectoralis major originates from the proximal
clavicle and the border of the sternum, including ribs two through six. The pectoralis major inserts
(rather than originates) on the humerus. The coracoid process is the insertion site for the pectoralis
minor, as well as the origin for the conjoined tendon. The pectoralis major has no attachment or
origin from the scapula. The anterior deltoid originates from the lateral one third of the clavicle and
the anterior acromion. PREFERRED RESPONSE: 1

90. A 24-year-old baseball pitcher reports pain over the posterior aspect of his shoulder
that occurs only during throwing. He notes that the discomfort is greatest during the late
cocking and early acceleration phases. Examination reveals localized tenderness with
palpation over the external rotators and posterior glenoid. Radiographs are shown in Figures
38a through 38c. What is the most likely diagnosis?

1- Fracture of the posterior glenoid


2- Triceps insertion avulsion
3- Calcific tendinitis
4- Posterior glenoid exostosis
5- Loose body

DISCUSSION: The radiographs show a posterior glenoid osteophyte, often termed a “thrower’s
exostosis.” These exostoses are best visualized on the Stryker notch view and may be missed on
other more standard radiographic views of the shoulder. CT and MRI scans may be used, but
usually add little information to the radiographic findings. Arthroscopic examination of patients
with this condition commonly reveals undersurface tearing of the rotator cuff and posterior labrum.
Treatment of this condition remains somewhat controversial, with avocation of both nonsurgical
and surgical techniques. PREFERRED RESPONSE: 4

91. What percent of the adult human meniscus is vascularized?

1- 0%
2- 5%
3- 25%
4- 50%
5- 100%

41
41
42  American Academy of Orthopaedic Surgeons

DISCUSSION: The adult menisci are considered to be relatively avascular structures, with the
peripheral blood supply originating predominately from the lateral and medial genicular arteries.
Branches of these vessels form the perimeniscal capillary plexus, which supplies the peripheral
border throughout its attachment to the joint capsule. Vascular penetration studies have shown that
10% to 30% of the peripheral portion of the medial meniscus and 10% to 25% of the lateral
meniscus are vascularized.

PREFERRED RESPONSE: 3

92. A 30-year-old man who participates in recreational sports reports the spontaneous
onset of intermittent pain and swelling about the right knee. Examination reveals a 3+
effusion, with a range of motion of 10° to 60°. He has mild diffuse tenderness but no
instability. MRI scans and an arthroscopic view are shown in Figures 39a through 39c.
Management should consist of

1- arthroscopic debridement of the articular lesion and resurfacing.


2- knee aspiration and an intra-articular cortisone injection.
3- rheumatologic evaluation.
4- infectious disease evaluation for possible Lyme disease.
5- arthroscopic synovectomy.

DISCUSSION: The patient has synovial chondromatosis. The MRI scans show multiple small
proscribed areas of signal intensity in the gutters and suprapatellar pouch, suggesting very small
loose bodies. The arthroscopic view shows the classic appearance of multiple small chondral loose
bodies. Synovial chondromatosis is a condition in which the synovium undergoes metaplasia,
producing multiple chondral loose bodies that can subsequently ossify. The treatment of choice,
removal of the loose bodies and arthroscopic synovectomy, results in a lower incidence of
recurrence than other treatment methods.

PREFERRED RESPONSE: 5

42
42
American Academy of Orthopaedic Surgeons
2001 Sports Medicine Self-Assessment Examination by Dr.Dhahirortho

93. Figure 40 shows the plain radiograph of a 30-year-old woman who has had a long
history of standing bilateral anterior knee pain and a sense of patellar instability without
frank dislocation. Nonsurgical management consisting of anti-inflammatory drugs and
physical therapy has failed to provide relief. Examination reveals full range of motion of both
knees, with moderate patellofemoral crepitance. Patellar apprehension and patellar grind
tests are positive. The Q-angle measures 20°. Management should now consist of

1-bilateral arthroscopic lateral releases.


2-bilateral arthroscopic lateral releases and medial
retinacular thermal shrinkage.
3-bilateral lateral releases and anteromedialization of the
tibial tubercles.
4-physical therapy and the use of patella-stabilizing braces.
5-physical therapy with taping.

DISCUSSION: The history, physical examination, and radiographs indicate that the patellofemoral
pain is most likely caused by excessive lateral patellar pressure and patellar maltracking. Because
the radiographs reveal the lateral tilt of the patella and lateral subluxation, the treatment of choice is
bilateral lateral releases with anteromedialization of the tibial tubercles. This procedure corrects not
only the excessive lateral patellar pressure, but also the lateral subluxation. The use of patella-
stabilizing braces or taping may provide temporary relief, but these implements are not well-
tolerated and they will not change the underlying biomechanics of the knee. Simple lateral release
is indicated for isolated lateral tilt, but it does not correct the lateral subluxation. The use of thermal
capsular shrinkage for the medial retinaculum has not been proven to provide long-term correction
of the deformity. PREFERRED RESPONSE: 3

94. A 24-year-old runner who underwent an allograft reconstruction of the anterior


cruciate ligament (ACL) 3 years ago now reports anterior knee pain. Examination reveals no
swelling or effusion, and the patient has full motion. A Lachman test and a pivot-shift test are
negative. Palpation reveals tenderness on the patellar tendon and at the inferior pole of the
patella. AP and lateral radiographs are shown in Figures 41a and 41b. Management should
consist of

1- immediate biopsy of the proximal tibia.


2- aspiration and culture of the knee.
3- observation with activity modification.
4- a white blood cell scan.
43 5- revision of the ACL reconstruction.
43
44  American Academy of Orthopaedic Surgeons

DISCUSSION: The radiographs show tunnel enlargement, which is seen after ACL reconstruction,
particularly with allografts. Occasionally, there will be formation of an associated subcutaneous
pretibial cyst. It has been proposed that the tunnel enlargement and cyst are the result of incomplete
incorporation of allograft tissues within the bone tunnels. There may be residual graft necrosis,
allowing synovial fluid to be transmitted through the tunnel to collect in the pretibial area,
manifesting as a synovial cyst. In the absence of cyst formation, the presence of tunnel enlargement
does not appear to adversely affect the clinical outcome. Based on studies by Fahey and associates,
continued tunnel expansion does not occur. Victoroff and associates report good results with
curettage and bone grafting of the tibial tunnel if a pretibial cyst is present. Because this patient
does not have a pretibial cyst, observation with activity modification is the preferred treatment.

PREFERRED RESPONSE: 3

95. What is the most common mechanism of injury that produces turf toe?

1- Valgus stress at the first metatarsophalangeal (MTP) joint


2- Hyperflexion stress
3- Hyperextension stress
4- Varus stress
5- Axial load

DISCUSSION: The most common mechanism of injury for turf toe is a hyperextension injury to the
MTP joint. The foot is typically in a dorsiflexed position with the heel raised when an external
force drives the MTP joint into further dorsiflexion. The joint capsule usually tears at the
metatarsal neck because its attachment is weaker there than at the proximal phalanx. Some
compression injuries to the dorsal articular surface of the metatarsal head can result from extension
or hyperextension.

PREFERRED RESPONSE: 3

96. A 68-year-old man embarks on a 24-week strength training program. He trains at


80% of his single repetition maximum for both the upper and lower extremities. Which of the
following changes can be anticipated?

1- An absolute decrease in aerobic capacity


2- A decrease in capillary density in the trained muscles
3- A significant increase in strength
4- A significant improvement in VO2max
5- No change in the cross-sectional area of the trained muscles

DISCUSSION: Consistent, long-term exercise training in older athletes has proven very beneficial
in reversing both endurance and strength losses that traditionally have been seen with aging. This
patient’s program will lead to a significant increase in the strength, cross-sectional area, and
capillary density of the trained muscles. No major changes in aerobic capacity are anticipated.
Strength improvements of up to 5% per day, similar to those for younger athletes, have been
identified in this population in one study.

PREFERRED RESPONSE: 3
44
44
American Academy of Orthopaedic Surgeons
2001 Sports Medicine Self-Assessment Examination by Dr.Dhahirortho

97. A 26-year-old ballet dancer reports posterolateral ankle pain, especially with maximal
plantar flexion. Examination reveals maximal tenderness just posterior to the lateral
malleolus, and symptoms are heightened with forced passive plantar flexion. Radiographs
are shown in Figures 42a and 42b. What is the most likely cause of the patient’s symptoms?

1- Posterior impingement of the os trigonum


2- Subluxation of the peroneal tendon
3- Posterior tibial stress fracture
4- Osteochondritis dissecans of the lateral dome of the talus
5- Stenosis of the peroneal tendon sheath

DISCUSSION: The patient has a symptomatic os trigonum caused by impingement that occurs with
maximal plantar flexion of the ankle in the demi-pointe or full-pointe position. Patients frequently
report posterolateral pain localized behind the lateral malleolus that may be misinterpreted as a
disorder of the peroneal tendon. Pain with passive plantar flexion (the plantar flexion sign)
indicates posterior impingement, not a problem with the peroneal tendon. The symptoms are not
characteristic of a stress fracture, nor do the radiographs show a stress fracture or an osteochondritis
dissecans lesion. The os trigonum is modest in its dimensions. The incidence or magnitude of
symptoms does not correlate with the size of the fragment. Large fragments may be asymptomatic,
while small lesions may create significant symptoms. PREFERRED RESPONSE: 1

98. A 23-year-old soccer player sustains a grade III complete posterior cruciate ligament
(PCL) tear after colliding with another player. In reconstructing the PCL, it is optimal to
reconstruct the

1- anterolateral bundle and tension the graft at 10° of flexion.


2- anterolateral bundle and tension the graft at 90° of flexion.
3- posteromedial bundle and tension the graft at 10° of flexion.
4- posteromedial bundle and tension the graft at 45° of flexion.
5- posteromedial bundle and tension the graft at 90° of flexion.

DISCUSSION: The PCL is a nonisometric structure with nonuniform tension during knee motion,
with maximum tension at 90° of flexion. While the posteromedial PCL fibers have been found to
be the most isometric, the anterolateral fibers represent the bulk of the ligament. Studies have
suggested that anterior placement of the femoral tunnel is superior to placement in an isometric
position. The anterolateral bundle tightens as the knee flexes; therefore, it is optimal to tension the
45 graft at 90° of flexion. PREFERRED RESPONSE: 2
45
46  American Academy of Orthopaedic Surgeons

99. Figure 43 shows the lateral radiograph of a patient who underwent anterior cruciate
ligament reconstruction. Based on the tunnel placement shown in the radiograph, evaluation
of postoperative knee range of motion will most likely show

1- normal flexion and extension.


2- loss of extension.
3- loss of flexion.
4- loss of flexion and extension.
5- hyperextension.

DISCUSSION: The radiograph shows the correct tibial tunnel and anterior femoral tunnel;
therefore, range of motion will most likely show loss of flexion.

PREFERRED RESPONSE: 3

100. A 10-year-old soccer player has bilateral heel pain and reports that the pain is worse
during and immediately after sports. Examination reveals that the calcaneal tuberosities are
painful to palpation bilaterally. What is the most likely diagnosis?

1- Plantar fasciitis
2- Calcaneal apophysitis
3- Achilles tendinitis
4- Calcaneal bursitis
5- Stress fractures of the calcanei

DISCUSSION: Calcaneal apophysitis (Sever’s disease) is a common cause of heel pain in children
who are active in sports. The symptoms are most commonly bilateral and will often respond to a
gastrocnemius-soleus complex stretching program. In addition, rest, anti-inflammatory drugs, and
heel pads for the shoe may be prescribed. There is no effect on the long-term growth of the
calcaneus.

PREFERRED RESPONSE: 2

46
46
American Academy of Orthopaedic Surgeons

47
American Academy of Orthopaedic Surgeons
2001 Adult Reconstructive Surgery of the Hip and Knee by Dr.Dhahirortho

1. Figure 1 shows the radiograph of a 68-year-old man who underwent revision hip arthroplasty
with impaction grafting of the femur and cementing of a tapered component into the graft 2
years ago. The patient remains symptom-free. Which of the following best describes the
most likely histologic appearance of the proximal femur if a biopsy was performed?

1- Complete restoration of the cortex, with interdigitation of cement into the patient’s
native bone
2- Fibrous membrane encapsulating the stem, surrounded by a cement mantle and dead
allograft
3- Healing by mixed endochondral ossification, similar to fracture healing, surrounding
the cement mantle
4- Allograft resorption, with some cortical restoration because of osteoinduction
5- Viable trabecular bone resulting from incorporation and remodeling of allograft

DISCUSSION: The radiograph shows three zones: an outer


regenerated cortical layer, an interface zone consisting of viable
trabecular bone and occasional particles of bone cement, and an inner
layer of necrotic bone embedded in cement. No fibrous membrane is
noted, and there is no direct contact of cement with native bone.
Based on these findings, it is believed that the middle layer is the
result of incorporation of the allograft with further remodeling.
PREFERRED RESPONSE: 5

2. Compared with cobalt-chromium, the biomechanical properties of titanium on polyethylene


articulation in total hip replacement result in

1- an increased rate of volumetric wear.


2- increased stability.
3- decreased frictional force.
4- a decreased rate of acetabular loosening.
5- a decreased rate of femoral stem loosening.

DISCUSSION: The surface hardness of titanium is low compared with that of cobalt-chromium
alloys. Titanium articulations are easily scratched, resulting in a significantly increased rate of wear
and debris production. The wear and resulting lysis can also result in an increased rate of loosening.
PREFERRED RESPONSE: 1

3. What is the most common reason for reoperation in total knee arthroplasty?

1- Polyethylene insert failure


2- Malalignment of the knee
3- Ligamentous instability
4- Perioperative infection
5- Patellar-related complications
48 1
American Academy of Orthopaedic Surgeons
2001 Adult Reconstructive Surgery of the Hip and Knee

DISCUSSION: Patellar problems currently constitute the largest number of complications after total
knee arthroplasty, affecting between 8% and 35% of patients. These problems include patellar
instability, fracture, component loosening, surface erosion, and pain. Malalignment, as an isolated
reason for revision, is uncommon, yet it contributes to accelerated wear of the components. Joint
instability affects up to 6% of patients, and the infection rate in knee arthroplasty is around 1% to
2%. PREFERRED RESPONSE: 5

4.Which of the following factors is most commonly associated with mechanical failure of a cemented
total hip arthroplasty?

1- Increased stem offset


2- Varus position of the stem
3- Osteoporotic bone
4- Patient weight of greater than 154 lb
5- Gender

DISCUSSION: Varus position of the stem is most commonly associated with failure of the cemented
femoral component because of association with an inadequate cement mantle in the proximal medial
and distal lateral zones. An inadequate cement mantle and obesity have been associated with
increased loosening but not as frequently as a varus deformity. The influences of gender and
osteoporotic bone on the outcome of cemented femoral components have not been established.
PREFERRED RESPONSE: 2

5.Figure 2 shows the radiograph of a 72-year-old woman who reports pain after a fall. History
includes several years of increasing thigh pain and limb shortening. Management consisting
of an extensive work-up for infection reveals normal laboratory studies, a positive bone scan,
and a negative hip aspiration. What is the most likely etiology of this complication?

1- Loosening of the prosthesis


2- Modulus mismatch
3- Chronic infection
4- Osteoporosis
5- Metastatic tumor

DISCUSSION: The patient has a midstem periprosthetic


fracture, which commonly results in loosening of the
prosthesis. Patients who have a large amount of bone loss
may require an allograft with the surgical reconstruction.
Although the patient reported a fall, her history is also
consistent with preexisting loosening of the prosthesis.
Chronic infection has been shown in up to 16% of these
fractures; however, the patient’s work-up revealed no
infection.
PREFERRED RESPONSE: 1

49 © Copyright by American Academy of Orthopaedic Surgeons. All rights reserved. 2


American Academy of Orthopaedic Surgeons
2001 Adult Reconstructive Surgery of the Hip and Knee by Dr.Dhahirortho
6.Figures 3a through 3c show the radiographs and bone scan of a patient who reports increasing pain
associated with activity for the past several months. Laboratory studies show an erythrocyte
sedimentation rate of 14 mm/h and a C-reactive protein level of 0.4. Aspiration is negative for
infection. Management should consist of

1- antibiotics for 6 weeks.


2- use of an unlocked brace.
3- revision arthroplasty.
4- resection of the implants.
5- two-stage reimplantation.

DISCUSSION: The radiographs show polyethylene wear, but exchange of this will not necessarily
provide pain relief. The presence of pain suggests the possibility of occult loosening, and the
surgeon must be prepared for this option intraoperatively. There is little evidence of infection.
PREFERRED RESPONSE: 3

7.Which of the following studies has the highest sensitivity and specificity in diagnosis of
osteonecrosis of the femoral head?

1- Intraosseous pressures
2- AP and frog-lateral radiographs
3- Technetium Tc 99m bone scan
4- MRI scan
5- CT scan

DISCUSSION: An MRI scan is both highly sensitive and specific for the evaluation of osteonecrosis.
The measurement of increased intraosseous pressure can be technically difficult and the results have
been variable. Plain radiographs can be normal early in the progression of osteonecrosis of the
femoral head. The technetium Tc 99m bone scan is a very sensitive test. However, it is not specific;
increased uptake can be noted in patients with arthritis, neoplastic disease, fracture, or sepsis. In
addition, because of bilaterality, the frequency of false-negative scans is relatively high.
PREFERRED RESPONSE: 4

50 3
American Academy of Orthopaedic Surgeons
2001 Adult Reconstructive Surgery of the Hip and Knee
8. Figure 4 shows the AP radiograph of a 28-year-old woman who has had moderate pain in the
left hip for the past year. Nonsurgical management has failed to provide relief. She denies
any history of hip pain, pathology, or trauma. Management should consist of

1- observation.
2- a Pemberton osteotomy.
3- a periacetabular osteotomy.
4- a Chiari osteotomy.
5- total hip arthroplasty.

DISCUSSION: The radiograph shows


developmental dysplasia of the hip with the
hip reduced and congruent. The treatment of
choice is a periacetabular osteotomy because
it can improve hip biomechanics and prolong
the function of the hip joint. This procedure
should be performed prior to the
development of severe degenerative changes.
Observation will not alter the patient’s natural history or the biomechanics of the hip. A total hip
arthroplasty should be delayed until severe degenerative changes are present. A Chiari osteotomy is
a salvage osteotomy used for a noncongruent subluxated hip. A Pemberton osteotomy requires an
open triradiate cartilage; therefore, it is not an option in an adult. PREFERRED RESPONSE: 3

9. What complication is most likely to develop after right total hip arthroplasty in the patient
shown in Figure 5?

1- Infection
2- Dislocation
3- Heterotopic bone formation
4- Early mechanical loosening
5- Excessive bleeding

DISCUSSION: The patient has hypertrophic arthritis,


which is a strong risk factor for heterotopic bone
formation. The remaining complications are possible
but do not have the same significant risks.
PREFERRED RESPONSE: 3

10. In revision total hip arthroplasty, an acetabular reconstruction cage is best indicated for which
of the following patterns of bone loss?

1- Enlarged acetabular rim


2- Cavitary central defect
3- Superior migration of 2 cm
4- Deficient anterior wall
5- Pelvic discontinuity

51 © Copyright by American Academy of Orthopaedic Surgeons. All rights reserved. 4


American Academy of Orthopaedic Surgeons
2001 Adult Reconstructive Surgery of the Hip and Knee by Dr.Dhahirortho
DISCUSSION: Acetabular cage reconstruction is indicated in severe disruption of acetabular bone
stock when a cementless acetabular component cannot be stabilized in intimate contact with a
sufficient bed of structurally sound and viable host bone, with or without a structural graft. Cages are
used in pelvic discontinuity where they provide a bridge between the ilium and the ischium, while
supporting a cemented cup. All of the other scenarios are amenable to achieving an adequate rim fit
for a cementless component, using a jumbo cup if necessary.
PREFERRED RESPONSE: 5

11. Mechanical reduction of the pain associated with the condition shown in Figure 6 can be
accomplished through the use of a cane on the contralateral side. Similarly, if this patient must carry
any type of load in his or her arms, it should be carried

1- on the ipsilateral side.


2- on the contralateral side.
3- in a backpack.
4- directly in front with both arms.
5- with a broad, padded strap on both
shoulders.

DISCUSSION: Patients with diseased hips often


must carry objects while walking, yet they are
rarely instructed on which hand to use. The
patient should be directed to carry the object on
the ipsilateral side, just the opposite of the side
he or she would use a cane. The cane pushes up on the weight of the body so that when the patient is
carrying a load, the weight in the hand on the same side as the hip pushes up on the weight of the
body, but now the patient has the fulcrum of the hip in between. Tan and associates mathematically
determined the hip forces that result when a load is carried in the ipsilateral hand versus the
contralateral hand. Using a free-body diagram of a single-leg supported stance, they found that when
a load was carried in the contralateral hand, the resultant forces on the hip were increased
considerably. Conversely, when the weight was carried in the ipsilateral hand, the forces were
actually lower than when no weight was carried at all. Therefore, carrying a weight on the opposite
side resulted in hip forces that were substantially greater than when the weight was carried on the
same side. PREFERRED RESPONSE: 1

12. Figure 7 shows the AP radiograph of a 60-year-old man


who has had pain in the thigh for past 6 months. History reveals
that he underwent hip replacement 1 year ago. The radiographic
changes are most likely the result of what process?

1- Microtrauma
2- Mechanical loosening
3- Septic loosening
4- Neoplasia
5- Congenital anomaly

52 5
American Academy of Orthopaedic Surgeons
2001 Adult Reconstructive Surgery of the Hip and Knee
DISCUSSION: The arrows in the radiograph point to circumferential radiolucencies that strongly
support the diagnosis of septic loosening. Radiolucent lines that occur in such a short time are also
typical of an infection. PREFERRED RESPONSE: 3

13.Figure 8 shows the radiograph of a 72-year-old man who has had severe pain in the left hip for the
past 3 weeks. History reveals alcohol abuse. The next most appropriate step should consist of

1- hip aspiration.
2- Doppler ultrasound.
3- AP tomograms.
4- a CT scan.
5- a technetium Tc 99m bone
scan.

DISCUSSION: The radiograph reveals


destruction of the femoral head with loss of
the articular cartilage. These findings are
consistent with an infected hip, and aspiration
will confirm the diagnosis. Although the
patient could have advanced osteonecrosis, typically the cartilage interval is maintained and such
destruction is rarely associated with osteonecrosis.
PREFERRED RESPONSE: 1

14. Which of the following articulation couplings shows the lowest coefficient of friction as
tested in the laboratory?

1- Cobalt-on-polyethylene
2- Cobalt-on-cobalt
3- Titanium-on-polyethylene
4- Stainless steel-on-polyethylene
5- Ceramic-on-ceramic

DISCUSSION: Alumina ceramic is highly biocompatible when used as a biomaterial for joint
arthroplasty implants. It has been shown to have good hardness, low roughness, and excellent
wettability, therefore resulting in very low friction. However, it is expensive and limited reports have
shown the problem of fracture on impact. The exact role for ceramic articulations is unknown at
present. PREFERRED RESPONSE: 5

15.Which of the following is considered the best cementless acetabular reconstruction method when
planning for total hip arthroplasty in a patient with developmental dysplasia of the hip (DDH)?

1- Cemented reconstruction with the cup in an anatomic position and cement filling the
defect
2- Medialized component positioning with no femoral head graft, leaving up to 20% of
the shell uncovered
3- High and lateral positioning of the acetabular component with a femoral head graft
4- Anatomic positioning of a small shell with a 28-mm liner and 4 mm of polyethylene
5- Anatomic positioning of the cup and a femoral head graft covering 70% of the cup

53 © Copyright by American Academy of Orthopaedic Surgeons. All rights reserved. 6


American Academy of Orthopaedic Surgeons
2001 Adult Reconstructive Surgery of the Hip and Knee by Dr.Dhahirortho
PREFERRED RESPONSE: 2
DISCUSSION: Anatomic positioning of the acetabular component has been shown to be the optimal
position for reconstruction of the acetabulum in total hip arthroplasty for DDH. The use of
medialized component positioning has been shown to be successful at maximizing the host bone
coverage and minimizing the use of bone graft to structurally support the acetabular component. A
small acetabular component can be used successfully as long as the femoral head is also reduced in
size to maintain the thickness of the acetabular polyethylene. High and lateral positioning for the
acetabular reconstruction will result in an increase in the joint reaction forces. In addition, a high and
lateral placement will not provide adequate bone to stabilize the reconstruction.

16. Which of the following mechanisms is considered the most common cause of failure of
osteoarticular allografts used for articular reconstruction?

1- Osteocyte surface antigens that trigger an immune rejection


2- Chondrocyte surface antigens that trigger an immune rejection
3- Graft collapse during revascularization
4- Mechanical loosening at the bone-bone junction
5- Infection via graft contamination

DISCUSSION: Mechanical loosening and infection can occur as complications after surgery, but the
most common cause of osteoarticular allograft failure is graft collapse during revascularization.
Clinical rejection because of an immune response is an unusual means of failure.
PREFERRED RESPONSE: 3

17. When compared with a patient who has a subluxated hip, a patient with a dislocated hip who
is undergoing acetabular reconstruction for developmental dysplasia of the hip will most
likely have

1- an increased need for revision.


2- a greater limb-length discrepancy.
3- a decreased rate of postoperative instability.
4- a decreased rate of wear.
5- a decreased rate of peroneal nerve palsy.

DISCUSSION: The rate of revision has been found to be significantly increased in patients with a
dislocated hip preoperatively compared with patients with a subluxated hip. This may be the result of
compromised acetabular bone stock. The rate of nerve palsy may be increased because of the greater
degree of lengthening required to reduce the reconstructed hip. PREFERRED RESPONSE: 1

18. The anticoagulant effect of the low-molecular-weight heparins (LMWH) is mediated by the
binding affinity of antithrombin III to which of the following coagulation factors?

1- III
2- V
3- IX
4- Xa
5- XII

54 7
American Academy of Orthopaedic Surgeons
2001 Adult Reconstructive Surgery of the Hip and Knee
PREFERRED RESPONSE: 4
DISCUSSION: Standard heparin mediates its anticoagulant effect largely through its interaction with
antithrombin III. A conformational change in antithrombin III occurs that markedly accelerates its
ability to inactivate the coagulation enzymes thrombin factor (II), factor Xa, and factor IXa. In
contrast, LMWHs do not contain the necessary saccharide units to bind thrombin and antithrombin
III simultaneously. The anticoagulant effect of LMWHs involves binding of antithrombin III to
factor Xa.

19. The primary purpose of obtaining the radiograph shown in Figure 9 is to assess

1- the anterior column of the acetabulum.


2- the acetabular rim.
3- the os acetabulae.
4- anterior coverage of the femoral head.
5- femoral anteversion.

DISCUSSION: The radiograph shows a faux profil view of the hip.


The primary purpose of this view is to evaluate anterior coverage of the
femoral head.
PREFERRED RESPONSE: 4

20. Figure 10 shows the radiograph of an active 75-year-old woman who reports severe leg pain
after a fall. Management should consist of

1- a total hip arthroplasty with a long-stem prosthesis.


2- a resection arthroplasty with skeletal traction.
3- hemiarthroplasty with a long-stem prosthesis.
4- retention of the prosthesis and open reduction and internal
fixation.
5- closed treatment with skeletal traction.

DISCUSSION: The patient has a comminuted fracture of the proximal


femur and joint space narrowing of the acetabulum. Therefore, the
prosthesis should be converted to a total hip arthroplasty. Because there
is extensive comminution, the revision stem should bypass the area of
bone loss by two bone diameters. A hemiarthroplasty is not indicated
because the patient has no acetabular cartilage. Open reduction and
internal fixation may not stabilize the prosthesis. A resection arthroplasty
or treatment in traction will not leave the patient with adequate function.

PREFERRED RESPONSE: 1

55 © Copyright by American Academy of Orthopaedic Surgeons. All rights reserved. 8


American Academy of Orthopaedic Surgeons
2001 Adult Reconstructive Surgery of the Hip and Knee by Dr.Dhahirortho
21. Which of the following procedures is considered most appropriate in patients with rheumatoid
arthritis?

1- Hip arthrodesis
2- Osteotomy of the hip
3- Core decompression of the hip
4- Synovectomy of the knee
5- Unicondylar knee arthroplasty

DISCUSSION: Synovectomy of the knee prior to loss of articular cartilage has been shown to
consistently relieve pain in patients with rheumatoid arthritis. Partial knee replacement will not arrest
the process of joint destruction. Osteotomy of the hip has not been found to be a successful
procedure in patients with rheumatoid arthritis. Hip arthrodesis should not be considered because of
the multiarticular involvement in patients with rheumatoid arthritis. Core decompression of the hip
has not been shown to save the femoral head because the necrosis appears to occur simultaneously
with the inflammatory joint process. PREFERRED RESPONSE: 4

22.In a patient who has rheumatoid arthritis with acetabular protrusion, what is the best
biomechanical position for the cup with respect to the preoperative center of rotation?

1- Medial and superior


2- Medial
3- Lateral and superior
4- Anterior and inferior
5- Posterior and lateral

DISCUSSION: Acetabular protrusion in patients with rheumatoid arthritis moves the center of hip
rotation medially and posteriorly. Positioning of the acetabular component in a patient with
protrusion is best accomplished in the normal (anterior and inferior) position and not in a protruded
position. This has been shown both clinically and in a finite-element analysis. Any medial
positioning will produce impingement of the prosthesis neck on the rim, and superior placement
produces improper hip mechanics. PREFERRED RESPONSE: 4

23. Figures 11a and 11b show the radiographs of a 50-year-old man who was struck by a car.
Treatment should consist of

56 9
American Academy of Orthopaedic Surgeons
2001 Adult Reconstructive Surgery of the Hip and Knee
1- cemented bipolar hemiarthroplasty.
2- cementless bipolar hemiarthroplasty.
3- hybrid total hip arthroplasty.
4- cementless total hip arthroplasty.
5- open reduction and internal fixation.

DISCUSSION: The patient has a displaced femoral neck fracture. Although the treatment remains
controversial, most clinicians advocate either a closed or open reduction in younger active patients.
Achieving an anatomic reduction is necessary to avoid loss of reduction, nonunion, or osteonecrosis.
An acceptable reduction may have up to 15° of valgus angulation and 10° of posterior angulation.
Parallel multiple screws or pins are the most common method of internal fixation. Prosthetic
replacement is generally reserved for older and less active individuals. PREFERRED RESPONSE: 5

24. One advantage of using onlay strut allograft in femoral revision surgery is that it can

1- provide some structural support to host bone.


2- provide better osteoconductive properties than cancellous graft.
3- completely incorporate into the host femur to restore bone stock.
4- be used for cavitary defects of the femur.
5- be used as a primary structural support for the femoral component.

DISCUSSION: Onlay grafts can provide more structural support than morcellized grafts. They are
more easily incorporated into the host femur than bulk segmental total femur allografts; however, the
incorporation is never complete. The use of onlay grafts is principally directed at addressing
segmental defects of the femur; their use can be applied with either cementless or cement fixation of
the femoral stem. PREFERRED RESPONSE: 1

25. Which of the following methodologies has been proven to be effective in reducing the use of
homologous blood transfusion following total hip arthroplasty (THA)?

1- Type of postoperative anticoagulation


2- Preoperative autologous blood donation
3- General anesthesia
4- Cementless fixation of the components
5- The use of wound drains

DISCUSSION: A variety of methodologies have been used to decrease the need for homologous
blood transfusions following THA. Some of the effective strategies include preoperative donation of
autologous units, intraoperative salvage and recycling, preoperative injection of erythropoietin, and
regional anesthesia. Cementless fixation and use of wound drains have been shown to increase the
blood loss with THA. PREFERRED RESPONSE: 2

26. A 67-year-old man is requesting revision surgery because of continued pain in the knee after
undergoing a total knee replacement 2 years ago. Examination reveals that the knee is not warm, the
incision is well-healed, and the skin has normal coloration and hair formation. No varus or valgus
instability is noted, and knee range of motion is 5° to 100°. Laboratory studies show an erythrocyte
sedimentation rate of 15 mm/h and a WBC of 5,000/mm3. Aspiration of the knee reveals clear fluid
that shows no growth on culture. Radiographs reveal an appropriately positioned cruciate-retaining
57 © Copyright by American Academy of Orthopaedic Surgeons. All rights reserved. 10
American Academy of Orthopaedic Surgeons
2001 Adult Reconstructive Surgery of the Hip and Knee by Dr.Dhahirortho
cemented total knee arthroplasty that is well-fixed. What is the probability that the patient’s pain will
be improved with revision surgery?

1- 5%
2- 10%
3- 40%
4- 60%
5- 90%

DISCUSSION: The patient has a well-fixed and aligned painful total knee replacement. The success
rate of revision knee replacement for pain when no mechanical problem can be identified is
approximately 40%. The critical step is to rule out the presence of infection with appropriate
laboratory studies and aspiration. If no infection is detected, revision should be avoided.
PREFERRED RESPONSE: 3

27. For patients undergoing a surgical procedure where the risk of requiring a transfusion is less
than 10%, the International Committee of Effective Blood Usage suggests

1- 1 unit of autologous blood.


2- 2 units of autologous blood.
3- 1 unit of direct donated blood.
4- use of cell saver intraoperatively.
5- no donation is necessary.

DISCUSSION: Recent studies have shown a high rate of waste of autologous blood. Therefore, the
Committee does not recommend autologous blood donation for procedures that carry a transfusion
risk of 10% or less. PREFERRED RESPONSE: 5

28. Figure 12 shows the radiograph of a 55-year-old man who has severe, painful osteoarthritis of
the left hip and is scheduled to undergo a left total hip arthroplasty. History reveals that he
underwent a right total hip arthroplasty 5 years ago that remains pain-free. Based on the preoperative
radiograph, the patient is at greatest risk for what complication?

1- Intraoperative fracture
2- Deep vein thrombosis
3- Limb-length discrepancy
4- Sciatic nerve palsy
5- Thigh pain

DISCUSSION: The patient is at increased


risk for limb-length discrepancy because
the radiograph shows that the left leg is
already longer than the right leg. To restore the proper biomechanics of the left hip, the left leg may
have to be lengthened, further increasing the limb-length discrepancy. Intraoperative fracture, deep
vein thrombosis, sciatic nerve palsy, and thigh pain are commonly associated with total hip
arthroplasty, but the patient is not at increased risk for these complications.
PREFERRED RESPONSE: 3

58 11
American Academy of Orthopaedic Surgeons
2001 Adult Reconstructive Surgery of the Hip and Knee
29.Figures 13a and 13b show the preoperative radiographs of a 60-year-old woman who is scheduled
to undergo total knee arthroplasty under epidural anesthesia. Postoperatively she reports a burning
sensation on the dorsum of her foot despite the administration of IV analgesics through a patient-
controlled analgesia (PCA) pump. Management should now include

1- increasing the dose released by the PCA.


2- administering a different narcotic with the PCA pump.
3- elevating the leg.
4- releasing the dressings and knee flexion.
5- immediately returning to the operating room for revision.

DISCUSSION: The patient has a significant flexion contracture and valgus deformity; therefore, the
risk of peroneal nerve injury is increased. Idusuyi and Morrey noted that epidural anesthesia also
increases the risk of peroneal nerve injury. The initial symptom can be a burning sensation on the
foot, followed by pain and then motor weakness. Initial management should consist of release of the
dressings and knee flexion. PREFERRED RESPONSE: 4

30. Figures 14a and 14b show the plain radiographs of an 85-year-old woman who has had severe
pain in the right knee for the past 4 months. Management should consist of

1- a hinged knee brace.


2- arthroscopic debridement.
3- high tibial osteotomy.
4- total knee arthroplasty.
5- osteochondral grafts.

59 © Copyright by American Academy of Orthopaedic Surgeons. All rights reserved. 12


American Academy of Orthopaedic Surgeons
2001 Adult Reconstructive Surgery of the Hip and Knee by Dr.Dhahirortho
PREFERRED RESPONSE: 4
DISCUSSION: The patient has osteonecrosis of the lateral femoral condyle with collapse of the
articular surface. Because there is already collapse of the articular surface, a total knee arthroplasty
is the treatment of choice. The results of total knee arthroplasty in these patients are usually
excellent. However, knee replacement is only a resurfacing procedure, and some patients with global
osteonecrosis of the distal femur may have residual pain after knee replacement. High tibial
osteotomy may be indicated in younger patients who have a varus deformity and localized
osteonecrosis. Arthroscopic surgery would provide minimal relief for this patient because there is
already collapse of the articular surface. A hinged knee brace will not adequately unload the joint.
An osteochondral allograft should be considered only for younger patients with localized
osteonecrosis.

31. The failure of the acetabular component shown in Figure 15 is most likely the result of the use
of a 32-mm head and

1- the material properties of the polyethylene.


2- the initial alignment of the component.
3- overuse of the component by the patient.
4- failure to stabilize the cup with screws.
5- increased femoral head offset.

DISCUSSION: Astion and associates analyzed 23 acetabular


components, out of a total of 173 implanted, that had failed
because of either migration or severe osteolysis. The
radiographic appearance of osteolysis was positively associated
with the duration that the implant had been in situ. The
prevalence of osteolysis was also significantly greater in
acetabular components with an outer diameter of 55 mm or less
(a polyethylene thickness of 8.5 mm or less). Thirteen of the
23 components were revised at a mean of 70 months after the
index operation. Examination of the retrieved acetabular
components revealed extensive polyethylene damage on the articular and back surfaces of the liners.
Cracks in the polyethylene rim of the liner and deformation of the antirotation notch in the
polyethylene rim were common findings. The density of the polyethylene was greater than expected,
and more particles than anticipated had not fused with the surrounding polyethylene. Factors related
to both the design and the material contributed to the failure of these porous-coated anatomic
acetabular components. PREFERRED RESPONSE: 1

32. The use of elevated rim acetabular liners and long femoral necks may result in

1- increased abductor tension.


2- an increased likelihood of impingement.
3- an increased likelihood of osteolysis.
4- restricted hip range of motion.
5- dissociation of polyethylene from the acetabular cup.

DISCUSSION: Elevated rim acetabular liners may improve the anteversion of the acetabular
component that, in turn, might improve the stability of the hip replacement through a range of
60 13
American Academy of Orthopaedic Surgeons
2001 Adult Reconstructive Surgery of the Hip and Knee
motion. Long femoral necks with skirts will increase the abductor tension and may be necessary to
equalize limb lengths. However, either of these measures may increase the likelihood of
impingement of the femoral component on the acetabular rim and may lead to dislocation. The
restricted range of motion secondary to impingement has been shown to lead to further polyethylene
wear that may result in osteolysis. PREFERRED RESPONSE: 2

33. Cementation technique has a definite influence on the long-term survival of cemented femoral
components. Both clinical and autopsy studies support the use of a cement mantle with a thickness
of how many millimeters?

1- 0.5
2- 1
3- 2
4- 3
5- 4

DISCUSSION: Long-term radiographic analysis of cemented total hips supports the creation of a 2-
to 5-mm cement mantle in the proximal medial region. Autopsy studies have shown that the
incidence of crack formation was greatest when the cement mantle was less than 2 mm.
PREFERRED RESPONSE: 3

34. A 73-year-old man is scheduled to have mature heterotopic bone resected from around his left
total hip arthroplasty. The optimal management for prophylaxis against the return of heterotopic
bone postoperatively is radiation therapy that consists of

1- 400 cGy in one dose.


2- 700 cGy in one dose.
3- 1,000 cGy in five doses.
4- 2,000 cGy in 10 doses.
5- 3,000 cGy in 10 doses.

DISCUSSION: Patients require prophylaxis for heterotopic bone after resection to prevent
recurrence. The optimal management has been found to be a dose of 700 cGy in one dose delivered
either pre- or postoperatively. A dose of 2,000 to 3,000 cGy is considered excessive. Radiation
therapy consisting of 1,000 cGy in five doses is an acceptable prophylaxis; however, it will require
an extended hospital stay of 3 to 4 days and is more problematic for the patient who must be
transported for radiation therapy for 5 days. A dose of 400 cGy is not as effective in prophylaxis for
heterotopic bone formation. PREFERRED RESPONSE: 2

35. Which of the following is considered a potential advantage in prophylaxis for the prevention
of deep venous thrombosis associated with the use of low-molecular weight heparin (LMWH)
as compared with fixed-dose unfractionated heparin?

1- Reduction in free fibrinogen radicals


2- Reduction in bleeding complications
3- Increased venous flow
4- Improved bioavailability
5- Inhibition of factors V, VI, and IX

61 © Copyright by American Academy of Orthopaedic Surgeons. All rights reserved. 14


American Academy of Orthopaedic Surgeons
2001 Adult Reconstructive Surgery of the Hip and Knee by Dr.Dhahirortho
PREFERRED RESPONSE: 4
DISCUSSION: One possible reason for improved efficacy of LMWHs is the relative improved
bioavailability compared with that of unfractionated heparin. This is, in part, the result of a more
predictable dose response and a longer half-life. There is no alteration of venous flow, and the rate of
bleeding complications is the same or slightly higher than that of other prophylactic agents.

36. Figure 16 shows the radiograph of a 75-year-old man who has progressive groin pain and a
limp following total hip replacement. At revision surgery, the anterior and posterior columns of the
acetabulum are noted to be intact. The optimal surgical technique for acetabular component
reconstruction is a

1- threaded (screw-in) cup with a hydroxyapatite coating.


2- protrusio cage reconstruction with a cemented cup.
3- large cementless cup with bone grafting of defects.
4- small cup with a high and lateral hip center.
5- bulk allograft reconstruction of the defect with a
cemented cup.

DISCUSSION: Large cementless acetabular components have


been shown to perform well in revision acetabular
reconstruction. The use of such components is predicated on the
presence of adequate anterior and posterior column bone. If a
good press-fit can be achieved between the anterior and
posterior columns, typically, the remaining defects can be filled
with morcellized bone graft. Protrusio cages are typically used
in situations where it is not possible to obtain adequate fixation
with a large acetabular component. The use of a high hip center
with small sockets is more typical of primary arthroplasty in
patients with developmental dysplasia of the hip. Bulk
acetabular allografts for large segmental defects might be necessary in certain situations, although the
use of bulk allografts has resulted in a high failure rate after 5 years. Early results of the use of
protrusio cages and bone grafting for large segmental defects have been favorable. PREFERRED
RESPONSE: 3

37. Which of the following is a recognized consequence of hip fusion?

1- Low back pain


2- Contralateral knee laxity
3- Difficulty delivering children
4- Meralgia paresthetica
5- Contralateral abductor weakness

DISCUSSION: Low back pain is an expected long-term complication of fusion; ipsilateral knee
laxity is frequently encountered, as is degeneration of the contralateral hip. Hip fusion is equally
valuable for both men and women, with both genders reporting satisfactory sexual function. Female
patients often deliver by elective Cesarean section, although vaginal deliveries are reported.
PREFERRED RESPONSE: 1

62 15
American Academy of Orthopaedic Surgeons
2001 Adult Reconstructive Surgery of the Hip and Knee
38. Treatment of a cruciate-retaining total knee that is unstable in flexion is best accomplished by

1- revising the implant to a posterior stabilized device.


2- revising the implant with a thinner polyethylene insert.
3- revising the implant with a larger femoral component.
4- limiting flexion to only 90°.
5- using quadriceps conditioning exercises and a derotation brace.

DISCUSSION: Pagnano and associates revised 25 painful primary posterior cruciate-retaining total
knee arthroplasties for flexion instability. The patients shared typical clinical presentations that
included a sense of instability without frank giving way, recurrent knee joint effusion, soft-tissue
tenderness involving the pes anserine tendons and the retinacular tissue, posterior instability of 2+ or
3+ with a posterior drawer or a posterior sag sign at 90° of flexion, and above-average motion of the
total knee arthroplasty. Twenty-two of the knee replacements were revised to posterior stabilized
implants, and three underwent tibial polyethylene liner exchange only. Nineteen of the 22 knee
replacements revised to a posterior stabilized implant showed marked improvement after the revision
surgery. Only one of the three knee replacements that underwent tibial polyethylene exchange was
improved. Flexion instability can be a cause of persistent pain and functional impairment after
posterior cruciate-retaining total knee arthroplasty. Revision surgery that focuses on balancing the
flexion and extension spaces, in conjunction with a posterior stabilized knee implant, seems to be a
reliable treatment for symptomatic flexion instability after posterior cruciate-retaining total knee
arthroplasty. PREFERRED RESPONSE: 1

39. The stiffness of a 16-mm femoral stem is mostly influenced by the

1- elastic modulus of the material.


2- surface coating or treatment.
3- diameter of the femoral stem.
4- length of the femoral stem.
5- ultimate tensile strength.

DISCUSSION: The stiffness is most influenced by the geometry, in particular the diameter of the
stem. The bending rigidity increases to the fourth power of the radius. The elastic modulus of the
material increases as a direct linear relationship. The surface coating does not affect the bending
rigidity greatly unless it increases the diameter significantly. PREFERRED RESPONSE: 3

40. Figures 17a and 17b show the AP and lateral radiographs of a 75-year-old woman who
reports giving way and shifting of the knee, particularly when she is descending stairs or
ambulating on level surfaces. History reveals a total knee replacement 5 years ago.
Treatment should consist of

63 © Copyright by American Academy of Orthopaedic Surgeons. All rights reserved. 16


American Academy of Orthopaedic Surgeons
2001 Adult Reconstructive Surgery of the Hip and Knee by Dr.Dhahirortho
1- extra-articular ligament repair.
2- resection arthroplasty with a cement spacer.
3- revision of the tibial tray.
4- revision of the patella to an all-polyethylene component.
5- revision to a posterior cruciate-substituting implant.

DISCUSSION: The radiographs show well-fixed components of a posterior cruciate-retaining total


knee replacement. The relative position of the femoral component is anteriorly subluxated relative to
the tibial component. The AP radiograph shows that the articular space is markedly asymmetric,
indicating either failure or fracture of the polyethylene or subluxation of the femur relative to the
tibia. The patient’s symptoms suggest a failure of the posterior cruciate ligament that is consistent
with the radiographic findings; therefore, the treatment of choice is revision to a posterior cruciate-
substituting implant. PREFERRED RESPONSE: 5

41. Factors contributing to an increased risk of hip fracture include reduced bone mineral density
of the femoral neck, cognitive status of the individual, and

1- increased trunk muscle activity.


2- increased muscle activity about the hip.
3- increased muscle activity about the shoulder.
4- a flexed hip configuration during impact.
5- falling forward on an outstretched hand.

DISCUSSION: The etiology of hip fractures in the elderly is multifactorial, and intervention and
prevention can occur at multiple points. Events leading to hip fracture from a fall include fall
initiation (during which the individual’s neuromuscular status, cognitive status, and vision come into
play along with environmental hazards); fall descent (fall direction toward the side being the most
influential, energy content of the fall, and fall height, along with muscle activity of the muscles of the
thigh); impact (impact location, soft-tissue attenuation such as from trochanteric padding or from
overlying fat, impact surface, and muscle activity); and the structural capacity of the femur (bone
mineral density, bone geometry, and bone architecture).
Hayes and Myers noted that striking the ground in a stiff state with the trunk muscles contracted
actually increased the peak impact force, whereas falling in a relaxed state actually reduced peak
impact force. Flexion of the trunk at impact had no bearing on the impact force. Direction of the fall
was important; falls to the side, not forward, were associated with an increased risk of hip fracture.
Increased muscle activity about the hip is thought to be associated with spontaneous fractures of the
hip and may actually account for up to 25% of hip fractures; however, it is not related to fractures
resulting from a fall. PREFERRED RESPONSE: 1

42. A healthy 70-year-old man has a swollen knee after undergoing a knee replacement 10 years
ago. Aspiration of the knee reveals cloudy, viscous synovial fluid. Laboratory studies show an
erythrocyte sedimentation rate of 10 mm/h and a C-reactive protein level of less than 0.5. What is
the most likely diagnosis?

1- Infected total knee arthroplasty


2- Polyethylene wear-related synovitis
3- Rheumatoid arthritis synovitis
4- Gout
5- Tibial component loosening
64 17
American Academy of Orthopaedic Surgeons
2001 Adult Reconstructive Surgery of the Hip and Knee

PREFERRED RESPONSE: 2
DISCUSSION: Polyethylene wear debris can result in significant synovitis and subsequent cloudy
appearing synovial fluid. Typically, laboratory studies show a WBC of less than 30,000/mm3 and no
left shift. Cytologic examination can reveal intra-articular polyethylene particles. Infected total knee
arthroplasty is extremely uncommon in a healthy, immune-competent patient who has a normal
preoperative erythrocyte sedimentation rate and C-reactive protein level.

43. The insurance carrier of a patient who underwent total knee arthroplasty 4 days ago is now
demanding that the patient be discharged from the hospital. However, examination reveals that the
patient has a range of motion of only 10° to 55°, and the patient is concerned whether she will ever
move her knee normally. The insurance company representative should be advised that

1- discharge at this time may result in loss of motion and the necessity of manipulation
under anesthesia.
2- the insurance company has no right to make such demands on the surgeon or the
patient.
3- if the patient is discharged and fails to regain full motion, she will most likely file a
suit against the insurance company.
4- the patient will require a follow-up examination in 6 weeks to evaluate her progress.
5- the patient will be given an extra set of exercises to perform at home.

DISCUSSION: Examination findings that show flexion of only 55° at discharge should alert the
surgeon that the patient will require close scrutiny and follow-up. Mauerhan and associates
examined the records of 745 patients who had a primary total knee arthroplasty from 1993 to 1996.
At their institution, development and implementation of clinical pathways resulted in a significant
decrease in the average length of stay, beginning in 1993 with 6.4 days +/- 1.8 days and progressively
decreasing to 4.4 days +/- 1.0 days in 1996. The rate of manipulation (patients manipulated at 6
weeks/total number of patients receiving total knee arthroplasty) was 6.0% in 1993, 11.3% in 1994,
13.5% in 1995, and 12.0% in 1996. In the period of 1993 to 1996, patients requiring manipulation
consistently had a lower range of motion of 69.0° +/- 10° at the time of discharge compared with
patients not requiring manipulation who had a range of motion of 80.7° +/- 10.6°. In this era of
outpatient services, however, another solution would be to arrange for outpatient physical therapy on
a more frequent basis and to see the patient more frequently in the office until an acceptable range of
motion is established. PREFERRED RESPONSE: 1

44. Figure 18 shows the radiograph of a patient with a total hip arthroplasty dislocation. During
revision, increasing the diameter of the femoral head while maintaining the ratio of head-to-neck
diameter constant has the effect of

1- increasing the force to dislocation of the femoral head.


2- increasing the range of motion until impingement of the neck on the
liner.
3- decreasing the resisting moment to dislocation of the femoral head.
4- decreasing the range of motion until dislocation of the femoral head.
5- decreasing the range of motion until impingement of the neck on the
liner.

65 © Copyright by American Academy of Orthopaedic Surgeons. All rights reserved. 18


American Academy of Orthopaedic Surgeons
2001 Adult Reconstructive Surgery of the Hip and Knee by Dr.Dhahirortho
DISCUSSION: Although there is strong clinical and laboratory evidence that suggests smaller head
size is linked with lower rates of polyethylene wear, moving to the use of 22-mm heads from larger
sizes would tend to increase the dislocation rate. The key premise to this argument is that the
absolute size of the femoral neck remains unchanged. While neck diameters were appropriate for the
early monoblock femoral components, the use of modular femoral stems allows the surgeon to place
22-mm heads onto the same neck and trunion as used by larger heads. This has the effect of
lessening the head-to-neck diameter ratio, which then accentuates the rate of impingement and
dislocation. Reducing the neck diameter in proportion to the head diameter would eliminate the
range-of-motion penalty accompanying head size reduction. PREFERRED RESPONSE: 2

Scifert and associates used a three-dimensional finite element model to study various combinations of
femoral head size and neck ratios. They found that increasing the diameter of the femoral head while
maintaining a constant head-to-neck diameter had the effect of significantly increasing the resisting
moment necessary to induce a dislocation. The higher the head-to-neck ratio, the greater the range of
motion until impingement and the greater the range of motion to dislocation.

45. During primary total knee arthroplasty with trial implants in place, the surgeon notes
technically satisfactory patellar resurfacing and restoration of a physiologic mechanical axis but
excessively lateral patellar tracking. Treatment should now include

1- a lateral retinacular release.


2- a tubercle transfer to reduce the Q angle.
3- a repeat of the tibial and femoral cuts to introduce 5° of varus.
4- release of the popliteus.
5- medial vastus advancement.

DISCUSSION: The most common causes of patellar instability after total knee arthroplasty are
valgus malalignment, internal rotation of the femoral or tibial component, medialization of the
femoral component, errors in patellar preparation and resurfacing, and failure to perform a lateral
release. These factors should be addressed before considering capsular closure. Distal extensor
mechanism realignment should be avoided because of the complication rate. The proximal extensor
mechanism would not adequately compensate for implant malrotation. PREFERRED RESPONSE: 1

46. Figure 19 shows the current radiograph of a 48-year-old man who reports hip pain and
marked difficulty walking after undergoing revision of a failed total hip replacement 2 years ago.
What is the mechanism of failure?

1- Fatigue
2- Crevice corrosion
3- Galvanic corrosion
4- Loosening
5- Wear

DISCUSSION: Fatigue from repetitive loading of the stem with the


distal aspect well-fixed resulted in stem failure. If the stem had
loosened, it would not have broken. Crevice corrosion occurs at a taper
interface; galvanic corrosion occurs at the junction of two metals of
differing electrochemical potentials, not along a uniform portion of the
implant. PREFERRED RESPONSE:
66 19
American Academy of Orthopaedic Surgeons
2001 Adult Reconstructive Surgery of the Hip and Knee
47.Torsional moments about the longitudinal axis of a total hip arthroplasty show what change during
stair climbing compared with walking?

1- Increase by a factor of 50% during stair climbing


2- Increase by a factor of 100% during stair climbing
3- Increase only during the first 6 to 8 weeks following implantation, then revert to
normal
4- Decrease by a factor of 50% during stair descent
5- Decrease by a factor of 100% during stair descent

DISCUSSION: The magnitudes of out-of-plane loads on a total hip replacement during activities of
daily living can be substantial. Bergmann and associates studied these forces about two instrumented
hip prostheses. They noted that the torsional moment about the hip during stair climbing is twice as
high as during slow walking and that similar moments are generated during slow jogging. Higher
loads were noted when the patients stumbled without falling. They also noted that the torsional
moments observed in vivo were close to or even exceeded the experimentally determined limits of
the torsional strength of implant fixations. PREFERRED RESPONSE: 2

48. When converting the knee shown in Figure 20 to a total knee arthroplasty, satisfactory
outcome can be expected in what percent of patients?

1- Less than 5%
2- Less than 50%
3- 60%
4- 80%
5- 90%

DISCUSSION: Naranja and associates reviewed 37 knees (35 patients, with 28 women and 7 men)
without any motion that were converted to total knee arthroplasties. After an average follow-up of 90
months, the patients lacked an average of 7° of extension and had 62° of flexion. Results showed a
short-term complication rate of 24% (stiffness requiring manipulation, delayed wound healing, and
recurrent hemarthrosis), a major complication rate of 35% (patellar tendon or tibial tubercle avulsion,
persistent pain requiring arthrodesis, loosening, and joint stiffness requiring arthrotomy for excision
of scar tissue), and an infection rate of 14%. The total complication rate was 57%. A satisfactory
outcome (no pain and an unlimited ambulation distance) was obtained in only 10 patients (29%).
There was no relationship between results and the angle at which the knee was ankylosed
preoperatively. This study revealed that although success in reconstructing a previously ankylosed or
arthrodesed knee is possible, the lack of consistent adequate motion and the complication rate may
suggest that the surgeon reconsider the risks and benefits of this difficult procedure.
PREFERRED RESPONSE: 2

49. The specificity of intraoperative frozen sections obtained for the evaluation of infected total
hip arthroplasty may be improved by

1- setting the threshold for diagnosis to 10 polymorphonuclear leukocytes per high-


powered field.
2- setting the threshold for diagnosis to 15 polymorphonuclear leukocytes per high-
powered field.
3- ensuring that each sample is obtained and submitted in a truly random fashion.
67 © Copyright by American Academy of Orthopaedic Surgeons. All rights reserved. 20
American Academy of Orthopaedic Surgeons
2001 Adult Reconstructive Surgery of the Hip and Knee by Dr.Dhahirortho
4- ensuring that polymorphonuclear leukocyte counts are obtained in a truly random
fashion.
5- correlating the frozen section results with those of the intraoperative Gram stain.

DISCUSSION: Lonner and associates conducted a prospective study to determine the reliability of
analysis of intraoperative frozen sections for the identification of infection during 175 consecutive
revision total joint arthroplasties (142 hips and 33 knees). The mean interval between the primary
arthroplasty and the revision arthroplasty was 7.3 years (range, 3 months to 23 years). Of the 175
patients, 23 had at least 5 polymorphonuclear leukocytes per high-powered field on analysis of the
frozen sections and were considered to have an infection. Of these 23 patients, five had 5 to 9
polymorphonuclear leukocytes per high-powered field and 18 had at least 10 polymorphonuclear
leukocytes per high-powered field. The frozen sections for the remaining 152 patients were
considered negative.
On the basis of cultures of specimens obtained at the time of the revision surgery, 19 of the 175
patients were considered to have an infection. Of the 152 patients who had negative frozen sections,
three were considered to have an infection on the basis of the results of the final cultures. Of the 23
patients who had positive frozen sections, 16 were considered to have an infection on the basis of the
results of the final cultures; all 16 had frozen sections that showed at least 10 polymorphonuclear
leukocytes per high-powered field.
The sensitivity and specificity of the frozen sections were similar regardless of whether an index of 5
or 10 polymorphonuclear leukocytes per high-powered field was used. Analysis of the frozen
sections had a sensitivity of 84% for both indices, whereas the specificity was 96% when the index
was 5 polymorphonuclear leukocytes and 99% when it was 10 polymorphonuclear leukocytes.
However, the positive predictive value of the frozen sections increased significantly (P < 0.05), from
70% to 89%, when the index increased from 5 to 10 polymorphonuclear leukocytes per high-powered
field. The negative predictive value of the frozen sections was 98% for both indices. At least 10
polymorphonuclear leukocytes per high-powered field was predictive of infection, while 5 to 9
polymorphonuclear leukocytes per high-powered field was not necessarily consistent with infection.
Less than 5 polymorphonuclear leukocytes per high-powered field reliably indicated the absence of
infection. PREFERRED RESPONSE: 1

50. Figures 21a through 21c show the radiographs of a 70-year-old woman who has persistent
pain with activity after undergoing hip revision 6 months ago. Treatment should now consist of

1- shortening of the femoral neck.


2- exchange of the acetabular liner.
3- revision of the femoral component.
4- revision of both components.
5- revision of the acetabular component.
68 21
American Academy of Orthopaedic Surgeons
2001 Adult Reconstructive Surgery of the Hip and Knee
DISCUSSION: The radiographs show disruption of the posterior column of the acetabulum with
radiolucencies about the component. Because the patient requires a stable construct to allow the
bone to heal, the treatment of choice is an antiprotrusio cage and a graft.
PREFERRED RESPONSE: 5

51. A patient who underwent a high tibial osteotomy (HTO) is now scheduled to undergo total
knee arthroplasty (TKA). When compared with a patient undergoing primary TKA without a prior
HTO, the patient should be advised to expect a higher incidence of

1- limited range of motion.


2- patella complications.
3- infection.
4- loosening.
5- tibia fracture.

DISCUSSION: Conversion TKA following a previous HTO can be successful; however, it is


associated with poorer clinical results when compared with other primary TKAs. There is an
increased likelihood of poor range of motion that is partially affected by patella infera created from
the osteotomy. Patella infera also results in difficulty with surgical exposure. There has been no
reported increase in the rate of infection, fracture, or loosening. PREFERRED RESPONSE: 2

52. Figures 22a and 22b show the radiographs of a patient who reports stiffness of the hip and
associated pain. Management should consist of

1- use of a cane for ambulation.


2- diphosphonate therapy.
3- physical therapy and indomethacin.
4- surgical excision and radiation therapy.
5- revision arthroplasty.

DISCUSSION: The patient has grade IV heterotopic ossification with the limb in an abnormal
nonfunctional position. Treatment should consist of excision of the bone to restore hip motion and
prophylaxis to prevent recurrent formation. The best time to excise the bone is controversial, with no
conclusive evidence supporting early or late excision.
PREFERRED RESPONSE: 4

69 © Copyright by American Academy of Orthopaedic Surgeons. All rights reserved. 22


American Academy of Orthopaedic Surgeons
2001 Adult Reconstructive Surgery of the Hip and Knee by Dr.Dhahirortho
53. Figure 23 shows the radiograph of a 55-year-old man who underwent a total hip arthroplasty
5 years ago. Management should now consist of

1- an Ogden-type plate with screws and cerclage bands or


cables.
2- allograft bone plates fixed with cerclage cables and wires.
3- skeletal traction for 8 weeks.
4- revision of the femoral stem.
5- resection arthroplasty.

DISCUSSION: Because the radiograph shows that the femoral stem is loose
within the femoral canal and there is a fracture in the distal cement mantle,
the stem should be revised. The Ogden-type plate and the allograft bone
plates will reconstruct the femur but will not restore stability to the stem.
Similarly, traction may allow the femur to heal but will not restore stability
to the femoral stem within the femur. Resection arthroplasty is considered a
salvage option following failure of the other procedures.
PREFERRED RESPONSE: 4

54. Compared with wear rates of metal-on-standard polyethylene bearings (75 to 250 m/y), the
wear rate of metal-on-metal bearings for hip arthroplasty is approximately how many
micrometers per year?

1- Less than 0.5


2- 2 to 5
3- 5 to 20
4- 20 to 50
5- 50 to 150

DISCUSSION: Studies on older systems, as well as newer designs, have confirmed that metal-on-
metal bearing surfaces undergo linear wear of 2 to 5 m per year. Ceramic bearing surfaces
produced with recent technology perform even better, with a wear rate of 0.5 to 2.5 m per year.
Clinical wear rates of metal-on-crosslinked polyethylene have not yet been determined.
PREFERRED RESPONSE: 2
.
55. A follow-up examination of a patient 6 weeks after knee surgery reveals a range of motion
from 5° to 55° of flexion. Which of the following statements best summarizes the role of
manipulation under anesthesia for this patient?

1- Manipulation under anesthesia offers the best chance of improving and maintaining
the patient’s range of motion.
2- The gains from manipulation under anesthesia are only temporary and rarely last more
than 6 months.
3- Increasing the frequency and intensity of physical therapy over the next 2 months will
have the same effect as manipulation under anesthesia.
4- The risks of fracture are so great from manipulating a knee that the patient should be
advised to live with a limited range of motion.
5- The patient’s final result will be poor with or without manipulation.

70 23
American Academy of Orthopaedic Surgeons
2001 Adult Reconstructive Surgery of the Hip and Knee
PREFERRED RESPONSE: 1
DISCUSSION: Esler and associates evaluated the use of manipulation under anesthesia in 47 knees.
Manipulation was considered when intensive physical therapy failed to increase flexion to more than
80°. The mean time from arthroplasty to manipulation was 11.3 weeks, and the mean active flexion
before manipulation was 62°. One year later, the mean gain was 33°. Definite sustained gains in
flexion were achieved even when manipulation was performed 4 or more months after arthroplasty.
An additional 21 patients who met the criteria for manipulation declined the procedure, and despite
continued physical therapy, they showed no significant increase in knee flexion.

56. The most compelling clinical reason to convert a hip arthrodesis to a total hip arthroplasty is
that the latter

1- improves hip range of motion.


2- relieves pain associated with arthritis of the lumbar spine.
3- relieves pain associated with arthritis of the knee.
4- relieves pain in the contralateral hip.
5- corrects a limb-length discrepancy.

DISCUSSION: Studies show that degenerative arthritis of the spine associated with a hip arthrodesis
can be decreased with conversion to a total hip arthroplasty. The pain associated with degenerative
arthritis of the knee usually persists after arthrodesis take-down procedures and often requires total
knee arthroplasty. Pain in the contralateral hip is not resolved by converting the arthrodesis.
Improving range of motion of the hip and correcting a limb-length discrepancy are not good
indications for take-down procedures. PREFERRED RESPONSE: 2

57. A 60-year-old woman reports a painful hip arthroplasty after undergoing surgery 18 months
ago. Radiographs show stable cementless implants without signs of ingrowth. Laboratory studies
show an erythrocyte sedimentation rate of 50 mm/h. Management should now consist of

1- a technetium Tc 99m scan.


2- an indium scan.
3- an ultrasound examination.
4- aspiration.
5- revision.

DISCUSSION: Significant elevation of the erythrocyte sedimentation rate in a patient with a painful
hip arthroplasty mandates a complete work-up for infection prior to considering revision surgery.
Reproducibility and reliability of ultrasonography as a diagnostic test still needs clarification.
Aspiration is the easiest and most cost-effective test and should be performed prior to nuclear
imaging. The latter is most valuable if the results are negative, strongly predicting the absence of
infection. PREFERRED RESPONSE: 4

58. Femoral osteotomy for dysplasia of the hip will most likely result in

1- improved range of motion.


2- improved abduction strength.
3- pain relief.
4- equal limb lengths.
5- a normal gait.
71 © Copyright by American Academy of Orthopaedic Surgeons. All rights reserved. 24
American Academy of Orthopaedic Surgeons
2001 Adult Reconstructive Surgery of the Hip and Knee by Dr.Dhahirortho
PREFERRED RESPONSE: 3
DISCUSSION: Patients should expect pain relief after femoral osteotomy for hip dysplasia. Patients
should not expect improved motion or abduction strength and should be counseled about a
postoperative limp and unequal limb lengths.

59. Figure 24 shows the radiograph of an otherwise healthy 56-year-old patient who reports hip
pain after undergoing a primary cementless hip replacement 4 months ago. The next most
appropriate step should consist of

1- indomethacin for 3 months.


2- C-reactive protein and a sed rate.
3- MRI scan.
4- bone scan.
5- follow-up radiograph in 3 months.

DISCUSSION: Periosteal new bone formation is a warning sign of


prosthetic infection. Indomethacin may prevent heterotopic
ossification if given early enough; however, it is irrelevant in this
patient. A C-reactive protein and a sed rate are useful screening
studies that add to the predictive value of the radiographs and may
be performed routinely if sepsis is suspected. A bone scan
obtained 4 months after surgery would show increased uptake in
all cases. If results of a sed rate and C-reactive protein are normal,
then a biopsy should be considered to rule out a neoplasm.
PREFERRED RESPONSE: 2

60. Which of the following is considered a physiologic effect of anemia?

1- Decreased heart rate


2- Decreased coronary artery flow requirement
3- Increased cardiac output
4- Increased peripheral resistance
5- Increased blood viscosity

DISCUSSION: The expected physiologic effects of anemia include an increased heart rate and
increased cardiac output. The coronary blood flow requirement increases. There is a decrease in
peripheral resistance and blood viscosity. PREFERRED RESPONSE: 3

61. A patient with severe rheumatoid arthritis reports progressive hip pain. Serial hip radiographs
will most likely show which of the following findings?

1- Asymmetric joint space narrowing


2- Sacroiliac joint ankylosis
3- Progressive superior and lateral migration of the femoral head
4- Periarticular osteopenia
5- Hip synovitis

DISCUSSION: Radiographic findings in patients with rheumatoid arthritis include symmetric joint
space narrowing, periacetabular and femoral head erosions, and diffuse periarticular osteopenia. In
72 25
American Academy of Orthopaedic Surgeons
2001 Adult Reconstructive Surgery of the Hip and Knee
advanced stages, protrusio acetabuli is a common finding. Ranawat and associates have shown a rate
of superior femoral head migration of 4.5 mm per year and medial (axial) migration of 2.5 mm per
year. Asymmetric joint space narrowing is a classic radiographic finding of degenerative arthrosis.
Sacroiliac joint ankylosis commonly occurs in ankylosing spondylitis. Hip synovitis is a pathologic
diagnosis, not a radiographic finding. PREFERRED RESPONSE: 4

62. A 70-year-old woman reports anterior knee pain after undergoing an uncomplicated total knee
arthroplasty 6 months ago. Examination reveals prepatellar tenderness, with no extensor lag.
The radiographs shown in Figures 25a through 25c reveal a well-fixed patellar component.
Management should consist of

1- closed treatment with early motion.


2- a cylindrical cast and restricted weight bearing.
3- open reduction and internal fixation.
4- patellar revision.
5- patellectomy.

DISCUSSION: Patellar fractures that occur after a total knee arthroplasty are usually stress fractures.
Integrity of the extensor mechanism precludes the need for surgical repair or internal fixation, while
stability and fixation of the patellar component determine whether revision is indicated. A
cylindrical cast and full weight bearing for 6 weeks is recommended for transverse fractures with an
intact extensor mechanism and a stable component. A similar fracture, if vertical, may be treated
with earlier motion. PREFERRED RESPONSE: 1

63. Figure 26 shows the MRI scan of a 60-year-old man who has had groin pain for the past 2
months. The patient reports pain with ambulation, and examination reveals an antalgic gait. He
denies any history of steroid or alcohol abuse. Plain radiographs are normal. Management should
include

1- core decompression.
2- a vascularized fibula graft.
3- intraosseous steroid injection.
4- total hip replacement.
5- protected weight bearing.
73 © Copyright by American Academy of Orthopaedic Surgeons. All rights reserved. 26
American Academy of Orthopaedic Surgeons
2001 Adult Reconstructive Surgery of the Hip and Knee by Dr.Dhahirortho
PREFERRED RESPONSE: 5
DISCUSSION: The patient has transient osteoporosis of the hip. Transient osteoporosis is usually a
self-limited condition that is most frequently seen in women in the third trimester of pregnancy and
in men in the sixth decade of life. Transient osteoporosis is best treated with protected weight
bearing.

64. Which of the following is considered the most common complication of the impaction
grafting technique for femoral revision surgery?

1- Loss of fixation
2- Osteolysis
3- Bone graft resorption
4- Early stem subsidence
5- Infection

DISCUSSION: Impaction grafting technique for femoral revision surgery has become increasingly
popular over the past decade. This technique is designed to address cavitary deficiencies of the
femur. The femoral stem is inserted with cement fixation. Its clinical efficacy has not been shown to
be superior to extensively porous-coated stems. Early subsidence of the stem has been reported in
more than 50% of the patients. However, loss of fixation has occurred infrequently (5%) in reported
series conducted by experienced surgeons. It has not been shown to have a higher infection rate.
PREFERRED RESPONSE: 4

65. What is the most likely late complication associated with cementless total knee replacement?

1- Loss of motion
2- Patellofemoral pain
3- Osteolysis
4- Heterotopic ossification
5- Patellar clunk

DISCUSSION: In cementless total knee replacement, the risk of osteolysis is 30% if both
components are placed without cement and screws are used for tibial fixation. The risk is 10% when
a cemented tibial component is used, and the risk is 0% when both components are cemented. Loss
of motion, patellofemoral pain, heterotopic bone formation, and patellar clunk are complications that
can occur after cemented or cementless components are placed. PREFERRED RESPONSE: 3

66. Figure 27 shows the radiograph of a 68-year-old woman with a history of rheumatoid arthritis
who was injured in a fall. History reveals that she has been asymptomatic since undergoing a left
total knee arthroplasty 9 years ago. Management should consist of

1- skeletal traction.
2- immediate application of a cast brace.
3- a retrograde supracondylar nail.
4- revision total knee replacement.
5- resection arthrodesis.

DISCUSSION: A supracondylar fracture of the femur that occurs


after total knee replacement can be treated effectively by a number
74 27
American Academy of Orthopaedic Surgeons
2001 Adult Reconstructive Surgery of the Hip and Knee
of methods. For this fracture, the use of a retrograde supracondylar nail has been found to be
effective in several series. The treatment of these complex injuries needs to be individualized based
on the stability of the implant, the quality of the bone, and the extent of comminution of the fracture.
Revision with the use of an unstemmed implant will not result in effective stabilization of the knee or
the fracture. PREFERRED RESPONSE: 3

67. Design factors that enhance the long-term survival of proximally coated cementless hip
implants include both initial stability and

1- circumferential porous coating.


2- a titanium porous coating.
3- a fluted stem.
4- a distal centralizer.
5- modular fixation pads.

DISCUSSION: Proximally coated femoral components were conceived in response to the proximal
stress shielding seen with extensively coated total hip stems, but initial patient studies showed
problems with osteolysis, thigh pain, and stability. However, Mont and Hungerford now report that
second-generation devices that have been in use more than 5 years clinically have shown very low
aseptic loosening rates (1% to 3%), and patients report less thigh pain (less than 5% in most studies).
These results can be attributed to improved geometry, instruments, and technique, which ensure
initial implant stability. The authors suggest that proximal coating must be circumferential to seal the
diaphysis from wear debris, and they note that the concept of proximal coating for cementless
femoral stems seems viable as long as the twin requirements of circumferential coating and rigid
initial stability are realized. PREFERRED RESPONSE: 1

68. A 45-year-old man with a painful varus knee is being considered for an upper tibial
osteotomy. Which of the following factors is considered the most compelling argument
against this procedure?

1- Flexion contracture of 5°
2- Subchondral cyst in the medial tibial condyle
3- Lateral meniscal degeneration seen in an MRI scan
4- Rheumatoid arthropathy
5- Previous medial meniscectomy

DISCUSSION: Proximal tibial osteotomy is appropriate for the younger and/or athletic patient who
has mild to moderate medial compartment osteoarthritis. Relative contraindications include limited
range of motion (eg, flexion contracture of 15°), anatomic varus of greater than 10°, advanced
patellofemoral arthritis, and tibial subluxation. Inflammatory arthritides involve all the
compartments and are a contraindication to osteotomies around the knee. PREFERR RESPONSE: 4

69. An obese patient undergoing total knee arthroplasty is at increased risk for which of the
following complications?
1- Wound complication
2- Infection
3- Lower knee score
4- Aseptic loosening
5- Patellar subluxation
75 © Copyright by American Academy of Orthopaedic Surgeons. All rights reserved. 28
American Academy of Orthopaedic Surgeons
2001 Adult Reconstructive Surgery of the Hip and Knee by Dr.Dhahirortho
PREFERRED RESPONSE: 1
DISCUSSION: The rate of wound complications is significantly increased after total knee
arthroplasty in obese patients. Knee scores and the rate of aseptic loosening or patellar subluxation
do not appear to be significantly altered.

70. Figures 28a and 28b show the radiographs of a 79-year-old man who has constant knee pain.
Prior to performing elective knee replacement surgery, management should include

1- reduction of the serum alkaline phosphatase level by 50%.


2- preoperative radiation therapy of 600 cGy to the surgical site.
3- aspiration of the knee joint with cell count.
4- insertion of a vena caval filter.
5- administration of 25 mg of indomethacin three times a day.

DISCUSSION: The radiographs show established Paget’s disease. Bony expansion is evident, with
thickened trabeculae consistent with the disordered bone remodeling process. A reduction of the
serum alkaline phosphatase level to 50% of the pretreatment level may reduce pain from Paget’s
disease, and it is recommended prior to consideration of joint replacement. In elective cases,
treatment of Paget’s disease should begin at least 6 weeks prior to surgery. The other modalities are
not related to the treatment of Paget’s disease. PREFERRED RESPONSE: 1

71. What is the most common complication of using structural bulk allograft to reconstruct
segmental defects of the acetabulum?

1- Infection
2- Early loss of cup fixation
3- Graft resorption and collapse
4- Limb-length discrepancy
5- Dislocation

DISCUSSION: Both autograft and allograft have been used for complex acetabular reconstructions.
They have been shown to be successful in the short term. However, graft resorption with collapse
and subsequent cup loosening have occurred at high rates for both types of grafts, especially if
reinforcement rings or cages are not used.

PREFERRED RESPONSE: 3

76 29
American Academy of Orthopaedic Surgeons
2001 Adult Reconstructive Surgery of the Hip and Knee
72. Radiographs of a 12-year-old boy who has knee pain show a 2-cm osteochondral lesion of the
lateral aspect of the medial femoral condyle. The fragments are not detached from the femur. Initial
management should consist of

1- casting in flexion.
2- observation.
3- arthroscopic drilling and pinning of the lesion.
4- removal and reattachment of the osteochondral lesion.
5- allograft transplantation for the lesion.

DISCUSSION: For a pediatric patient without mechanical symptoms, initial management of an


osteochondral defect lesion that is not detached should consist of casting in flexion. Failure to
respond to several weeks or months of nonsurgical management may warrant surgical treatment.
PREFERRED RESPONSE: 1

73. Which of the following drawbacks is associated with the Ganz periacetabular osteotomy?

1- The tendency to anterior displacement of the hip joint


2- The need for two incisions
3- Limited potential for acetabular reorientation
4- Posterior column disruption
5- Devascularization of the acetabulum

DISCUSSION: Although technically challenging, the Ganz periacetabular osteotomy offers


advantages over other rotational pelvic osteotomies. Posterior column integrity is maintained, as is
the acetabular vascular supply. Free mobility of the fragment makes large corrections in the center
edge angle possible. Because of the asymmetric cuts and the need to restore anterior coverage, there
is a tendency to anterior displacement of the joint while flexing the acetabulum. The procedure is
commonly performed through a Smith-Petersen incision. PREFERRED RESPONSE: 1

74. Which of the following lesions is best suited for autologous chondrocyte implantation?

1- Patellofemoral arthritis
2- Lateral femoral condylar arthritis
3- Medial femoral condylar arthritis
4- Medial femoral chondral defect
5- Medial femoral and tibial articular chondral matching defects

DISCUSSION: Articular chondrocyte implantation is best performed for focal chondral defects of
one area of the joint. It is not indicated for osteoarthritis. PREFERRED RESPONSE: 4

75. The additional risk of complications in organ transplant patients receiving a total joint
arthroplasty is attributed to

1- infection.
2- dislocation.
3- deep venous thrombosis.
4- periprosthetic fracture.
5- myocardial infection.
77 © Copyright by American Academy of Orthopaedic Surgeons. All rights reserved. 30
American Academy of Orthopaedic Surgeons
2001 Adult Reconstructive Surgery of the Hip and Knee by Dr.Dhahirortho

PREFERRED RESPONSE: 1
DISCUSSION: Tannenbaum and associates found that patients who had a joint replacement after an
organ transplantation had a rate of infection of 19% (five of 27 joint replacements in 16 patients).
They retrospectively reviewed the results of 35 joint (hip or knee) replacements in 19 patients who
had an organ transplant. The patients received a standard immunosuppressive induction regimen at
the time of the transplantation and were maintained on a combination of prednisone, azathioprine,
and cyclosporin A. All patients received antibiotics perioperatively, but antibiotic-impregnated bone
cement was not used for any procedure. Six joint replacements in three patients (median patient age
of 48.2 years at the time of the arthroplasty) were performed before a renal transplantation. Twenty-
four joint replacements in 14 patients (average patient age of 40.9 years at the time of the
arthroplasty) were performed after an organ transplantation. Two patients, with an average age of
53.8 years at the time of the arthroplasty, each had a joint replacement both before and after a liver
transplantation (a total of five joint replacements). The average duration of follow-up after the first
joint replacement was 8.8 years (range, 1 to 23 years). An infection developed around the implant in
five patients who had undergone the joint replacement after a transplantation. The average interval
from implantation of the prosthesis until detection of the infection was 3.4 years (range, 1 to 6 years).
Of two patients who underwent a liver transplant, one had Pseudomonas aeruginosa infection and the
other Escherichia coli infection. Of three patients who underwent a renal transplantation, one was
infected with Staphylococcus epidermidis, one with Enterococcus, and one with Serratia marcescens.

76. Figure 29 shows the radiograph of a 55-year-old patient who has recurrent total hip
dislocation. Dislocation is most likely to occur in this patient when the hip is in which of the
following positions?

1- Neutral rotation
2- External rotation
3- Internal rotation
4- Hyperflexion
5- Midstance phase of gait

DISCUSSION: The patient has an acetabular component


that is placed in excessive anteversion; this is confirmed
by the shoot-through radiograph. The most common
reasons for dislocation of a total hip replacement include inappropriate positioning of the
components, inadequate abductor tension, or impingement. Implants placed without adequate total
anteversion tend to dislocate posteriorly, and implants with excessive anteversion tend to dislocate
anteriorly. Superior dislocations can occur if the acetabular component is placed in a severely
vertical position with inadequate lateral coverage. PREFERRED RESPONSE: 2

77. Back pain and ipsilateral knee pain are common long-term sequelae of hip arthrodesis. To
limit these problems, what position should be avoided during fusion of the hip?

1- Flexion
2- Abduction
3- Adduction
4- External rotation
5- Internal rotation

78 31
American Academy of Orthopaedic Surgeons
2001 Adult Reconstructive Surgery of the Hip and Knee
PREFERRED RESPONSE: 2
DISCUSSION: The recommended position for a hip fusion is flexion of 20° to 30°, slight adduction
(5°) or neutral, and 10° of external rotation. In long-term follow-up, patients who underwent fusion
in abduction had more ipsilateral knee and low back pain than patients who were positioned in
adduction. Internal rotation should be avoided to prevent interference with the opposite foot during
gait. External rotation facilitates the application of shoe wear.

78. Which of the following methods most reliably detects mechanical loosening of the hip?

1- Serial planar radiographs


2- Joint aspiration
3- Aspiration and arthrogram
4- Technetium Tc 99m scan
5- CT scan

DISCUSSION: Mechanical loosening of the hip is best revealed by serial radiographs of the
prosthetic joint. None of the other methods of evaluation is considered reliable in diagnosing
mechanical loosening. PREFERRED RESPONSE: 1

79. A 55-year-old man underwent cementless total hip arthroplasty for advanced painful
osteoarthritis of the hip 2 years ago. The follow-up radiograph shown in Figure 30 shows

1- spot welds and calcar atrophy.


2- subsidence.
3- distal cortical hypertrophy.
4- distal pedestal formation.
5- complete lucent line around the stem.

DISCUSSION: The radiograph shows a well-osseointegrated tapered


stem with a metaphyseal porous coating, spot welds in the porous
region, and calcar rounding. Trochanteric stress shielding and distal
cortical hypertrophy are also signs of ingrown stems but are seen more
frequently in association with extensively porous-coated stems
exhibiting diaphyseal ingrowth. There is no evidence of lucent lines or a pedestal, signs that suggest
instability. Femoral stem subsidence can be determined only by a review of sequential radiographs.
PREFERRED RESPONSE: 1

80. A 52-year-old woman has bicompartmental osteoarthritis following patellectomy. Treatment


should consist of

1- high tibial osteotomy.


2- arthroscopic debridement.
3- patella arthroplasty.
4- total knee arthroplasty.
5- knee arthrodesis.

DISCUSSION: The patient has extensive degenerative changes in both the medial and lateral
compartments within the knee; therefore, arthroscopic debridement or an osteotomy will not be
helpful. A patellar arthroplasty will not address the medial and lateral compartments. Because the
79 © Copyright by American Academy of Orthopaedic Surgeons. All rights reserved. 32
American Academy of Orthopaedic Surgeons
2001 Adult Reconstructive Surgery of the Hip and Knee by Dr.Dhahirortho
extensor mechanism provides a significant amount of anteroposterior stability, a posterior cruciate-
substituting total knee arthroplasty is the treatment of choice for this patient. PREFE RESPON: 4

81. In hybrid arthroplasty, the use of a polymethylmethacrylate (PMMA) precoated femoral


component has been shown to result in

1- increased survivorship compared with nonprecoated stems.


2- increased bonding of the stem to the cement mantle.
3- a reduced rate of wear compared with nonprecoated stems.
4- a reduced rate of revision compared with nonprecoated stems.
5- a reduced rate of postoperative infection.

DISCUSSION: Precoating of the femoral stem with PMMA results in increased bonding of the stem
to the cement mantle. However, this has not been shown to result in superior survivorship compared
with nonprecoated stems of similar design. In one series, the rate of revision of precoated stems was
greater than that of nonprecoated cohorts. The wear and infection rates have not been shown to differ
between precoated and nonprecoated stems. PREFERRED RESPONSE: 2

82. A 72-year-old woman has had progressively increasing pain in the right knee for the past 6
months. She denies any trauma and has no pain in any other joints, but she notes occasional swelling
in the knee and a catching sensation. Figures 31a and 31b show the plain radiographs and Figure 31c
shows the MRI scan. Treatment should consist of

1- arthroscopy and subtotal meniscectomy.


2- arthroscopy and shaving chondroplasty.
3- osteochondral bone graft.
4- high tibial valgus osteotomy.
5- total knee replacement.

DISCUSSION: The plain radiograph shows a defect in the lateral femoral condyle and narrowing of
the lateral joint space. The MRI scan shows a lesion consistent with osteonecrosis of the lateral
femoral condyle. The treatment alternatives for this condition are an osteotomy or a total knee
replacement, but a total knee replacement is the treatment of choice for a 72-year-old patient.
Arthroscopy or an osteochondral bone graft will not address her symptoms. A valgus osteotomy will
exacerbate the problem by overloading the lateral joint, which is already diseased.
PREFERRED RESPONSE: 5
80 33
American Academy of Orthopaedic Surgeons
2001 Adult Reconstructive Surgery of the Hip and Knee
83. Which of the following is considered the most appropriate indication for conversion of a hip
fusion to total hip arthroplasty?

1- Moderate arthritis of the ipsilateral knee


2- Progressive arthritis of the contralateral hip
3- Severe disabling back pain
4- Limb-length discrepancy
5- Increased hip motion

DISCUSSION: Hip fusion provides successful long-term results (20 to 30 years). The usual mode of
failure is symptomatic arthrosis of the lower back, contralateral hip, or the ipsilateral knee. Disabling
low back pain is the best indication for conversion and responds well to the procedure. Degenerative
changes in the other joints do not respond as well and frequently require replacement arthroplasty.
Restoration of limb length is not predictable after conversion to hip replacement.
PREFERRED RESPONSE: 3

84. Which of the following methods is effective in correcting recurrent dislocation following total
hip arthroplasty?

1- Lateralization of the greater trochanter without advancement


2- Use of a shorter neck length
3- Use of a constrained acetabular component
4- Use of a small diameter head
5- High cup abduction angle

DISCUSSION: Recurrent dislocation following total hip arthroplasty is a difficult problem to correct.
Studies conducted by the Mayo Clinic show a failure rate of close to 40% with surgical treatment. A
variety of methods have been successful, but no specific approach has been reported to be the most
predictably successful. To select and institute the proper treatment option, the cause of the
dislocation must be identified. Surgical options fall into several broad categories that include
increasing soft-tissue tension (trochanteric advancement or longer neck lengths) or more stable
articulation (larger diameter head component, bipolar prosthesis, or a constrained component). In a
series of total hip arthroplasties done with a constrained cup, the loosening rates of the cup and the
stem were reported to be 6% each, comparable to a reported series of complex revision total hip
arthroplasties at a similar follow-up interval. PREFERRED RESPONSE: 3

85. A 58-year-old woman who underwent a successful total hip replacement for degenerative
arthritis 8 years ago reports groin pain for the past 6 months. A radiograph of the hip is
shown in Figure 32. At revision, severe deficiency of the posterior column is noted. What
reconstructive option would be most appropriate for the acetabulum?

1- Cementless cup without graft


2- Cemented cup without graft
3- Cemented cup with structural bone graft
4- Bone graft, reconstruction cage, and cemented cup
5- Bilobed cementless acetabular component

81 © Copyright by American Academy of Orthopaedic Surgeons. All rights reserved. 34


American Academy of Orthopaedic Surgeons
2001 Adult Reconstructive Surgery of the Hip and Knee by Dr.Dhahirortho
DISCUSSION: The radiograph shows medial migration of the cementless acetabular component,
strongly suggesting acetabular discontinuity with a combined segmental and cavitary medial
deficiency. The treatment of choice is a morcellized or structural graft, supported with a
reconstructive cage bridging the pelvic discontinuity, and a cemented cup.
PREFERRED RESPONSE: 4

86. A patient with a previously pain-free knee replacement now reports a sudden inability to
ambulate. Radiographs of the knee are shown in Figures 33a and 33b. Management should
consist of

1- bracing and physical therapy.


2- insertion of a thicker polyethylene insert.
3- revision with a cementless modular prosthesis.
4- revision with a cemented semiconstrained prosthesis.
5- reconstruction of the extensor mechanism.

DISCUSSION: The radiographs show a patellar tendon rupture following a total knee replacement.
This infrequent, but serious, complication is reported to occur in 0.17% to 1.4% of patients after total
knee arthroplasty. Although the radiographs show concerning features such as incomplete tibial and
femoral periprosthetic lucencies, it is most important for the surgeon to recognize extensor
mechanism disruption. PREFERRED RESPONSE: 5

. 87. Which of the following factors will adversely affect bone ingrowth in a revision porous-
coated stem?

1- Pore size of 400 m


2- Interface instability of 25 m of micromotion
3- Use of a nonmodular implant
4- Noncircumferential metaphyseal patch coating
5- Failure of ingrowth in the previous stem

DISCUSSION: The optimal conditions for bony ingrowth include a pore size of 100 to 400 m,
interface micromotion of 50 m or less, intimate contact between the bone and the implant,
circumferential porous coating of the implant, and use of a biocompatible material. Stem designs
with patch coatings have a poor record of bony ingrowth, especially in the revision setting. Failure of

82 35
American Academy of Orthopaedic Surgeons
2001 Adult Reconstructive Surgery of the Hip and Knee
ingrowth in the previous stem would be the result of its own mechanical milieu and would not
necessarily predict results for the new stem. PREFERRED RESPONSE: 4

88. In the preoperative planning of revision acetabular reconstruction, the surgeon should identify
significant posterior column deficiency by noting which of the following radiographic
features?

1- Excessive vertical position of the acetabular component


2- Medial displacement of the hip center to the ilioischial line
3- Visible wear of the polyethylene articulation
4- Osteolysis in the ischium
5- Superior migration of 1 cm

DISCUSSION: Proximal and medial migration of the femoral head usually indicates deficiencies of
the dome or anterior column. Wear of the polyethylene may result in osteolysis and impingement,
which are not indicative of any major bone deficiency. A significant osteolytic lesion in the ischium
may represent a major posterior column deficiency that can create a technical challenge during the
reconstruction. PREFERRED RESPONSE: 4

89. An 82-year-old woman reports right buttock pain after a car trip. Laboratory studies show an
erythrocyte sedimentation rate of 30 mm/h and WBC of 4,600/mm3. Figure 34a shows a plain AP
radiograph of the pelvis, and Figure 34b shows a delayed technetium Tc 99m bone scan.
Management should consist of

1- bed rest and pain medication.


2- revision of the right acetabular component.
3- revision of the right femoral component.
4- revision of the right total hip replacement.
5- resection arthroplasty.

DISCUSSION: The radiograph shows bilateral cemented total hip arthroplasties. The acetabular
components are loose bilaterally, but there has been no acute change. Therefore, it is unlikely that
the acetabular loosening is contributing to the patient’s pain. The bone scan is consistent with a
sacral insufficiency fracture. This is best treated with bed rest and pain medication. Activity can be
increased as the pain allows. Revision will not address the pain.

PREFERRED RESPONSE: 1
83 © Copyright by American Academy of Orthopaedic Surgeons. All rights reserved. 36
American Academy of Orthopaedic Surgeons
2001 Adult Reconstructive Surgery of the Hip and Knee by Dr.Dhahirortho

90. Figures 35a and 35b show the radiographs of a patient who underwent debridement of a
chronically infected, fully constrained knee prosthesis and now reports pain and instability
despite bracing. History reveals that the patient has had no drainage since undergoing the last
debridement 6 months ago. A C-reactive protein level and aspiration are negative for
infection. Treatment should now consist of

1- knee arthrodesis.
2- insertion of a semiconstrained prosthesis.
3- insertion of an antibiotic-impregnated polymethylmethacrylate (PMMA) spacer.
4- reconstruction of the extensor mechanism.
5- amputation.

DISCUSSION: The radiographs show a significant loss of the proximal anterior tibial cortex,
consistent with an extensively damaged or deficient extensor mechanism. Such a deficit precludes
insertion of another knee arthroplasty. Arthrodesis is the treatment of choice for this patient and is
indicated for loss of the extensor mechanism and knee instability. A recent report on arthrodesis
following removal of an infected prosthesis showed a union rate of 91% using a short intramedullary
nail. Insertion of an antibiotic-impregnated PMMA spacer is not indicated because the rationale for
using a spacer is to maintain a space for reinsertion of another prosthesis. Reconstruction of the
extensor mechanism would not address the loss of the joint. Amputation is the final treatment option
if the arthrodesis fails. PREFERRED RESPONSE: 1

91. Which of the following is considered an advantage of metal femoral heads compared with
ceramic heads?

1- Superior lubrication properties


2- Smoother surface
3- Less susceptible to third body wear
4- More inert material
5- Greater neck-length options

DISCUSSION: Ceramic-on-ceramic bearing surfaces have superior tribological properties and show
lower linear wear than metal-on-metal implants. However, because of their lower strength and
vulnerability to fracture, design considerations constrain the neck-length options available to ensure
optimal taper fit. PREFERRED RESPONSE: 5
84 37
American Academy of Orthopaedic Surgeons
2001 Adult Reconstructive Surgery of the Hip and Knee
92. What is the most common result if the acetabulum is rotated too far anteriorly during a
periacetabular osteotomy?

1- Posterior dislocation
2- Limited hip flexion
3- Heterotopic ossification
4- Femoral nerve injury
5- Fracture of the posterior column

DISCUSSION: In patients with hip dysplasia who undergo a periacetabular osteotomy, the authors
note that the freed acetabular segment can be overcorrected for the deformity. If it is placed too
anteriorly, then hip flexion is limited. Posterior dislocation is a rare complication. The other
complications should not occur as a result of this procedure. PREFERRED RESPONSE: 2

93. Which of the following radiographic views best assesses anterior coverage of the dysplastic
hip?

1- AP of the hip
2- Obturator oblique
3- Lauenstein lateral
4- Faux profil
5- Pelvic inlet

DISCUSSION: Anterior coverage of the hip may be best estimated by the anterior center edge angle
of Lequesne and de Seze (analogous to Wiberg’s angle), which is measured on the well-defined faux
profil view. Evaluation with CT scans also has been described. PREFERRED RESPONSE: 4

94. Figure 36a shows the current radiograph of a 65-year-old woman who slipped and fell.
History reveals that prior to the fall she was actively functioning without pain. Figure 36b shows a
radiograph obtained 1 year ago. Based on the fracture pattern, the failure is most likely related to

1- repetitive loading and fatigue failure.


2- incomplete bone ingrowth with focal osteolysis.
3- rotational bone axial loading.
4- a fixed component with a modulus mismatch.
5- use of titanium instead of cobalt-chromium.
85 © Copyright by American Academy of Orthopaedic Surgeons. All rights reserved. 38
American Academy of Orthopaedic Surgeons
2001 Adult Reconstructive Surgery of the Hip and Knee by Dr.Dhahirortho

PREFERRED RESPONSE: 4
DISCUSSION: The radiograph shows a fracture distal to the prosthesis in a stable, apparently well-
fixed prosthetic stem. The well-fixed prosthesis-bone composite is stiff, creating a modulus
mismatch between the proximal and distal femur. Therefore, the risk of fracture, particularly in
osteoporotic bone, is increased at this level. Revision of the stem to a longer construct is
unnecessary, and standard plate and screw fixation has been shown to yield union rates of greater
than 90%. Nonsurgical treatment of fractures distal to the tip of the prosthesis results in high
nonunion rates, reported to be from 25% to 42%.

95. A 70-year-old man has worn through his metal-backed patellar component and sustained
damage to the femoral component. Following removal of the components and debridement of the
metal-stained synovium, the surgeon finds that the thickness of the remaining patella is 10 mm.
Treatment should now include

1- insertion of a thicker cement mantle and a thicker patellar insert to achieve a total
patellar thickness of 24 mm.
2- a lateral release after inserting a standard patella.
3- a distal femoral augmentation to maximize the moment-arm on a standard patellar
insert.
4- leaving the patella alone and performing a lateral release, if necessary, for proper
patellar tracking.
5- an oversized femoral component to improve the moment-arm on a standard patellar
insert.

DISCUSSION: Revision of a failed patellar component can be difficult because of bone loss and
damage to the extensor mechanism. Several authors have advised against reinsertion of a patellar
component if the residual patellar thickness is 10 mm or less. Leaving an unresurfaced bony remnant
in place at the time of revision or reimplantation surgery has been shown to be a reasonable option;
however, the results are of a lower quality when compared with revision surgery where the patellar
component can be retained or revised. The other treatment options have not been shown to be
effective approaches to this problem. PREFERRED RESPONSE: 4

96. A 65-year-old man has a painful and often audible crepitus after undergoing a total knee
arthroplasty 8 months ago. His symptoms are reproduced with active extension of about 30°.
Examination reveals no effusion or localized tenderness, a stable knee, and a range of motion of 5° to
120°. Radiographs are shown in Figures 37a and 37b. Management should consist of

86 39
American Academy of Orthopaedic Surgeons
2001 Adult Reconstructive Surgery of the Hip and Knee
1- revision of all components to ensure patellar tracking.
2- athroscopic debridement.
3- arthrotomy and keloid excision.
4- intra-articular corticosteroid injections.
5- patellar component revision.

DISCUSSION: This is a typical presentation of the patellar clunk syndrome. The syndrome usually
follows implantation of a posterior stabilized prosthesis. It is thought to be the result of femoral
component design and altered extensor mechanics. The condition usually resolves with arthroscopic
debridement of the suprapatellar fibrous nodule. Arthrotomy or revision is seldom warranted.

PREFERRED RESPONSE: 2

97. What clinical parameter will most likely decrease the need for blood transfusion after total
joint arthroplasty?

1- Bilateral total joint replacement


2- Rheumatoid arthritis
3- Preoperative donation of autologous blood
4- Age greater than 65 years
5- Hemoglobin level of greater than 15 g/dL

DISCUSSION: Bilateral joint replacement, chronic disease, and preoperative autologous donation all
increase the risk of needing blood. Young patients and a high hemoglobin level (greater than 15
g/dL) are considered clinical parameters that decrease the risk for requiring allogenic blood.

PREFERRED RESPONSE: 5

98. Figure 38 shows the radiograph of a 40-year-old woman who reports severe groin pain and
lack of motion of the right hip. History reveals that the patient underwent a femoral
osteotomy for hip dysplasia approximately 30 years ago. Treatment should include

1- femoral osteotomy.
2- periacetabular osteotomy.
3- arthroscopic debridement.
4- total hip arthroplasty.
5- hip arthrodesis.

DISCUSSION: Although the patient is young, a total


hip arthroplasty will provide pain relief and improve
her range of motion. The arthritis is too advanced for
the patient to benefit from an osteotomy. In addition,
periacetabular osteotomy and hip arthrodesis do not
improve range of motion of the hip. It has not been
established that patients with severe osteoarthritis will
benefit from arthroscopic debridement of the hip.

PREFERRED RESPONSE: 4

87 © Copyright by American Academy of Orthopaedic Surgeons. All rights reserved. 40


American Academy of Orthopaedic Surgeons
2001 Adult Reconstructive Surgery of the Hip and Knee by Dr.Dhahirortho
99. What is the primary concern for arthrodesis of a failed infected total knee arthroplasty using
internal fixation?

1- Recurrent infection
2- Lack of stability
3- Lack of soft-tissue coverage
4- Stress shielding
5- Stress risers

DISCUSSION: Arthrodesis of the failed infected total knee arthroplasty may be accomplished by
external fixation, intramedullary rod fixation, and dual plates and screws. External fixation runs the
risk of pin tract infection, although after its removal, there are no metal surfaces left in place.
Intramedullary rods have been used successfully in the treatment of infected total knees, although
they also leave metal within the region of the infection. The dual plate technique of knee fusion is
useful in patients with rheumatoid arthritis who require fusion in the absence of infection because it
provides good initial stability and avoids the use of external pins. However, in the face of infection,
the large surface area of the screws and plates may serve as a site for bacteria to hide within a
glycocalyx and make eradication of the infection almost impossible.

PREFERRED RESPONSE: 1

100. Oxidation of polyethylene after sterilization occurs most rapidly when the implant undergoes

1- gamma radiation in air.


2- gamma radiation in nitrogen.
3- gamma radiation in argon.
4- gas plasma exposure.
5- ethylene oxide exposure.

DISCUSSION: The use of gamma radiation to sterilize polyethylene will result in the formation of
free radicals in the material that increase the susceptibility of the material to oxidation and wear. The
packaging can also have an impact. If the polyethylene is packaged in air, the oxygen in the
packaging can significantly oxidize the material on the shelf prior to clinical use. Gas plasma and
ethylene oxide sterilization do not appear to increase oxidation of polyethylene.

PREFERRED RESPONSE: 1

88 41
American Academy of Orthopaedic Surgeons

89
American Academy of Orthopaedic Surgeons 1
2001 Pediatric Orthopaedic Self-Assessment Examination.by Dr.Dhahirortho
1. The inheritance of the deformity shown in Figure 1 is most commonly

1- autosomal-recessive.
2- autosomal-dominant.
3- x-linked dominant.
4- mitochondrial.
5- sporadic.

DISCUSSION: Cleft hand and cleft foot


malformations are commonly inherited
as autosomal-dominant traits and are
associated with a number of syndromes.
An autosomal-recessive and an x-linked
inheritance pattern have also been
described, but these are much less common and are usually atypical. In the common autosomal-
dominant condition, nearly one third of the known carriers of the gene show no hand or foot
abnormalities. This is known as reduced penetrance. The disorder may be variably expressed; affected
family members often exhibit a range from mild abnormalities in one limb only to severe anomalies in
four limbs. Variable expressivity and reduced penetrance can cause difficulty in counseling families
regarding future offspring in an affected family. Many patients have a cleft hand that may be caused
by the split-hand, split-foot gene (SHFM1) localized on chromosome 7q21.
PREFERRED RESPONSE: 2

2. Examination of a 12-year-old girl with bilateral anterior knee pain reveals excessive femoral
anteversion and excessive external tibial torsion. The patient has no patellofemoral instability.
Nonsurgical management consisting of muscle strengthening and nonsteroidal medication has
failed to relieve the patient’s pain. Treatment should now consist of

1- corrective osteotomies with internal rotation of the distal part of the tibia and/or external
rotation of the femur.
2- external rotation of the distal part of the tibia.
3- internal rotation of the distal part of the femur.
4- arthroscopic retinacular release.
5- a patellar realignment procedure.

DISCUSSION: Children with symptomatic severe torsional malalignment of the lower extremity and
patellofemoral pathology show excessive femoral anteversion and external tibial torsion on physical
examination and analysis of gait. The functional effect of this torsional malalignment is centered about
the knee joint. If nonsurgical management fails to alleviate patellofemoral pain, definitive surgical
treatment should consist of corrective osteotomies, including internal rotation of the distal part of the
tibia or external rotation of the femur, or both. Patients with surgical correction by osteotomy show an
improved gait pattern and appearance of the extremity and a marked decrease in knee pain. External
rotation of the distal part of the tibia or internal rotation of the distal part of the femur worsens the
torsional malalignment. No additional soft-tissue realignment procedures, including retinacular release
or patellar realignment, are required.
PREFERRED RESPONSE: 1
90 1
American Academy of Orthopaedic Surgeons 2
2001 Pediatric Orthopaedic Self-Assessment Examination.by Dr.Dhahirortho
3.Which of the following patients with cerebral palsy is considered the ideal candidate for a selective
dorsal rhizotomy?

1- An ambulatory 6-year-old patient with spastic diplegia


2- An ambulatory 10-year-old patient with spastic right hemiplegia
3- An ambulatory 16-year-old patient with spastic diplegia
4- A nonambulatory 8-year-old patient with spastic quadriplegia
5- A nonambulatory 18-year-old patient with rigid quadriplegia

DISCUSSION: The enthusiasm with which dorsal rhizotomy was received led to the broadening of
selection criteria with poorer results. The ideal candidate is an ambulatory 4- to 8-year-old child with
spastic diplegia who does not use assistive devices or have joint contractures. The child must be old
enough to actively participate in the rigorous postoperative physical therapy program. The use of the
procedure in an ambulatory 16-year-old patient is less desirable because joint contractures will most
likely have developed to a varying degree. The hemiplegic child is best treated by orthopaedic
interventions. PREFERRED RESPONSE: 1

4.A 3-year-old boy sustains a complete paralysis following a high thoracic spinal cord injury consistent
with a SCIWORA-type injury (spinal cord injury without radiographic abnormality). Subsequent
progressive spinal deformity will develop in what percent of patients with this injury?

1- 10%
2- 25%
3- 50%
4- 75%
5- Greater than 75%

DISCUSSION: Spinal cord injury in skeletally immature patients almost always leads to the
development of paralytic spinal deformity. The age at injury is the most important factor affecting the
development of scoliosis. Spinal cord injury that occurs more than 1 year prior to skeletal maturity is
almost always followed by the development of scoliosis. In one study, scoliosis developed in 100% of
children who were younger than age 10 years at the time of spinal cord injury. Scoliosis can occur
after injury at any level. Spasticity is often a contributing factor. Up to two thirds of patients who have
paralytic scoliosis prior to skeletal maturity will eventually require surgery for curve control.
PREFERRED RESPONSE: 5

5.A 12-year-old girl has progressive development of cavus feet. Examination reveals slightly
diminished vibratory sensation on the bottom of the foot. Reflexes are 1+ at the knees and ankles.
Motor examination shows that all muscles are 5/5 in the foot, except the peroneal and anterior tibial
muscles are rated as 4+/5. Which of the following studies is considered most diagnostic?

1- Nerve conduction velocity studies


2- Biopsy of the quadriceps femoris muscle
3- Biopsy of the sural nerve
4- DNA testing
5- Chromosomal analysis

91 2
American Academy of Orthopaedic Surgeons 3
2001 Pediatric Orthopaedic Self-Assessment Examination.by Dr.Dhahirortho
PREFERRED RESPONSE: 4
DISCUSSION: The patient most likely has a form of Charcot-Marie-Tooth disease, or hereditary motor
sensory neuropathy (HMSN). The most common varieties can now be diagnosed by DNA testing.
Mutations have been detected in the peripheral myelin protein-22 (PMP-22) gene in HMSN type IA
and in the connexin gene in the x-linked HMSN. Specific DNA diagnosis is useful in genetic
counseling. Routine chromosomal testing most likely would not detect these mutations. Nerve
conduction velocity study results are normal in some types of HMSN, and delayed nerve conduction,
when found, indicates a peripheral neuropathy but does not specify the type or inheritance pattern.
Biopsy of the sural nerve or of the quadriceps can be informative in some patients, but is not as specific
as DNA testing. These procedures are most often reserved for patients with negative DNA test results.

6.A 2-year-old girl was born with the toe deformity shown in Figure 2. She has difficulty wearing
shoes despite having adequate room in the toe box. Management at this time should consist of

1- stretching exercises, followed by taping of the toes in a derotational maneuver.


2- lengthening of the extensor digitorum longus tendon of the second toe and release of the
metatarsophalangeal joint dorsal capsule.
3- resection arthroplasty of the proximal interphalangeal joint of the third toe, with release
of the volar plate.
4- tenotomy of the flexor digitorum longus and brevis of the third toe.
5- a Girdlestone-Taylor transfer of the flexor digitorum longus to the extensor digitorum
longus of the third toe.

DISCUSSION: The patient has a congenital curly toe deformity of the third toe, and tenotomy of the
toe flexors is highly effective for this problem. Stretching and taping are ineffective for this deformity.
The position of the second toe is secondary; therefore, procedures on that toe are unnecessary and
ineffective. The flexor to extensor transfer is a more complicated procedure that produces negligible
results, or may even worsen the deformity. Resection arthroplasty is contraindicated because it causes
abnormal growth of the toes. PREFERRED RESPONSE: 4

7.Figure 3 shows the radiograph of an asymptomatic 10-year-old boy. Management should consist of

1- physical therapy.
2- restriction from contact sports.
3- periodic observation, but no activity restriction.
4- immobilization with a thoracolumbosacral orthosis (TLSO).
5- direct surgical repair.

92 3
American Academy of Orthopaedic Surgeons 4
2001 Pediatric Orthopaedic Self-Assessment Examination.by Dr.Dhahirortho
PREFERRED RESPONSE: 3DISCUSSION: Asymptomatic spondylolysis in a child or adolescent
should be observed for the possible development of spondylolisthesis, but no other active intervention
is needed. The initial treatment of choice for symptomatic spondylolysis includes rest and activity
modifications, nonsteroidal anti-inflammatory drugs, physical therapy, bracing, and casting.
Immobilization with a TLSO or pantaloon spica cast may permit healing of an acute pars fracture.
Rarely, surgical treatment may be necessary. Surgical options include posterolateral L5-S1 fusion or
direct repair of the pars defect.

8.A 12-year-old girl has had lower back pain for the past 6 months that interferes with her ability to
participate in sports. She denies any history of radicular symptoms, sensory changes, or bowel or
bladder dysfunction. Examination reveals a shuffling gait, restriction of forward bending, and tight
hamstrings. Radiographs show a grade III spondylolisthesis of L5 on S1, with a slip angle of 20°.
Management should consist of

1- brace treatment.
2- laminectomy, nerve root decompression, and in situ fusion of L4 to the sacrum.
3- in situ fusion of L4 to the sacrum.
4- excision of the L5 lamina.
5- physical therapy.

DISCUSSION: Indications for surgical treatment of spondylolisthesis include pain and/or progression
of deformity. Specifically, surgery is necessary when there is persistent pain or a neurologic deficit
that fails to respond to nonsurgical therapy, there is significant slip progression, or the slip is greater
than 50%. For patients with mild spondylolisthesis, in situ posterolateral L5-S1 fusion is adequate. In
patients with more severe slips (greater than 50%), extension of the fusion to L4 offers better
mechanical advantage. Postoperative immobilization may be achieved with instrumentation, casting,
or both. In patients with a slip angle of greater than 45°, reduction of the lumbosacral kyphosis with
instrumentation or casting is desirable to prevent slip progression. Laminectomy alone is
contraindicated in a child. Nerve root decompression is indicated if radiculopathy is present clinically.
PREFERRED RESPONSE: 3

9.Figures 4a through 4c show the clinical photographs and radiographs of a 12-month-old boy who has
progressive difficulty wearing shoes because of the length of the second toe, as well as width of the
forefoot. Management should consist of

1- form-fitted shoes.
2- amputation of the second toe at the metatarsophalangeal joint.
3- amputation of the first ray and amputation of the second toe.
4- amputation of the second ray.
5- an MRI scan of the foot, a CT scan of the chest, and a biopsy of the foot with the
possibility of ankle disarticulation amputation.
93 4
American Academy of Orthopaedic Surgeons 5
2001 Pediatric Orthopaedic Self-Assessment Examination.by Dr.Dhahirortho
PREFERRED RESPONSE: 4
DISCUSSION: The patient has macrodactyly involving the second ray, with significant enlargement of
the width and height of the foot. The radiographs show widening of the interval between the first and
second metatarsal and between the second and third metatarsal. With this degree of involvement,
amputation of the second ray with excision of the overgrowth of affected soft tissue provides the most
consistent desired reduction in foot size. A threaded Steinmann pin should be inserted across the
remaining metatarsals until healing has occurred. Patients with macrodactyly should be examined to
exclude neurofibromatosis type 1 and Klippel-Trenaunay-Weber syndrome.

10.Figures 5a and 5b show the radiographs of an 11-year-old boy who felt a pop and immediate pain in
his right knee as he was driving off his right leg to jam a basketball. Examination reveals that the knee
is flexed, and the patient is unable to actively extend it or bear weight on that side. There is also a large
effusion. Management should include

1- ice and elevation, followed by graduated range-of-motion exercises.


2- a long leg cast.
3- excision of the fragment.
4- open reduction and internal fixation.
5- observation until maturity, followed by anterior cruciate ligament repair.

DISCUSSION: Fractures through the cartilage on the inferior pole of the patella, the so-called sleeve
fracture, are often difficult to diagnose because of the paucity of ossified bone visible on the
radiographs. If the fracture is missed and the fragments are widely displaced, the patella may heal in an
elongated configuration that may result in compromise of the extensor mechanism function. The
treatment of choice is open reduction and internal fixation using a tension band wire technique to
achieve close approximation of the fragments and restore full active knee extension.
PREFERRED RESPONSE: 4

11.A 2-year-old child has been referred for management of congenital kyphosis. Neurologic
examination is normal, and radiographs show a type I congenital kyphosis. Which of the following
anomalies is seen in the MRI scan shown in Figure 6?

1- Chiari II malformation
2- Syrinx
3- Diastematomyelia
4- Meningocele
5- Tethered cord

94 5
American Academy of Orthopaedic Surgeons 6
2001 Pediatric Orthopaedic Self-Assessment Examination.by Dr.Dhahirortho
PREFERRED RESPONSE: 5
DISCUSSION: There is a high incidence of intraspinal anomalies in patients with congenital scoliosis
and kyphosis. Bradford and associates reported an incidence rate of 38% in 42 patients. The MRI scan
shows that the filum terminale is thickened and adherent distally in the spinal canal. Although the
conus is at L1, which may be normal, neurologic dysfunction may occur with further growth. There
are no signals of high intensity within the cord that would suggest a syrinx. A Chiari II malformation
would be found in the upper cervical region, not shown in this MRI scan. Meningocele and
diastematomyelia are not present.

12.A 15-year-old boy with a type I hereditary sensory motor neuropathy (Charcot-Marie-Tooth
disease) reports recurrent ankle sprains and significant pain in the hindfoot and midfoot despite orthotic
management. Examination reveals that he walks with a drop foot and has dynamic clawing of the toes.
Clinical photographs of the left foot are shown in Figure 7. Management should consist of

1-revision of the current orthotics.


2-metatarsal osteotomies to correct cavus.
3-a sliding calcaneal osteotomy to correct hindfoot varus.
4-triple arthrodesis, with anterior transfer of the posterior tibialis.
5-extensor transfer to the metatarsal necks, soft-tissue releases, and
anterior transfer of the posterior tibialis tendon.

DISCUSSION: The clinical photographs show a patient with a type


I hereditary sensory motor neuropathy who has cavus feet with a
flexible hindfoot. The Coleman block test shows that the hindfoot
corrects into valgus. To prevent progressive cavus, patients with
this condition may benefit from soft-tissue releases at a younger
age while the foot is flexible. Once there is fixed deformity,
combined soft-tissue and bone procedures usually are necessary.
Metatarsal osteotomies will correct the cavus, but will do nothing
for the drop foot. Transfer of the extensor hallucis longus to the
neck of the first metatarsal and modified transfer of the extensor
digitorum longus to the dorsum of the foot will prevent further claw toes and improve foot dorsiflexion.
Anterior transfer of the posterior tibialis tendon will also aid in dorsiflexion. Calcaneal osteotomy
should be reserved for fixed hindfoot varus that does not correct with block testing, and triple
arthrodesis should be avoided as long as possible because the long-term outcome is poor.
PREFERRED RESPONSE: 5

13.Figures 8a through 8d show the radiographs and CT scans of a 14-year-old girl who has a painful,
rigid planovalgus foot. Management consisting of arch supports and anti-inflammatory drugs failed to
provide relief. A below-knee walking cast resulted in pain resolution, but she now reports that the pain
has recurred. Management should now consist of

95 6
American Academy of Orthopaedic Surgeons 7
2001 Pediatric Orthopaedic Self-Assessment Examination.by Dr.Dhahirortho
1- custom orthotics.
2- a below-knee non-weight-bearing cast.
3- exploration and resection of the calcaneocuboid joint.
4- exploration and resection of the talocalcaneal joint.
5- triple arthrodesis.

DISCUSSION: Tarsal coalitions commonly present in the preadolescent age group as a rigid,
planovalgus foot. Small coalitions of the calcaneonavicular joint or the middle facet of the
talocalcaneal joint can be excised with interposition of fat or muscle tissue. Isolated calcaneocuboid
joint coalitions are very rare. This patient has an associated large talocalcaneal coalition; therefore,
resection is contraindicated. Surgery is warranted after failure of nonsurgical management, and
because of the involvement of two joints, the only viable option for the severely symptomatic foot is
triple arthrodesis. PREFERRED RESPONSE: 5

14.Figures 9a and 9b show the spinal radiographs of a 3-year-old child with short limb dwarfism. The
lateral radiograph is obtained with maximal lumbar extension. Management should consist of

1- close clinical follow-up.


2- a thoracolumbosacral orthosis.
3- anterior spinal fusion.
4- posterior spinal fusion.
5- fibroblast growth factor replacement.

DISCUSSION: The patient has kyphosis in association with


achondroplasia. The AP radiograph shows decreased
interpedicular distance at the lower lumbar vertebrae, a feature
considered to be a distinctive sign of achondroplasia. Most
patients with achondroplasia have kyphosis, and this usually
resolves spontaneously. When the fixed component is greater
than 30°, however, brace treatment is recommended. Spinal fusion is seldom required.
PREFERRED RESPONSE: 2

15.A 10-year-old boy with severe hemophilia A (factor VIII) sustained an injury to his right forearm 2
hours ago when a classmate fell on his arm during a scuffle. Examination reveals moderate swelling in
the forearm, decreased sensation in the distribution of the median and ulnar nerves, and pain on passive
extension of the fingers. What is the most appropriate sequence of treatment?

1-Measurement of volar compartment pressures and, if elevated, a bolus transfusion of 4 unit/kg of


factor VIII concentrate, followed by continuous transfusion of factor VIII
2- Splinting of the extremity with the elbow flexed and the wrist in extension, elevation of the
extremity, observation, and if no improvement, continuous transfusion of factor VIII (4 unit/kg)
3-A bolus transfusion of 4 unit/kg of factor VIII concentrate, followed by continuous transfusion of
factor VIII, measurement of forearm compartment pressures, splinting of the extremity with the elbow
and wrist flexed, and surgical release of the volar forearm compartments if pressures are elevated
4-A bolus transfusion of 4 unit/kg of factor VIII concentrate, followed by continuous transfusion of
factor VIII and an emergency fasciotomy
5-A bolus transfusion of 4 unit/kg of factor VIII concentrate, laboratory studies consisting of serum
electrolytes and creatine phosphokinase levels, IV injections of bicarbonate to alkalize the urine, and
renal dialysis
96 7
American Academy of Orthopaedic Surgeons 8
2001 Pediatric Orthopaedic Self-Assessment Examination.by Dr.Dhahirortho

PREFERRED RESPONSE: 3
DISCUSSION: The patient has severe hemophilia with a volar forearm hemorrhage and an emerging
compartment syndrome. Therefore, it is critical to normalize the clotting deficiency as the first step in
treatment. In a patient who has a factor VIII level of less than 1% and no inhibitors to factor VIII,
transfusion with 4 unit/kg will typically raise the factor VIII level to 100%. Continuous transfusion can
then be used to maintain this level. Compartment pressures can be safely measured after infusion of
factor VIII. Because the hemorrhage is of limited duration and any surgery is considered serious in a
patient with hemophilia, the compartment pressure should be measured before making a decision
regarding a fasciotomy. However, it is important to note that the use of factor VIII concentrates allows
both emergency and elective surgery provided that adequate hematology backup is available. Splinting
the elbow and wrist in flexion reduces the pressure in the volar compartments, protects the forearm
from further trauma, and makes the patient more comfortable.

16. Figures 10a and 10b show the radiographs of an athletic 9-year-old boy who has activity-related
anterior knee pain with intact active knee extension. Examination reveals tenderness to palpation over
the inferior pole of the patella. There is no effusion or ligamentous instability. Initial management
should consist of

1- long leg cast immobilization for 6 weeks.


2- open reduction and internal fixation.
3- activity restrictions and nonsteroidal anti-inflammatory drugs.
4- cessation of sports for 6 to 18 months.
5- diagnostic arthroscopy.

DISCUSSION: The radiographs show fragmentation of the inferior pole of the patella. This finding,
along with the clinical presentation, is most consistent with Sindig-Larsen-Johansson disease. This is
an overuse syndrome commonly seen in boys ages 9 to 11 years. The differential diagnosis includes
bipartite patella and patellar sleeve fracture. Like most overuse syndromes, Sindig-Larsen-Johansson
disease responds to activity modification and nonsteroidal anti-inflammatory drugs. While symptoms
usually resolve with short periods of activity restriction, radiographic findings may persist.

PREFERRED RESPONSE: 3

97 8
American Academy of Orthopaedic Surgeons 9
2001 Pediatric Orthopaedic Self-Assessment Examination.by Dr.Dhahirortho
17. A 2-week-old infant has been referred for evaluation of nonmovement of the left hip. History
reveals that the patient was delivered 6 weeks premature by cesarean section. Examination reveals no
fever, and there is mild swelling of the thigh. Passive movement of the hip appears to elicit tenderness
and very limited hip motion. A radiograph of the pelvis shows mild subluxation of the left hip. The
next step in evaluation should consist of

1- aspiration of the left hip.


2- application of a Pavlik harness.
3- a gallium scan.
4- an MRI scan of the spine.
5- modified Bryant traction.

DISCUSSION: The diagnosis of bone and joint sepsis in a newborn is difficult because of the relative
lack of obvious signs and symptoms. Fever is usually absent. A study of 34 newborns with
osteomyelitis identified prematurity and delivery by cesarean section as predisposing factors. In that
study, the most common clinical findings were pseudoparalysis, local swelling, and pain on passive
movement. Because early diagnosis is so important, any infant who exhibits these findings should be
suspected as having bone or joint sepsis. Once the area of involvement is identified, aspiration is
mandatory. In newborns who have an infection about the hip, radiographs may reveal subluxation. In
this patient, septic arthritis must be ruled out by aspiration of the hip. Developmental dysplasia of the
hip is not painful and is not accompanied by localized swelling. If no purulent material is obtained at
the time of hip aspiration, an arthrogram should be obtained to rule out epiphysiolysis of the proximal
femur. Because the area of involvement has been identified by clinical examination, a gallium scan or
MRI scan of the spine is not indicated. PREFERRED RESPONSE: 1

18. Figure 11 shows the lateral radiograph of a 16-year-old boy who has been unable to participate
in sports activities because of pain in the anterior aspect of the knee. He states that the pain is aching in
nature and is located in the region of the tibial tuberosity. He denies having joint effusion or symptoms
of instability. Management should consist of
1-phonophoresis treatment and a program of quadriceps stretching.
2-application of a hinge, post, and shell design functional knee brace.
3-application of a hinge, post, and strap design functional knee brace.
4-an MRI scan of the knee, a CT scan of the chest, a technetium Tc 99m
bone scan, liver function tests, and a biopsy of the proximal tibia.
5-excision of the ossicle and prominence of the tibial tuberosity.

DISCUSSION: The prognosis for most patients with Osgood-Schlatter


disease is good. When the secondary ossification center unites with the
main body of the tibial tubercle, the patellar tendon has a more rigid
anchor, and heterotopic ossification and its associated reaction often
become quiescent. However, even after closure of the growth plates,
some patients have persistent symptoms. Excision of the ossicle and
prominence of the tibial tuberosity decompresses the patellar tendon and allows most patients to
resume sports activities. Nonsurgical modalities are ineffective. Better results have been reported after
excision than after drilling of the tubercle. Excision of the ossicle is not indicated prior to skeletal
maturity because symptoms will resolve in most patients when the secondary ossification center unites.
PREFERRED RESPONSE: 5
98 9
American Academy of Orthopaedic Surgeons 10
2001 Pediatric Orthopaedic Self-Assessment Examination.by Dr.Dhahirortho
19. A 10-lb, 2-oz infant who was born via a difficult breech delivery 12 hours ago is now being
evaluated for hip pain. Although the infant is resting comfortably, examination reveals that the patient
is not moving the right lower extremity and manipulation of the right hip causes the infant to cry. The
Galeazzi sign is positive. An AP radiograph of the pelvis shows proximal and superior migration of the
right proximal femoral metaphysis. What is the most likely diagnosis?

1- Teratologic hip dislocation


2- Septic right hip
3- Congenital coxa vara
4- Transphyseal fracture of the proximal femur
5- Postpartum ligamentous laxity

DISCUSSION: Transphyseal fractures of the proximal femur at birth are more likely to occur in large
newborns after a difficult delivery. At rest, the patients are comfortable and show a pseudoparalysis;
however, passive motion of the lower extremity results in discomfort. Teratologic hip dislocations will
have a positive Galeazzi sign, but are not painful. Development of a septic hip would be unlikely
within 12 hours postpartum. Congenital coxa vara is typically painless. Postpartum ligamentous laxity
might account for a positive Ortolani sign, but is painless. PREFERRED RESPONSE: 4

20. A 10-year-old girl has been referred for evaluation of a prominence at the lower cervical spine.
The patient is asymptomatic, and the examination reveals no evidence of neurologic abnormality. A
radiograph and CT scans are shown in Figures 12a through 12c. What is the most likely diagnosis?

1- Tuberculosis
2- Congenital kyphosis
3- Blastomycosis
4- Aneurysmal bone cyst
5- Osteoblastoma
DISCUSSION: Tuberculosis is uncommon in the cervical spine but has a relatively greater incidence in
young children. In a review of 40 patients with lower cervical spine involvement (C2 to C7), 24 were
younger than age 10 years at presentation. In children, the disease is characterized by more extensive
involvement with the formation of large abscesses. In older patients with lower cervical tuberculosis,
the disease is more localized but is more likely to cause paraplegia. Four-drug antituberculosis therapy
should be used. For patients with pain or neurologic dysfunction, anterior excision of diseased bone
and grafting are indicated. Whether vertebral body excision and grafting should be done in an
asymptomatic 10-year-old child is debatable. The CT scan shows a large “cold” abscess that is
partially calcified. PREFERRED RESPONSE: 1
99 10
American Academy of Orthopaedic Surgeons 11
2001 Pediatric Orthopaedic Self-Assessment Examination.by Dr.Dhahirortho
21. Which of the following types of iliac osteotomy provides the greatest potential for increased
coverage?

1- Ganz periacetabular
2- Pemberton innominate
3- Salter innominate
4- Sutherland double innominate
5- Steele triple innominate

DISCUSSION: The degree of acetabular dysplasia and the age of the child are important considerations
when choosing what type of osteotomy to perform. The ability to obtain concentric reduction is a
prerequisite of all osteotomies that redirect the acetabulum. Procedures that cut all three pelvic bones
allow more displacement and, therefore, correction of acetabular dysplasia. The closer the osteotomy
is to the acetabulum, the greater the coverage of the femoral head. Compared with the other acetabular
osteotomies, the Ganz periacetabular osteotomy provides the greatest potential for correcting acetabular
deficiency because there are no bone or ligamentous restraints to limit correction, but it has the
disadvantage of being a technically demanding procedure. The amount of coverage provided by the
Salter osteotomy is limited.. PREFERRED RESPONSE: 1

22. The mother of a 26-month-old boy reports that he has been unwilling to bear weight on his left
lower extremity since he awoke this morning. She denies any history of trauma. He has a temperature
of 99.4°F (37.4°C), and examination reveals that abduction of the left hip is limited to 30°. Laboratory
studies show a WBC of 11,000/mm3 and an erythrocyte sedimentation rate of 22 mm/h. A radiograph
of the pelvis is shown in Figure 13. Management should consist of
1-aspiration of the hip.
2-a bone scan.
3-an MRI scan.
4-bed rest, frequent temperature checks, and reevaluation in
24 to 48 hours.
5-hospital admission and placement in traction.

DISCUSSION: The most likely diagnosis is transient


synovitis. Initial management should consist of bed rest
and serial observation to rule out atypical septic arthritis of the hip. In an unreliable family situation,
hospitalization for bed rest and observation may be indicated. Other disorders such as proximal
femoral osteomyelitis, leukemia, juvenile rheumatoid arthritis, pelvic osteomyelitis, diskitis, and
arthralgia secondary to other inflammatory disorders should be considered. However, these disorders
are unlikely because of the paucity of abnormal clinical signs exhibited by the patient. On the other
hand, transient synovitis of the hip in children is a diagnosis of exclusion; other possibilities should be
explored if the patient’s symptoms do not follow a typical course and resolve in 4 to 21 days.
PREFERRED RESPONSE: 4

23. A 10-year-old girl with a monoarticular pattern of juvenile rheumatoid arthritis (JRA) has had a
3-cm limb-length discrepancy since age 8 years when inflammation in the right knee came under good
medical control. Because her right leg is longer, the patient states that she would like her legs to be
close to equal in length in the future. A growth-remaining chart is shown in Figure 14. Management
should consist of

100 11
American Academy of Orthopaedic Surgeons 12
2001 Pediatric Orthopaedic Self-Assessment Examination.by Dr.Dhahirortho
1-immediate proximal right tibial physeal arrest.
2-immediate distal right femoral physeal arrest.
3-a shoe lift and follow-up in 2 years.
4-observation with the expectation that the discrepancy
may correct itself and not require surgery.
5-limb lengthening of the shorter left limb.

DISCUSSION: In a subgroup of patients with


monoarticular JRA and a limb-length discrepancy that
developed before the age of 9 years, Simon and
associates showed that a subsequent growth deceleration
on the affected side may correct a large part of the
difference in length. This possibility would make
surgery unnecessary and should prompt further
observation.
PREFERRED RESPONSE: 4

24. A 14-year-old girl with polyarticular juvenile rheumatoid arthritis (JRA) has severe neck pain and
reports the onset of urinary incontinence. A lateral radiograph and lateral tomogram of the cervical
spine are shown in Figures 15a and 15b. An MRI scan of the upper cervical spine is shown in Figure
15c. Management should consist of

1- a rigid cervical orthosis.


2- a soft cervical collar.
3- posterior C1-2 fusion with halo immobilization.
4- administration of methotrexate.
5- activity restrictions.

DISCUSSION: The plain radiograph and tomogram show an abnormality of the upper cervical spine,
with erosion of the dens. The MRI scan shows evidence of cord impingement. The cervical spine is
frequently involved in polyarticular JRA. Stiffness and autofusion are commonly seen, but C1-2
instability can also occur secondary to synovitis and bony erosion. Basilar invagination is rare in JRA.
There is no consensus regarding fusion in the asymptomatic patient. In patients with symptoms and
neurologic signs, C1-2 posterior fusion is indicated.

PREFERRED RESPONSE: 3
101 12
American Academy of Orthopaedic Surgeons 13
2001 Pediatric Orthopaedic Self-Assessment Examination.by Dr.Dhahirortho
25. Figures 16a and 16b show the radiographs of an otherwise healthy 3 1/2-year-old boy who has
an isolated deformity of the left leg. Definitive primary treatment of this condition should consist of

1- distraction osteogenesis.
2- an intramedullary nail and onlay bone graft.
3- a short leg cast.
4- a vascularized free fibular graft.
5- plate fixation and an autogenous bone graft.

DISCUSSION: Treatment of congenital pseudarthrosis of the tibia


is problematic. To achieve union, a resection of the pseudarthrosis,
stabilization, and bone grafting must be performed. Simple cast
immobilization does not yield union. There are various options for
the resection, immobilization, and grafting. On the first surgical
attempt, retrograde intramedullary nailing offers the best chance
for success by transfixing the ankle and subtalar joints with
abundant autogenous bone grafting. Distraction osteogenesis and
vascularized free fibular graft are reserved as salvage procedures. PREFERRED RESPONSE: 2

26. Figure 17 shows the radiograph of a 2-year-old girl who sustained a fracture of the femur in a
fall while walking with her parents. History reveals that this is her third long bone fracture, having
sustained a humerus fracture 1 year ago and a fracture of the opposite femur 9 months ago. There is no
family history of any similar problem. Examination reveals distinctly blue sclerae, normal appearing
teeth, and no skin lesions. What is the most likely cause of this patient’s
disorder?

1- A quantitative defect of type I collagen synthesis


2- A deficiency of vitamin D
3- Parental abuse
4- Abnormal osteoclast function
5- Excess excretion of keratin sulfate in the urine

DISCUSSION: Osteogenesis imperfecta (OI) is a genetically determined


disorder of type I collagen synthesis that is characterized by bone fragility.
This patient has had three fractures of the long bones by age 2 years, with
the last one occurring after relatively minor trauma. The patient’s history
and clinical features are consistent with a diagnosis of Sillence type IA OI.
Type I OI is the mildest and most common form. Inheritance is
autosomal-dominant; however, as in this patient, new mutations are
frequent. Type I is subclassified into the A type (absence of
dentinogenesis imperfecta) and B type (presence of dentinogenesis imperfecta). The sclerae are blue,
and the first fractures usually occur in the preschool years after walking has begun. Cells from
individuals with type I OI largely demonstrate a quantitative defect of type I collagen; they synthesize
and secrete about half the normal amount of type I procollagen. Qualitative mutations that lead to an
abnormal type I procollagen molecule result in more severe types of the disorder. There are no
indications that this child has been abused. Radiographs of the femur show no evidence of rickets,
pyknodysostosis, or osteopetrosis. Morquio syndrome, characterized by excess excretion of keratin
sulfate in the urine, is not associated with bone fragility. PREFERRED RESPONSE: 1
102 13
American Academy of Orthopaedic Surgeons 14
2001 Pediatric Orthopaedic Self-Assessment Examination.by Dr.Dhahirortho
27. An 8-year-old boy with severe hemophilia A (factor VIII) and no inhibitor is averaging eight
transfusions per month for bleeding into the right ankle. Examination shows synovial hypertrophy;
range of motion consists of 0° of dorsiflexion and 20° of plantar flexion. The patient’s knees, elbows,
and left ankle have no restriction of motion. Standing radiographs of the right ankle are shown in
Figure 18. Management should consist of

1-prophylactic transfusions three times per week.


2-application of ankle-foot orthoses.
3-ankle synovectomy.
4-ankle arthrodesis performed with physeal protection.
5-pantalar arthrodesis.

DISCUSSION: The patient has bilateral hypertrophic


synovitis that is causing repeated hemarthroses and
progressive arthropathy. Ankle synovectomy in
patients with hemophilia is effective in significantly
reducing the rate of joint bleeding and in slowing the
progression of the arthropathy; therefore, bilateral
synovectomies is the treatment of choice. Range of motion can be effectively maintained after ankle
synovectomy. Bracing and prophylactic transfusions would be ineffective at this time. Ankle
arthrodesis should be reserved for patients with severe pain. Compared with patients who have
juvenile rheumatoid arthritis, patients with hemophilia generally do not have involvement of the
subtalar joint and rarely require a pantalar arthrodesis. PREFERRED RESPONSE: 3

28. Figures 19a through 19c show radiographs of the cervical spine of an asymptomatic patient with
Down syndrome who wants to participate in a Special Olympics running event. The neurologic
examination is normal. Management should consist of

1- an MRI scan.
2- fusion from the occiput to C2.
3- fusion of C1-C2.
4- application of a cervical collar and no participation in any sports.
5- periodic follow-up examinations and no contact sports.

DISCUSSION: An atlanto-dens interval (ADI) of up to 4 mm in children is considered normal.


Children with Down syndrome have increased ligamentous laxity, with atlantoaxial instability
occurring in as many as 15% to 20% of patients. These patients are at risk for catastrophic injury
103 14
American Academy of Orthopaedic Surgeons 15
2001 Pediatric Orthopaedic Self-Assessment Examination.by Dr.Dhahirortho
following minor trauma and should be routinely screened for instability, generally beginning when the
patient starts to walk. Patients with an ADI of greater than 5 mm should avoid contact sports and high-
risk activities such as gymnastics, diving, the high jump, and the butterfly stroke. The American
Academy of Pediatrics Committee of Sports Medicine and Fitness guidelines recommend that lateral
views of the cervical spine in neutral, flexion, and extension should be obtained in all children with
Down syndrome who wish to participate in sports. Patients with normal radiographs and examinations
do not need repeat radiographs, although some authors suggest that instability increases with age, and
therefore recommend repeat radiographs every 5 years. Cervical spine fusion in patients with Down
syndrome has a high rate of complications and should be performed only on patients with symptoms
and evidence of myelopathy.

PREFERRED RESPONSE: 5

29. Compared with amputation, limb salvage for osteosarcoma of the distal end of the femur will
result in

1- an improved chance for long-term survival.


2- an improved quality of life.
3- a lessened need for chemotherapy.
4- a better functional outcome.
5- a reduced need for reoperation.

DISCUSSION: Major advances in diagnostic imaging, neoadjuvant chemotherapy, and surgical


techniques have allowed limb salvage to be performed as an alternative to amputation in children with
osteosarcoma. The outcome of treatment of nonmetastatic, high-grade osteosarcoma of the distal
femur was studied in 227 patients from 26 institutions. The authors found no difference in the long-
term survival or quality of life between patients treated with limb salvage and those treated with
amputation. Patients treated with limb salvage had a higher rate of reoperation, but a better functional
outcome.

PREFERRED RESPONSE: 4

30. Examination of a 7-year-old boy reveals 20° of valgus following a lawn mower injury to the
lateral femoral epiphysis. Treatment consists of total distal femoral epiphyseodesis and varus
osteotomy. Following surgery, he has a limb-length discrepancy of 3 cm and 5° of genu valgum.
Assuming that he undergoes no further treatment, the patient’s predicted limb-length discrepancy at
maturity would be how many centimeters?

1- Less than 7
2- 7 to 10
3- 11 to 13
4- 14 to 17
5- Greater than 17

DISCUSSION: The distal femoral epiphysis grows approximately 1 cm per year and in boys, growth
ceases at approximately age 16 years. Therefore, the patient’s limb-length discrepancy at maturity
would be 12 cm (9 cm plus the 3-cm discrepancy he has from the previous surgery).

PREFERRED RESPONSE: 3
104 15
American Academy of Orthopaedic Surgeons 16
2001 Pediatric Orthopaedic Self-Assessment Examination.by Dr.Dhahirortho
31. When the iliac apophysis starts ossifying in the normal adolescent, growth of the sitting height
or trunk height is characterized by

1- no appreciable change during the remainder of growth for either boys or girls.
2- an increase of 3 to 5 cm in girls and an increase of 5 to 8 cm in boys.
3- an increase of 5 to 8 cm in girls and an increase of 3 to 5 cm in boys.
4- an increase of 8 to 11 cm in girls and an increase of 11 to 13 cm in boys.
5- an increase of 11 to 13 cm in girls and an increase of 8 to 11 cm in boys.

DISCUSSION: Studies by Anderson and associates have resulted in a growth-remaining chart for
sitting height that shows an increase of 3 to 5 cm in girls and an increase of 5 to 8 cm in boys. Future
growth of the spine may impact brace longevity and fit. PREFERRED RESPONSE: 2

32. A 10-year-old girl was thrown over the handlebars of her bicycle and landed directly on her left
shoulder. She was treated with a figure-of-8 strap and analgesics. Follow-up examination 2 weeks
later reveals that the lateral end of the clavicle is superiorly dislocated relative to the acromion. A
radiograph of the shoulder shows calcification lateral to the coracoid process at the level of the
acromion, and the clavicle is superiorly displaced. Management should consist of

1- supportive immobilization until the patient is pain-free.


2- coracoclavicular ligament repair.
3- open reduction and internal fixation of the acromioclavicular dislocation.
4- resection of the distal clavicle and Weaver and Dunn reconstruction.
5- open biopsy of the calcific mass.

DISCUSSION: In adults, a direct blow on the acromion usually results in an acromioclavicular


dislocation. In children, however, the usual injury from this mechanism is a physeal fracture of the
lateral clavicle. The clavicular shaft fragment, analogous to the metaphyseal portion of a physeal
fracture, herniates through the periosteum, leaving the distal periosteal sleeve in contact with the lateral
(distal) physeal fragment. The treatment of choice is immobilization until the patient is pain-free.
PREFERRED RESPONSE: 1

33. Figures 20a and 20b show the radiographs of an obese 15-year-old boy who has severe left
groin pain and is unable to bear weight following a minor injury. Treatment should consist of

1- fixation with one or two screws.


2- cast immobilization.
3- manipulative reduction with single screw fixation.
4- in situ fixation with multiple screws.
5- open epiphyseodesis.
105 16
American Academy of Orthopaedic Surgeons 17
2001 Pediatric Orthopaedic Self-Assessment Examination.by Dr.Dhahirortho
PREFERRED RESPONSE: 1
DISCUSSION: The radiographs and history are consistent with an acute unstable slipped capital
femoral epiphysis. Aronson and Loder documented an increased rate of osteonecrosis associated with
manipulative reduction. They recommended bed rest with skin traction to allow the synovitis to
resolve, followed by in situ pinning. They noted, however, that many of these slips reduced with
anesthesia and positioning on a fracture table. Biomechanic studies have shown a slight increased
resistance to shear stress when two screws are used, but it is unknown if this is significant in the
clinical setting. Open epiphyseodesis does not provide postoperative stability; therefore, adjunctive
fixation or immobilization is required. Numerous studies have noted the inadvisability of using
multiple screws. Casting has a high rate of complications, including chondrolysis and progression of
the slip.

34. What is the recommended treatment of a skeletally immature 12-year-old boy who has an
anterior cruciate ligament-deficient knee?
1- Reduced activity, rehabilitation exercises, and functional bracing until the patient is near
skeletal maturity
2- Bone-patellar tendon-bone autograft reconstruction
3- Allograft reconstruction
4- Hamstring tendon intra-articular repair using a centrally placed tibial tunnel and an over-
the-top femoral attachment
5- Extra-articular repair

DISCUSSION: Traditional surgeries for anterior cruciate ligament-deficient knees carry the potential
risk of premature physeal closure in young athletes. Therefore, most surgeons are reluctant to
recommend intra-articular reconstruction using bone tunnels with bone-patellar tendon-bone autografts
or hamstring tendons. The current recommendation for young athletes is activity modification,
rehabilitation, and functional bracing until the patient is near skeletal maturity. At that time, for the
very symptomatic patient, the treatment of choice is intra-articular repair of the anterior cruciate
ligament. If a skeletally immature patient continues to have instability despite rehabilitation and
bracing, a modification of the femoral tunnel to the over-the-top position will not place the lateral
femoral physis at risk for premature closure and deformity. A centrally placed tibial tunnel will
minimize the risk of angular deformity and minimize limb-length discrepancy if physeal arrest occurs.
PREFERRED RESPONSE: 1

35. Figures 21a and 21b show the radiographs of a 12-year-old patient with an L4-level
myelomeningocele who has scoliosis that has been slowly progressing for the past several
years. There has been no loss of motor function. An MRI scan shows no syringomyelia or
increased hydrocephalus. Management should consist of

106 17
American Academy of Orthopaedic Surgeons 18
2001 Pediatric Orthopaedic Self-Assessment Examination.by Dr.Dhahirortho
1- follow-up with repeat radiographs in 6 months.
2- brace treatment.
3- posterior spinal fusion with instrumentation.
4- anterior and posterior spinal fusion with instrumentation.
5- anterior spinal fusion with instrumentation.

DISCUSSION: Scoliosis is a common occurrence in children with myelomeningocele, with the


incidence increasing as the neurologic level moves cephalad. The rate of pseudarthrosis for isolated
anterior or posterior fusions has been reported as high as 75%. The combination of anterior and
posterior fusions with some type of instrumentation has been shown to decrease the rate of
pseudarthrosis to 20%. Brace treatment in smaller curves can be used as a temporizing measure to
delay surgery, but as with idiopathic scoliosis, the brace is ineffective for larger curves. Observation is
not indicated with a curve of this magnitude. PREFERRED RESPONSE: 4

36. A 3-year-old child is referred for evaluation of bowed legs. History reveals no dietary
deficiencies; however, family history is significant for several members with bowed legs. Examination
reveals genu varum, and the child is in the 5th percentile for height and weight. Laboratory studies
show normal renal function, a normal calcium level, a decreased phosphate level, and an elevated
alkaline phosphatase level. A plain radiograph of the lower extremities is shown in Figure 22. What is
the most likely diagnosis?
1- Blount’s disease
2- Chondrometaphyseal dysplasia
3- Renal osteodystrophy
4- Vitamin D-deficient rickets
5- Vitamin D-resistant rickets

DISCUSSION: The differential diagnosis of genu varum includes


physiologic genu varum, Blount's disease, skeletal dysplasia, and
metabolic bone disease. Children with Blount's disease are generally in
the 95th percentile for height and weight, and usually multiple family
members are not affected. The radiographs show widening of the physis
and metaphyseal flaring. In Blount's disease, the characteristic
radiographic changes involve only the tibia, and at this age, most
commonly show beaking of the medial metaphysis. Skeletal dysplasias,
such as chondrometaphyseal dysplasia, are associated with short stature,
and the radiographic changes are similar to those seen here. However,
laboratory studies in these children will be within normal limits. Children
with chronic renal disease will often be of short stature, and the
radiographic findings are again similar to those shown here. However, BUN and creatinine levels are
elevated and phosphate levels are elevated rather than decreased in children with renal disease. The
absence of dietary deficiencies and positive family history rules out vitamin D-deficient rickets. There
are four types of vitamin D-resistant rickets: failure of production of 1,25-dihydroxy vitamin D,
phosphate diabetes (hypophosphatemic rickets), end organ insensitivity to vitamin D, and renal tubular
acidosis. All types of vitamin D-resistant rickets are resistant to treatment with physiologic doses of
vitamin D. The patient’s clinical picture, family history, laboratory studies, and radiographs are most
consistent with hypophosphatemic rickets. This entity is inherited as a sex-linked dominant trait.

PREFERRED RESPONSE: 5
107 18
American Academy of Orthopaedic Surgeons 19
2001 Pediatric Orthopaedic Self-Assessment Examination.by Dr.Dhahirortho
37. A 14-year-old boy sustained a femoral neck fracture in a fall from a tree and underwent open
reduction and internal fixation 6 months ago. Follow-up examination now reveals an antalgic
Trendelenburg gait and painful range of motion. A radiograph is shown in Figure 23, and a CT scan
shows a nonunion. Treatment should consist of

1-revision in situ pinning and application of a cast.


2-hip fusion.
3-curettage of the nonunion, intertrochanteric valgus
osteotomy, and revision internal fixation.
4-non-weight-bearing on the affected side and electrical
stimulation of the fracture site.
5-vascularized fibula grafting and application of a cast.

DISCUSSION: The coxa vara deformity and fracture


nonunion should be treated simultaneously; therefore, the
treatment of choice is curettage of the nonunion,
intertrochanteric valgus osteotomy, and revision internal
fixation. In addition, valgus osteotomy will convert the
shear forces across the nonunion to compression, aiding
in healing of the nonunion. None of the other procedures addresses both issues, and hip fusion is
inappropriate under these conditions.

PREFERRED RESPONSE: 3

38. A 22-month-old child has scrapes and bruises on his head and a severe deformity of the forearm
after being thrown from a car as an unrestrained passenger in a motor vehicle accident. Examination
reveals a Glasgow Coma Scale score of 12. Prior to treatment of the forearm, management should
include

1- a mannitol infusion of 0.25 to 1 g/kg.


2- high-dose IV methylprednisolone, consisting of a 30 mg/kg bolus, followed by 5.4
mg/kg/h for 23 hours.
3- an immediate CT scan.
4- an electroencephalogram.
5- radiographs of the skull.

DISCUSSION: As CT scanning has become available, the use of radiographs of the skull has decreased
in importance for evaluation of head trauma. The indications for CT scanning for suspected head
trauma include any degree of obtundation, focal neurologic deficit, history of a high-velocity injury,
amnesia for the injury, progressive headache, persistent vomiting, children younger than age 2 years,
serious facial injury, posttraumatic seizure, skull penetration, or a Glasgow Coma Scale score of 13 or
less. Evidence of improved outcome with use of steroids in head trauma is lacking. Steroids are useful
for increased intracranial pressure caused by brain tumors or abscesses. High-dose IV
methylprednisolone is indicated for spinal cord trauma and improves the ultimate degree of recovery of
function. When herniation is suspected in a patient with asymmetric neurologic findings or the
patient’s condition is deteriorating rapidly, a mannitol infusion may be used.

PREFERRED RESPONSE: 3
108 19
American Academy of Orthopaedic Surgeons 20
2001 Pediatric Orthopaedic Self-Assessment Examination.by Dr.Dhahirortho
39. Examination of a 5-year-old boy with amyoplasia shows a flexion contracture of 70° of the
right knee. The active arc of motion is from 70° to 90°, and the opposite knee has a flexion contracture
of 10°. Both hips are dislocated with flexion contractures of 10°, passive hip motion is from 10° to 90°
of flexion, and the feet are plantigrade and easily braceable. Despite a daily stretching program, the
parents and physical therapists note that it is increasingly difficult for him to walk because of the
flexion contracture of the right knee. Management of the knee flexion contracture should now include

1- intense physical therapy.


2- an intramuscular injection of botulinum toxin A.
3- radical posterior soft-tissue release.
4- supracondylar femoral extension osteotomy.
5- gradual correction with a circular ring external fixator.

DISCUSSION: Most children with amyoplasia are ambulatory and when a decrease in function occurs
because of a severe contracture, it must be addressed. A radical posterior soft-tissue release, including
the posterior knee capsule and often the collateral ligaments and the posterior cruciate ligament, is
needed to obtain extension. After the age of 1 year, aggressive physical therapy will do little to correct
a contracture. Botulinum toxin A is indicated for spasticity and is contraindicated with severe
contractures. Supracondylar femoral extension osteotomy works well, but will remodel at an average
rate of 1° per month, which is not considered ideal in a young patient. Gradual correction with a
circular ring external fixator is an option, but a soft-tissue release will also most likely be needed for a
contracture of this severity.

40. A 13-year-old girl who is 2 years postmenarche has been referred for management of scoliosis.
She denies any history of back pain. Radiographs show a right thoracic curve of 35°. She has a Risser
sign of 4 and a bone age of 15.5 years. Management should consist of

1- a low-profile spinal orthosis.


2- observation and follow-up radiographs in 6 months.
3- anterior spinal fusion with instrumentation.
4- posterior spinal fusion with instrumentation.
5- in situ posterior spinal fusion.

DISCUSSION: Because the patient is skeletally mature with a curve of less than 40°, there is no
benefit to bracing and surgery is not indicated. Management should consist of observation and follow-
up radiographs in 6 months. PREFERRED RESPONSE: 2

41. In children with isolated zone II lacerations of the flexor tendon, poor digital motion is best
correlated with

1- a patient age of less than 5 years.


2- a patient age of greater than 15 years.
3- early passive motion.
4- immobilization for 3 weeks.
5- immobilization for 6 weeks.

109 20
American Academy of Orthopaedic Surgeons 21
2001 Pediatric Orthopaedic Self-Assessment Examination.by Dr.Dhahirortho
DISCUSSION: In a recent study on restoration of motion following zone I and zone II flexor tendon
repairs in children, age was found to have no effect on the results of zone II tendon repairs. Early
passive motion offered no better results than immobilization for 3 weeks. Immobilization for more
than 4 weeks correlated with poorer results. PREFERRED RESPONSE: 5

42.In a longitudinal study of children with spastic diplegia, analysis of long-term function will most
likely reveal

1- a deterioration of gait stability and an increase in double support time.


2- a deterioration of gait stability and a decrease in double support time.
3- improved excursion about the knee, ankle, and pelvis.
4- improvement in the popliteal angle.
5- increases in single support time.

DISCUSSION: In a longitudinal study of 18 patients with spastic diplegia over a period of 32 months,
three-dimensional gait analysis revealed a deterioration of gait stability with increases in double
support time and decreases in single support time. Kinematic data also identified a loss of excursion
about the knee, ankle, and pelvis. Interestingly, the static examination of the children showed a
decrease in the popliteal angle over time. The authors concluded that ambulatory ability tends to
worsen over time in children with spastic diplegia. PREFERRED RESPONSE: 1

43.Examination of a 7-year-old girl with myelomeningocele reveals calcaneal deformities of both feet.
She ambulates on both extremities wearing ankle-foot orthoses and has no upper extremity aids. She
has grade 5/5 motor strength to the tibialis anterior muscles and absent motor strength to the triceps
surae. There is no varus or valgus deformity of the hindfoot, and the skin over the heels is intact;
however, mild callosities are present. Management should consist of

1- bilateral calcaneal osteotomies.


2- modification of the ankle-foot orthoses.
3- extra-articular subtalar arthrodesis.
4- transfer of the anterior tibial tendon to the calcaneus.
5- Achilles tendon tenodesis to the fibula.

DISCUSSION: A calcaneal deformity of the foot may occur in children who have low lumbar
myelomeningocele. Strong dorsiflexors overcome a weak or absent gastrocnemius-soleus complex,
leading to downward growth of the calcaneal apophysis. The deformity is usually progressive and does
not respond to nonsurgical management. Most authorities recommend transfer of the tibialis anterior
muscle through the interosseous membrane to the posterior aspect of the calcaneus. This procedure has
been reported to be effective in limiting progression of the deformity. An extra-articular subtalar
arthodesis, a treatment option for valgus deformity of the hindfoot, is not indicated. Similarly, Achilles
tendon tenodesis to the fibula, an option for managing valgus of the ankle, is not indicated. Calcaneal
osteotomy may be used in older children with severe calcaneal deformity.

PREFERRED RESPONSE: 4

110 21
American Academy of Orthopaedic Surgeons 22
2001 Pediatric Orthopaedic Self-Assessment Examination.by Dr.Dhahirortho

44.Figure 24 shows the radiograph of a 4-year-old girl with spina bifida. Examination reveals an L3
motor level, excellent sitting and standing balance, and satisfactory range of motion at the hips.
Management should consist of

1- observation.
2- closed reduction and cast immobilization.
3- bilateral medial open reduction of the hips.
4- anterior open reduction with femoral shortening.
5- valgus osteotomy.

DISCUSSION: Children with spina bifida and bilateral


symmetrical dislocation of the hips usually do not
require treatment. A level pelvis and good range of
motion of the hips are more important for ambulation
than reduction of bilateral hip dislocations. Because the patient has good sitting and standing balance
and good range of motion, maintenance of that range of motion and symmetry is more important than
reduction. Surgery is not recommended. PREFERRED RESPONSE: 1

45. Posterior spinal fusion for scoliosis should be performed on a patient with Duchenne muscular
dystrophy when

1- the patient is still ambulatory.


2- lordotic posture is present.
3- the forced vital capacity (FVC) is less than 30% of the predicted value.
4- curve magnitude measures 25° or greater.
5- orthotic management fails.

DISCUSSION: Progressive scoliosis develops in most patients with Duchenne muscular dystrophy.
The onset of spinal deformity typically follows the cessation of walking, and curves can be expected to
progress about 10° per year. Posterior spinal fusion with instrumentation should be performed as soon
as a curve of 25° or greater is documented and before deterioration of pulmonary function (a FVC of
less than 30%) precludes surgery. Patients with kyphotic posture tend to progress more rapidly than
those with lordotic posture. Brace treatment is contraindicated because it is not definitive and it may
mask curve progression while pulmonary function is concomitantly worsening. PREFER RESPO: 4

46. Figures 25a and 25b show the radiograph and MRI scan of a 7 1/2-year-old boy who has been
limping for 1 year. His pain has worsened over the past 2 weeks, and his parents note swelling over the
dorsum of the foot for the past 4 days. Examination reveals no fever, and laboratory studies show a
WBC of 6,700/mm3, an erythrocyte sedimentation rate of 26 mm/h, and a normal C-reactive protein
level. What is the most likely diagnosis?

1- Acute hematogenous osteomyelitis


2- Kohler’s disease
3- Tarsal coalition
4- Tuberculosis
5- Lisfranc fracture-dislocation

111 22
American Academy of Orthopaedic Surgeons 23
2001 Pediatric Orthopaedic Self-Assessment Examination.by Dr.Dhahirortho
DISCUSSION: The diagnosis of tuberculous osteomyelitis in children is often delayed. In one series
of 23 children, the average interval between the onset of symptoms and definite diagnosis was 4.3
months. In these patients, the presenting signs and symptoms were found to be mild, with the most
common signs being localized swelling (69.6%) and a painful disability of the involved limbs (65.2%).
A mild elevation of the erythrocyte sedimentation rate may be present, but the C-reactive protein level
is usually normal. In patients who have osteoarticular tuberculosis, an MRI scan generally shows large
intra-articular effusions, periarticular osteoporosis, and gross thickening of the synovial membrane.
Differential diagnosis between tuberculosis and pyogenic arthritis is difficult, and an accurate diagnosis
usually requires biopsy of synovial tissue. Aspiration of synovial fluid often results in insufficient
information to make a diagnosis. Treatment generally consists of surgical debridement and combined
antituberculous chemotherapy with isoniazid, ethambutol, and rifampin. PREFERRED RESPONSE: 4

47.A 10-year-old boy who plays baseball reports acute pain after throwing a softball from the outfield
to second base. A radiograph is shown in Figure 26. Management should consist of

1- observation during healing of the fracture, followed by bone


marrow or corticosteroid injection.
2- biopsy and appropriate chemotherapy.
3- curettage and bone grafting.
4- high-dose systemic corticosteroids.
5- pulsed electromagnetic fields.

DISCUSSION: The patient has a fracture through a unicameral bone cyst,


as evidenced by the “falling leaf” sign on the radiograph. Following
healing of the fracture, treatment should consist of corticosteroid injection
or bone marrow injection. Some cysts heal with the fracture and do not
require injections. Biopsy is unnecessary because the radiograph shows
that the cyst is benign. Curettage and bone grafting are seldom necessary
because these cysts regularly heal with injections. Corticosteroids are useful when injected into the
cyst, but are not used systemically. Pulsed electromagnetic fields have not been used therapeutically in
this condition. PREFERRED RESPONSE: 1

48. The mother of an otherwise healthy 1-month-old infant reports that he is not moving his left leg
after falling from his high chair 2 days ago. He has a temperature of 99.5°F (37.5°C). Examination
reveals that the left thigh is moderately tender to palpation. Because the infant is apprehensive, range
of motion is difficult to quantify, but appears to be normal at the hips and ankles. Range of motion of
the left knee is approximately 25° to 90°. A radiograph of the leg is shown in Figure 27. Management
should consist of

1- application of a long leg splint and admission to the hospital for further
evaluation and consultation.
2- application of a long leg cast, with follow-up radiographs in 3 days.
3- immediate application of a spica cast under general anesthesia.
4- a CBC, an erythrocyte sedimentation rate, blood cultures, aspiration of the
distal femur for culture, and hospital admission with administration of IV
antibiotics if required.
5- a CBC, an erythrocyte sedimentation rate, a technetium bone scan, and
consultation for bone marrow aspiration.

112 23
American Academy of Orthopaedic Surgeons 24
2001 Pediatric Orthopaedic Self-Assessment Examination.by Dr.Dhahirortho
PREFERRED RESPONSE: 1
DISCUSSION: The patient has a bucket-handle fracture of the distal femur with bilateral corner
fractures of the distal femur and a transverse fracture of the proximal tibia. These fractures are
virtually pathognomonic of child abuse. The infant should be admitted to the hospital, and child
protection services should be notified for investigation of possible abuse. A skeletal survey should be
obtained, along with laboratory studies that include a CBC, a platelet count, a prothrombin time, a
partial thromboplastin time, and a bleeding time.

49. A 12-year-old girl with juvenile rheumatoid arthritis (JRA) has had chronic pain and synovitis
about the knee that is now well-controlled medically. Examination reveals 20° of valgus at the knee.
Knee range of motion shows 10° to 90° of flexion. Treatment should consist of

1- arthroscopic synovectomy.
2- open synovectomy.
3- staple hemiepiphyseodesis.
4- knee arthrodesis.
5- varus osteotomy.

DISCUSSION: Children with JRA frequently have valgus in association with hypervascularity because
of chronic inflammation. This is normally caused by overgrowth of the medial femoral epiphysis.
Staple hemiepiphyseodesis, if done early, can reverse the deformity. Osteotomy is usually unnecessary
at this age, and there is a risk of stiffness of the knee following the procedure. Synovectomy may be
helpful but will not prevent or correct a deformity.

PREFERRED RESPONSE: 3

50. Figure 28 shows the radiograph of a 6-year-old girl who has a right thoracic scoliosis that
measures 60°. Examination shows multiple cafe-au-lait spots, and family history reveals that the
child’s mother has the same disorder. The gene responsible for this disorder codes for

1- dystrophin.
2- frataxin.
3- neurofibromin.
4- peripheral myelin protein.
5- sulfate transport protein.

DISCUSSION: The patient has the dystrophic type of scoliosis seen in


patients with neurofibromatosis type I (NF-1). The NF-1 gene is located
on chromosome 17 and codes for neurofibromin, believed to be a tumor-
suppresser gene. Abnormalities in the dystrophin gene are seen in
Duchenne muscular dystrophy and Becker muscular dystrophy. A
mutation in the frataxin gene is responsible for Friedreich ataxia. The
most common type of hereditary motor and sensory neuropathy (Charcot-Marie-Tooth), HMSN type
IA is caused by a complete duplication of the peripheral myelin protein gene. A defect in the cellular
sulfate transport protein results in undersulfation of proteoglycans seen in diastrophic dysplasia.

PREFERRED RESPONSE: 3
113 24
American Academy of Orthopaedic Surgeons 25
2001 Pediatric Orthopaedic Self-Assessment Examination.by Dr.Dhahirortho
51. Figures 29a and 29b show the radiographs of a 13-year-old competitive gymnast who has had
elbow pain for the past 2 weeks. The pain is worse with tumbling activities. Examination
reveals a mild effusion and slight limitation of extension and forearm rotation with no locking.
Initial management should consist of

1- elbow arthroscopy.
2- arthrotomy and internal fixation of the lesion.
3- cessation of gymnastic activities.
4- use of an elbow brace and continued gymnastic activities.
5- open drilling of the lesion.

DISCUSSION: The radiographs show a lesion in the capitellum that is consistent with osteochondritis
dissecans. There is no evidence of a loose body at this time. Initial management should consist of
cessation of gymnastic activities. Nonsteroidal anti-inflammatory drugs and ice may help to alleviate
acute symptoms; most symptoms usually resolve in 6 to 12 weeks. The patient may then begin range-
of-motion and strengthening exercises, with a slow return to activities once full range of motion and
good strength have been achieved. However, the prognosis for a return to high-level competitive
gymnastics is guarded. Surgery is indicated for intra-articular loose bodies, a locked elbow, or failure
of nonsurgical management. Surgery may be done either open or arthroscopically. Loose bodies
should be removed, and cartilage flaps should be debrided. The results of bone grafting and internal
fixation generally have been poor. Drilling the base of the defect may stimulate replacement with
fibrocartilage, but the benefits of this procedure are not well documented. PREFERRED RESPON: 3

52. A 12-year-old boy who has had a 1-month history of right thigh pain and a limp reports
worsening of the pain after a fall, and he can no longer walk or bear weight on the involved extremity.
Radiographs of the pelvis reveal a slipped capital femoral epiphysis with moderate to severe
displacement. While positioning the patient on the fracture table for screw fixation, partial reduction of
the slip is achieved. No further reduction maneuvers are attempted, and the epiphysis is stabilized with
a single cannulated screw. What complication is most likely to develop following this procedure?

1- Infection
2- Chondrolysis
3- Nonunion
4- Osteonecrosis
5- Epiphyseal arrest

114 25
American Academy of Orthopaedic Surgeons 26
2001 Pediatric Orthopaedic Self-Assessment Examination.by Dr.Dhahirortho
PREFERRED RESPONSE: 4
DISCUSSION: Traditional classification of slipped capital femoral epiphyses is based on the following
temporal criteria: acute (symptoms that persist for less than 3 weeks); chronic (symptoms that persist
for more than 3 weeks); or acute on chronic (acute exacerbation of long-standing symptoms). A newer
classification differentiates between a stable slip where weight bearing is possible, and an unstable slip
if it is not. Reduction of an unstable slip often occurs unintentionally with induction of anesthesia and
positioning of the patient for surgery. The rate of satisfactory results is lower primarily because of a
much higher incidence of osteonecrosis following internal fixation of an unstable slip.

53. Figure 30 shows the AP radiograph of a 9-month-old girl who has been referred for evaluation
of unequal leg lengths. Examination reveals symmetrical abduction of the hips. When the hips are
flexed 90°, the right knee height is greater than the left knee. The girth of the right thigh and calf is
larger than the contralateral side. There are no cutaneous lesions, and examination of the spine is
normal. The infant is moving all extremities equally and spontaneously. Management should consist
of

1- assessment of the bone age in the left wrist and hand.


2- an MRI scan of the spine.
3- an ultrasound of the hips.
4- a renal ultrasound.
5- a Pavlik harness.

DISCUSSION: Hemihypertrophy or hemihypotrophy is usually


idiopathic, and either the leg or the entire side of the body may be
involved. In the infant or young child, it is often difficult to
determine which side is abnormal if the condition is mild. Because
of the association of Wilms’ tumor with hemihypertrophy, these
patients should undergo a yearly renal ultrasound until at least age 5
years. Other conditions that may exhibit hemihypertrophy include
Klippel-Trenaunay-Weber syndrome, Proteus syndrome, and neurofibromatosis. In this patient, the
mild hemihypertrophy is idiopathic. Because of the normal spinal examination and absence of
neurologic findings, an MRI scan is unnecessary. The absence of clinical and radiographic evidence of
hip dysplasia makes both an ultrasound of the hips and application of a Pavlik harness unnecessary.
PREFERRED RESPONSE: 4

54. What is the mechanism of action of an intramuscular injection of botulinum type A toxin in
reducing spasticitiy?

1- Blocks the release of presynaptic acetylcholine release at the neuromuscular junction


and end plate
2- Acts as a -aminobutyric acid (GABA) agonist
3- Inhibits afferent fibers in the dorsal root of the spinal nerve
4- Competitively blocks the acetylcholine receptor
5- Destroys the neuromuscular junction

DISCUSSION: The use of intramuscular botulinum type A toxin has been shown to be a useful
adjuvant in the management of dynamic deformity in patients with cerebral palsy. Botulinum type A
toxin is a neurotoxin produced by Clostridium botulinum that works by interfering with presynaptic
acetylcholine release at cholinergic nerve terminals. At the cellular level, the mechanism involves
115 26
American Academy of Orthopaedic Surgeons 27
2001 Pediatric Orthopaedic Self-Assessment Examination.by Dr.Dhahirortho
endocytosis of the intact botulinum toxin molecule by cells in the end plate, followed by disulfide
cleavage and translocation of the light chain into the cytosol where it disrupts the normal binding of the
synaptosomal vesicles to the axon terminal membrane. Neither the nerve terminal nor the
neuromuscular junction is damaged. The muscle paralysis is reversible and dose-dependent. Baclofen
is a neuropharmacologic agent that functions as a GABA agonist. Dorsal rhizotomy is a neurosurgical
procedure that reduces spasticity by dividing afferent (excitatory) fibers in the posterior rootlet of the
spinal nerves. PREFERRED RESPONSE: 1

55. A 5-year-old boy has had right hip pain and a limp for the past 3 months. Examination of the
right hip reveals irritability and restricted abduction and internal rotation. AP and lateral radiographs of
the hips are shown in Figures 31a and 31b. Initial management should consist of

1- a Salter innominate osteotomy.


2- a shelf acetabuloplasty.
3- a varus femoral osteotomy.
4- symptomatic treatment, including traction, activity modification, and nonsteroidal anti-
inflammatory drugs.
5- abduction bracing.

DISCUSSION: A favorable prognosis can be expected in up to 70% of children with Legg-Calve-


Perthes disease who are younger than age 6 years. Containment treatment has not been shown to alter
the outcome in this age group. The goals of treatment in this patient are to reduce pain (synovitis),
restore motion, and improve function. Symptomatic treatment modalities include bed rest, traction,
crutches, activity modification, and nonsteroidal anti-inflammatory drugs. PREFERRED RESPON: 4

56. Hamstring lengthening and posterior transfer of the rectus femoris will be most successful in a
patient with cerebral palsy who has which of the following gait abnormalities?

1- Excessive knee flexion through stance, stiff knee during swing, and rectus femoris
activity in swing
2- Excessive knee flexion through stance and continuous quadricep activity in stance and
swing
3- Excessive knee flexion through stance, short stride length, and continuous hamstring
activity in swing
4- Knee hyperextension in stance, knee extension during swing, and rectus femoris activity
in swing
5- Knee hyperextension in stance, continuous rectus femoris activity in stance and swing,
and drop foot

116 27
American Academy of Orthopaedic Surgeons 28
2001 Pediatric Orthopaedic Self-Assessment Examination.by Dr.Dhahirortho
PREFERRED RESPONSE: 1
DISCUSSION: Children with cerebral palsy typically ambulate with a crouched gait characterized by
excessive flexion of the hips and knees during stance. Many patients exhibit co-contracture of the
quadriceps and hamstrings, causing a stiff-knee gait. Normally, the rectus femoris fires at the initiation
of swing and in terminal swing through initial contact. Prolonged activity of the rectus femoris
throughout the swing phase interferes with normal knee flexion. This contributes to a stiff knee during
swing phase and prevents clearance of the foot. Lengthening of the hamstrings alone will not improve
foot clearance. Hamstring lengthening is contraindicated when there is hyperextension during stance.
Transfer of the rectus femoris to one of the knee flexors has been shown to improve knee flexion
during swing by an average of 15°. This allows improved foot clearance.

57. Figures 32a and 32b show the radiographs of a 13-year-old boy who sustained a fracture while
playing football 1 week ago. Management at the time of injury included application of a cast and the
use of crutches. A follow-up office visit reveals a normal neurologic examination, and the patient
reports no discomfort with the cast and crutches. Management should now include
1-cast wedging in the outpatient clinic.
2-closed reduction under anesthesia and application of a new
long leg cast.
3-reduction and placement of an intramedullary rod.
4-anatomic open reduction and compression plating with
interfragmentary screws.
5-pins and plaster to maintain the reduction.

DISCUSSION: Stable fractures and minimally displaced


fractures in children can and should be treated by closed
methods. Because loss of reduction is common, alignment of
tibia fractures must be monitored closely for the first 3 weeks
after cast application. This is most easily handled in a
cooperative patient by cast wedging. Some children require
application of a second cast under general anesthesia 2 to 3
weeks after injury, particularly if the subsidence of swelling has caused the cast to loosen. Surgical
indications include the presence of soft-tissue injuries, unstable fracture patterns, fractures associated
with compartment syndrome, and the child with multiple injuries. Surgical options in children include
percutaneous pins, external fixation, plates and screws, and intramedullary nails.
PREFERRED RESPONSE: 1

58. A 14-year-old patient with an L3 myelomeningocele underwent anterior and posterior spinal
fusion for a curve of 50°. Follow-up examination 1 week after the procedure now reveals persistent
drainage from the posterior wound. Results of laboratory cultures show Streptococcus viridans,
Staphylococcus aureus, and Enterococcus. In addition to IV antibiotics, surgical irrigation, and
debridement, management should include

1- removal of all hardware.


2- temporary placement of antibiotic beads.
3- wound closure over drains.
4- bedside dressing changes.
5- a RAST test for latex allergy.
117 28
American Academy of Orthopaedic Surgeons 29
2001 Pediatric Orthopaedic Self-Assessment Examination.by Dr.Dhahirortho

PREFERRED RESPONSE: 3
DISCUSSION: The rate of wound infections has dramatically decreased with the routine use of
prophylactic antibiotics. Factors known to increase the risk of infection include instrumentation,
prolonged surgical time, excessive blood loss, poor perioperative nutritional status, a history of surgery,
and a history of infection. The use of allograft does not result in an increased rate of infection.
Adequate treatment requires early diagnosis and intervention. Temperature elevation and persistent
wound drainage are highly suspicious for infection. An erythrocyte sedimentation rate and a WBC are
not useful in diagnosis unless serial examinations show rising levels. Patients should be taken to the
operating room where the entire wound can be reopened, irrigated, and debrided. Bone graft can be
washed and replaced. Hardware should not be removed. The wound should be closed over suction
drains. IV antibiotics should be given for a period of at least 10 days, followed by 6 weeks orally.
Leaving the wound open to granulate with dressing changes results in prolonged hospitalization,
inadequate treatment of the infection, and a poor cosmetic result.

59.What is the primary mechanism of injury for the fracture shown in Figures 33a and 33b?

1- Hyperdorsiflexion
2- External rotation of the foot
3- Internal rotation of the foot
4- Adduction of the foot and ankle
5- Excessive eversion of the foot and ankle

DISCUSSION: The radiographs show a triplane fracture of the ankle. In adolescence, closure of the
distal tibial physis starts peripherally at the anteromedial aspect of the medial malleolus and extends
posteriorly and laterally. The anterolateral quadrant of the physis is the last to close, making this
region the most susceptible to separation. When the foot is twisted into external rotation, the
anterolateral portion of the epiphysis is avulsed by the pull of the anterior tibiofibular ligament. When
this fragment alone is avulsed, the result is a juvenile Tillaux fracture. When the fracture extends to
involve the remainder of the physis and posterior metaphysis, as in this patient, the result is a triplane
fracture.
PREFERRED RESPONSE: 2
118 29
American Academy of Orthopaedic Surgeons 30
2001 Pediatric Orthopaedic Self-Assessment Examination.by Dr.Dhahirortho
60.Figure 34 shows the standing AP radiograph of a 2-year-old girl who has a left bowleg deformity.
Her mother states that she first noticed the problem when the child began walking at age 10 months,
and the deformity has worsened over the past 6 months. Examination reveals a definite lateral thrust of
the knee during the stance phase of gait. Management should consist of

1- observation.
2- a proximal tibial and fibular osteotomy.
3- daytime ambulatory bracing.
4- elevation of the medial tibial plateau.
5- an MRI scan of the knee.

DISCUSSION: Infantile tibia vara is a developmental condition


characterized by a varus angulation of the proximal end of the tibia that
is caused by a growth disturbance of the proximal medial physis. In a
study of 42 affected extremities in 24 children younger than age 3 years,
it was found that daytime ambulatory brace treatment favorably altered
the natural history of tibia vara. Another study of 27 patients with stage
II Langenskiöld disease found a success rate of 70% (improved
alignment without the need for osteotomy) using brace treatment.
These authors also noted that children with unilateral disease were more
likely to obtain correction of the deformity compared with those with bilateral disease. In this patient,
observation is not warranted because untreated tibia vara has a significant risk for progressive
worsening. Osteotomy is best reserved for those patients who, despite bracing, do not show
satisfactory clinical and radiographic improvement by age 4 years. Elevation of the medial tibial
plateau is a treatment option for older patients who have more advanced disease. An MRI scan would
not provide any useful clinical information at this time. PREFERRED RESPONSE: 3

61. Figures 35a and 35b show the radiographs of a 7-year-old patient who has progressive
deformity of the right thigh accompanied by a dull persistent pain radiating to the knee. Examination
reveals an obvious bulge in the right thigh, with flexion of the hip beyond 50° only if the hip is allowed
to externally rotate. Management should consist of

1- multiple osteotomies and femoral rodding.


2- pharmocologic doses of vitamin D and phosphate.
3- biopsy, followed by appropriate chemotherapy.
4- pamidronate therapy.
5- radiation therapy and a bone marrow transplant.

119 30
American Academy of Orthopaedic Surgeons 31
2001 Pediatric Orthopaedic Self-Assessment Examination.by Dr.Dhahirortho
PREFERRED RESPONSE: 1
DISCUSSION: The patient has radiographic signs of osteogenesis imperfecta, including osteopenia,
mild acetabular protrusio, cortical thinning, and bowing associated with anterior stress fracturing. The
treatment of choice is correction of the bow with osteotomies, followed by intramedullary fixation to
prevent further deformity. Biphosphonates, such as pamidronate, may be useful in increasing bone
density and preventing fractures. Large multicenter studies on biphosphonate efficacy are currently in
progress.

62. Figures 36a and 36b show the MRI scans of a 15-year-old girl who has had pain and recurrent
hemarthrosis in the knee for the past year. Plain radiographs are normal. What is the most likely
diagnosis?

1- Hemangioma of the knee


2- Hemophilia
3- Discoid lateral meniscus
4- Torn medial meniscus
5- Pauciarticular-type juvenile rheumatoid arthritis (JRA)

DISCUSSION: In pediatric patients who have pain and recurrent hemarthrosis in the knee,
hemangioma is often seen as an internal derangement of the knee, and long delays in diagnosis are
common. An MRI scan is noninvasive and will best aid in diagnosis. In this patient, the MRI scan
shows a hemangioma with no evidence of meniscal injury or discoid meniscus. Hemophilia is unlikely
because the patient is female. The presence of hemarthrosis makes JRA an unlikely diagnosis.

PREFERRED RESPONSE: 1

63. A 2-year-old child has marked hypotonia and depressed reflexes. History reveals that the child
was normal at birth and developed normally for the first year. The child also began to ambulate, but
lost this ability during the next 6 months. Laboratory studies show a creatine phosphokinase level that
is within the normal range. DNA testing confirms a deletion in the survival motor neuron (SMN) gene.
What is the most likely diagnosis?

1- Rett syndrome
2- Spinal muscular atrophy, type 2
3- Congenital muscular dystrophy
4- Duchenne muscular dystrophy
5- Congenital myotonic dystrophy

120 31
American Academy of Orthopaedic Surgeons 32
2001 Pediatric Orthopaedic Self-Assessment Examination.by Dr.Dhahirortho
PREFERRED RESPONSE: 2DISCUSSION: The patient has spinal muscular atrophy, type 2. This
type is intermediate in severity between the Werdnig-Hoffmann type (type 1) and the Kugelberg-
Welander type (type 3). It normally manifests itself between the ages of 3 and 15 months. Survival
until adolescence is common. All three types of spinal muscular atrophy have been linked to the SMN
gene at the 5q12.2-13.3 locus. DNA testing is available and is preferred to muscle biopsy because it is
less invasive and more definitive.

64. A 13-year-old boy sustains a valgus stress injury to the knee while playing football, and he is
unable to bear weight after the injury. Examination reveals tenderness medially superior to the joint
line. The knee is held in flexion, and he has a large effusion and localized medial swelling. Plain
radiographs show no obvious fracture. What is the next diagnostic step?

1- Arthroscopy
2- MRI scan
3- Stress radiographs
4- Tomogram
5- Arthrogram

DISCUSSION: In the skeletally mature individual, this mechanism of injury will often result in a
sprain of the medial collateral ligament. In skeletally immature patients, the same mechanism can
cause a fracture of the distal femoral physis. If the fracture is nondisplaced, the plain radiographs may
show only soft-tissue swelling or effusion. While the MRI scan may show edema in the soft tissues on
the medial side of the knee and even an abnormality of the physis, stress radiographs provide a quicker
and less expensive means of making the diagnosis. Arthroscopy and arthrography would not be helpful
in making the diagnosis. Arthroscopy may result in further displacement of the injury.
PREFERRED RESPONSE: 3

65. Figure 37 shows the clinical photograph of a 1-day-old infant who weighed 10.25 lb at birth.
Examination reveals an absent right Moro reflex and limited active motion of the right shoulder, elbow,
and wrist, but flexion of the fingers. Passive range of motion of the shoulder and elbow is normal.
What is the most likely diagnosis?
1- Pseudoparalysis secondary to fracture of the proximal humerus
2- Cervical myelomeningocele
3- Erb palsy
4- Arthrogryposis
5- Cerebral palsy and spastic hemiplegia

DISCUSSION: The patient’s right upper extremity is held in the “head


waiter’s” posture with the shoulder internally rotated, the elbow extended,
and the wrist in flexion. The Erb type of obstetrical brachial plexus palsy
involves the C5 and C6 nerve root, and occasionally, as in this child, the
C7 nerve root. Obstetrical palsy is a traction injury, and is associated with
a high birth weight, shoulder dystocia, cephalopelvic disproportion, or the
use of forceps. Erb palsy is four times more common than injury to the
entire plexus or injury to the C8 and T1 nerve roots. It results from the shoulder being depressed while
the head and neck are laterally rotated, extended, and tilted in the opposite direction. Most patients
121 32
American Academy of Orthopaedic Surgeons 33
2001 Pediatric Orthopaedic Self-Assessment Examination.by Dr.Dhahirortho
recover wrist extension and elbow flexion. Patients with residual weakness of shoulder external
rotation and abduction will benefit from release of the pectoralis major, latissimus dorsi, and teres
major, with transfer of the latissimus dorsi and the teres major to the posterosuperior aspect of the
rotator cuff. Recent studies using arthrograms and CT scans have shown a higher incidence of
posterior glenoid deficiency and posterior subluxation than that observed with plain radiographs. The
posterior subluxation or dislocation can be effectively reduced by tendon release and transfer
procedures. PREFERRED RESPONSE: 3

66. Figure 38 shows the radiograph of a 5-year-old child who sustained a type III supracondylar
fracture. Examination reveals the absence of a radial pulse, but an otherwise well-perfused hand.
Following closed reduction and percutaneous pinning, the radial pulse remains absent; however, the
hand is pink and well perfused. Management should now include
1- close observation with frequent neurovascular checks.
2- emergency angiography.
3- emergency exploration of the brachial artery.
4- removal of pin fixation and exploration of the brachial artery.
5- thrombectomy.

DISCUSSION: In a study of over 400 patients with displaced supracondylar


fractures, 3.2% of the fractures were associated with the absence of the radial
pulse with an otherwise well-perfused hand. Based on this study, a period of
close observation with frequent neurovascular checks should be completed
before attempting invasive correction of the problem. Because of the
satisfactory results with expectant management, angiography, exploration,
removal of fixation and exploration, and thrombectomy are contraindicated.
PREFERRED RESPONSE: 1

67. Figures 39a and 39b show the radiographs of an otherwise healthy 10-year-old boy who has had
thigh pain and a limp for the past 9 months. Examination reveals that the left lower extremity is 1 cm
shorter, with reduced flexion, abduction, and internal rotation on the left side. The patient is at the 50th
percentile for height and the 90th percentile for weight. Serum studies will most likely show

1- an elevated thyroid-stimulating hormone level.


2- an elevated estrogen level.
3- elevated blood urea nitrogen and creatinine levels.
4- a growth hormone deficiency.
5- normal laboratory values.
122 33
American Academy of Orthopaedic Surgeons 34
2001 Pediatric Orthopaedic Self-Assessment Examination.by Dr.Dhahirortho
PREFERRED RESPONSE: 5
DISCUSSION: The patient has a slipped capital femoral epiphysis (SCFE) at a younger than average
age (average age 13.5 years for boys and 12.0 years for girls); therefore, an etiology that is not
idiopathic must be considered. Hypothyroidism can result in a SCFE, but these children typically fall
into the category of less than the 10th percentile for height. SCFE may develop in children with a
growth hormone deficiency who have undergone hormonal replacement. Osteodystrophy caused by
chronic renal failure may result in a SCFE, but the bone quality is markedly osteopenic on radiographs
and the children are chronically ill with both low height and weight percentiles. An elevated estrogen
level results in physeal closure and is protective to physeal slippage. Therefore, this child will most
likely have normal laboratory values.

68. A 7-year-old patient has had a painless limp for several months. Examination reveals pain and
spasm with internal rotation, and abduction is limited to 10° on the involved side. Management
consists of 1 week of bed rest and traction, followed by an arthrogram. A maximum abduction/internal
rotation view is shown in Figure 40a, and abduction and adduction views are shown in Figures 40b and
40c. The studies are most consistent with

1- Catterall II involvement.
2- tubercular synovitis.
3- Herring type A involvement.
4- hinge abduction.
5- osteochondritis dissecans.

DISCUSSION: The radiographs show classic hinge abduction. The diagnostic feature is the failure of
the lateral epiphysis to slide under the acetabular edge with abduction, and the abduction view shows
medial dye pooling because of distraction of the hip joint. Persistent hinge abduction has been shown
to prevent femoral head remodeling by the acetabulum. Radiographic changes are characteristic of
severe involvement with Legg-Calve-Perthes disease. The Catterall classification cannot be well
applied without a lateral radiograph, but this degree of involvement would likely be considered a grade
III or IV. Because the lateral pillar is involved, this condition would be classified as type C using the
Herring lateral pillar classification scheme. PREFERRED RESPONSE: 4

69. A 14-year-old football player has had thigh pain and weakness following a full-contact
scrimmage 24 hours ago. He recalls that he felt a sharp pain in his back after colliding with a much
heavier player. Examination reveals that the spine is minimally tender to palpation in the upper lumbar
region. Motor testing reveals quadriceps weakness bilaterally, and a reverse straight leg raising test is
positive. Plain radiographs of the thoracolumbar spine are normal. A myelogram, a CT scan with
contrast, and an MRI scan are shown in Figures 41a through 41c. What is the most likely diagnosis?

123 34
American Academy of Orthopaedic Surgeons 35
2001 Pediatric Orthopaedic Self-Assessment Examination.by Dr.Dhahirortho

1- Disk herniation
2- Congenital spinal stenosis
3- Intraspinal tumor
4- Vertebral end plate fracture
5- Facet subluxation

DISCUSSION: Fracture of the vertebral end plate is a relatively uncommon injury that is most often
seen in adolescent boys. The injury is characterized by traumatic displacement of the vertebral ring-
apophysis into the spinal canal and associated disk herniation. Over one third of these injuries are seen
in children with lumbar Scheuermann disease. The injury most frequently involves the midlumbar
vertebra, and symptoms are often indistinguishable from those associated with a herniated disk. The
injury is usually not visible on plain radiographs. The diagnosis is typically made after obtaining MRI
or contrast CT scans. Treatment consists of laminotomy and excision of the osteochondral fragments.
PREFERRED RESPONSE: 4

70. Figure 42 shows the radiograph of a 12-year-old boy who has a limp and pain in the left hip
with athletic activity. Examination reveals decreased abduction and internal rotation of the left hip,
with pain at the extremes of motion and a 1-cm limb-length discrepancy. Management should consist
of

1- total hip arthroplasty.


2- innominate osteotomy.
3- varus osteotomy of the proximal femur.
4- valgus osteotomy of the proximal femur.
5- a shoe lift.

DISCUSSION: The radiograph shows changes that are most consistent


with Legg-Calve´ Perthes disease. Valgus extension osteotomy is the
salvage procedure of choice in patients with late symptomatic Perthes
disease with severe joint incongruity. Prerequisites for valgus
extension osteotomy include an adequate range of hip adduction and
proof of improved congruity in the new position. Total hip
124 35
American Academy of Orthopaedic Surgeons 36
2001 Pediatric Orthopaedic Self-Assessment Examination.by Dr.Dhahirortho
arthroplasty is not a good alternative in the young patient. Varus osteotomy would further shorten the
extremity and place a flattened portion of the femoral head in the acetabulum. A prerequisite of the
innominate osteotomy is a congruent reduction. PREFERRED RESPONSE: 4

71. The mother of a 5-year-old child reports that he has had a fever of 103°F (39.4°C), leg swelling,
and has been unwilling to bear weight on his right lower leg for the past 7 days. Examination reveals
point tenderness at the distal femur. Aspiration at the metaphysis yields 10 mL of purulent fluid, and a
Gram stain reveals gram-positive cocci. In addition to hospital admission, management should include

1- IV antibiotics.
2- IV antibiotics and immobilization of the lower extremity.
3- IV antibiotics, surgical incision and drainage of the abscess, and immobilization of the lower
extremity.
4- IV antibiotics and repeated aspirations of the metaphyseal region.
5- oral antibiotics with careful monitoring of blood levels.

DISCUSSION: The patient has a subperiosteal abscess. Because aspiration revealed 10 mL of purulent
fluid, the treatment of choice is surgical incision and drainage of the abscess, followed by
immobilization to reduce the risk of pathologic fracture. With an adequate response to IV antibiotics
and a susceptible bacteria, the patient may then be switched to oral antibiotics. PREFE RESPONSE: 3

72. Figure 43 shows the lateral radiograph of a 12-year-old boy with mild osteogenesis imperfecta
who injured his left elbow after pushing his brother. Treatment should consist of

1-closed reduction and cast immobilization.


2-closed reduction and percutaneous pinning.
3-open reduction and fixation using an absorbable suture.
4-open reduction and fixation using two Kirschner wires and a
figure-of-8 tension band of absorbable suture.
5-open reduction and fixation using an intramedullary screw.

DISCUSSION: The patient has a displaced fracture of the


apophysis of the olecranon for which most authorities
recommend surgical treatment. In older children, stability of the
reduction may be achieved by the use of two parallel medullary
Kirschner wires and a figure-of-8 tension band loop of either stainless steel wire or absorbable suture.
The use of an absorbable suture does not require removal of the implant. Absorbable suture alone is
best used in very young patients who have this type of injury. An intramedullary screw would pose an
unnecessary risk of future growth disturbance.
A displaced, isolated fracture of the apophysis of the olecranon is an unusual injury in a child. It has
been suggested by several authors that children who have osteogenesis imperfecta may be especially
prone to this injury. One study reported seven of these fractures occurring in five children who had the
mild form of osteogenesis imperfecta (Sillence type IA). The authors of this study suggest that the
diagnosis of osteogenesis imperfecta be considered in any child who has a displaced fracture of the
apophysis of the olecranon, especially when the injury is associated with relatively minor trauma.

PREFERRED RESPONSE: 4
125 36
American Academy of Orthopaedic Surgeons 37
2001 Pediatric Orthopaedic Self-Assessment Examination.by Dr.Dhahirortho
73. Figure 44 shows the radiograph of an 11-year-old girl who has hip pain. Further diagnostic
workup should include

1- a renal ultrasound.
2- an echocardiogram.
3- an electromyogram.
4- an MRI scan of the spine.
5- an iliac crest biopsy.

DISCUSSION: The patient has severe acetabular


protrusio, a condition that is frequently associated with
Marfan syndrome. An echocardiogram is necessary to
rule out the most serious consequence of this syndrome,
aortic root widening, which can lead to aortic valve
dysfunction or fatal aortic rupture. An electromyogram may be indicated for Charcot-Marie-Tooth
disease, which is associated with acetabular dysplasia, but not protrusio. The renal ultrasound, the MRI
scan, and the biopsy would be of no value in this patient. Protrusio can also be seen in patients with
osteogenesis imperfecta and juvenile rheumatoid arthritis. PREFERRED RESPONSE: 2

74. Figure 45 shows the radiograph of a 2-year-old patient who has progressive lumbar scoliosis as
the result of hemivertebra. Examination reveals no associated cutaneous lesions, and an MRI scan
shows no associated intraspinal anomalies. Treatment should consist of

1-hemivertebra excision.
2-anterior and posterior spinal fusion with instrumentation from T4 to L4.
3-convex anterior hemiepiphyseodesis.
4-convex posterior hemiarthrodesis.
5-an orthosis.

DISCUSSION: In a retrospective review of 10 patients treated with


hemivertebra excision for hemivertebra in the levels of T12 to L3, the
procedure was found to be safe and effective. The procedure provided an
average curve correction of 67° and was greatest in patients who were
younger than age 4 years at the time of surgery. Long anterior and
posterior fusion with instrumentation is not the treatment of choice at this
age. Either anterior hemiepiphyseodesis or posterior hemiarthrodesis in
this isolated hemivertebra setting would be inadequate. Brace treatment is ineffective in management
of the primary curvature. PREFERRED RESPONSE: 1

75. A 10-year-old girl with a history of an obstetrical brachial plexus palsy has been referred for
evaluation. Examination reveals a severe adduction internal rotation contracture of the shoulder and a
mild flexion contracture of the elbow. Hand function is normal. Radiographs show mild glenohumeral
joint incongruity. To achieve the best functional outcome, management should consist of

1- physical therapy to stretch the tight structures.


2- a humeral rotational osteotomy.
3- anterior shoulder release and posterior muscle transfers.
4- anterior shoulder release.
5- shoulder fusion.
126 37
American Academy of Orthopaedic Surgeons 38
2001 Pediatric Orthopaedic Self-Assessment Examination.by Dr.Dhahirortho

PREFERRED RESPONSE: 2
DISCUSSION: The patient has an upper plexus palsy (Erb palsy) with severe shoulder contracture.
While physical therapy for stretching is the treatment of choice to prevent contracture in the newborn,
it is unlikely to be of benefit in the older child with an established contracture. Contracture release
alone or in combination with muscle transfers can improve the cosmetic appearance, and in the case of
a mild deformity, may also improve function. These procedures are less likely to help when there is
deformity of the shoulder joint or when arthritic changes are present. The procedure of choice for an
older child with joint deformity is rotational osteotomy of the proximal humerus because it can
improve cosmesis and function, even in the face of joint deformity.

76. A 4-year-old boy sustained a nondisplaced, but complete, fracture of the left proximal tibial
metaphysis 1 year ago. The fracture healed uneventfully in an anatomic position. Examination of the
injured extremity now reveals 18° of valgus compared with 3° of valgus on the opposite side.
Management should now include

1- observation.
2- a knee-ankle-foot orthosis.
3- a medial proximal tibial epiphyseodesis.
4- a proximal tibial osteotomy.
5- lateral proximal tibial stapling.

DISCUSSION: The development of a valgus deformity after this type of fracture is a well-known
occurrence, and the patient’s parents should be informed about this risk. In a patient who is age 4
years, the natural history is one of gradual correction by the development of a physiologic varus
deformity at the distal tibial physis; therefore, no active intervention is needed at this time. Bracing has
no effect on the deformity, and the child is too young for any procedure on the growth plate. Proximal
tibial osteotomy is reserved until the patient nears skeletal maturity because of the risk of recurrence of
the deformity. Lateral stapling can be done near skeletal maturity if the deformity persists, but this is
unlikely to be necessary. PREFERRED RESPONSE: 1

77. A 16-year-old boy with spastic quadriplegic cerebral palsy has been referred for evaluation and
management of scoliosis. His parents report increasing problems with sitting balance, positioning, and
hygiene because of the deformity. The radiograph shown in Figure 46 reveals a lordoscoliosis of 105°
with marked pelvic obliquity. Attempts at correcting the pelvic obliquity on supine bending
radiographs show significant rigidity. Management should consist of

1- a thoracolumbosacral orthosis.
2- posterior spinal fusion.
3- anterior and posterior spinal fusion.
4- electrical stimulation.
5- wheelchair modifications.

DISCUSSION: Spinal stabilization is the treatment of choice in patients with


severe scoliosis who have progressive positioning, sitting balance, and/or
hygiene problems despite maximal nonsurgical management. Pelvic rigidity
and marked frontal plane deformity necessitate anterior and posterior
procedures so as to maximize correction and fusion.
PREFERRED RESPONSE: 3
127 38
American Academy of Orthopaedic Surgeons 39
2001 Pediatric Orthopaedic Self-Assessment Examination.by Dr.Dhahirortho
78. A 10-year-old boy reports a gradual onset of weakness; however, he is fully ambulatory.
History reveals that he has a 17-year-old brother who has just stopped walking because of a similar
condition. Laboratory studies show a creatine kinase level of 5,480 IU/L (normal 25 to 232 IU/L), and
examination shows a slightly positive Gower sign. What is the most likely diagnosis?

1- Spinal muscular atrophy, type II


2- Myotonic dystrophy
3- Duchenne muscular dystrophy
4- Becker muscular dystrophy
5- Hereditary motor sensory neuropathy, type II (HMSN)

DISCUSSION: The patient has Becker muscular dystrophy. Patients with this condition have a slower
rate of progression of disease compared with patients who have Duchenne muscular dystrophy, and
walking may continue into the late teens. The creatine kinase level is not as high as in Duchenne
muscular dystrophy, which can range from 20,000 to 30,000 IU/L. Becker muscular dystrophy is
allelic to Duchenne muscular dystrophy, resulting in a mutation in the dystrophin gene. Myotonic
dystrophy is characterized by a progressive inability to relax the muscles after contracture. The Gower
sign is not helpful in this disease. Patients with Charcot-Marie-Tooth disease, one type of which is also
known as HMSN type II, do not have elevated creatine kinase levels and usually present with a foot
deformity. Spinal muscular atrophy, type II, usually presents with severe weakness in the second year
of life. PREFERRED RESPONSE: 4

79. Figure 47 shows the radiograph of a 2-day-old girl who has been referred for swelling and
limited use of the right upper extremity. The second of twins, the infant was breech and delivered with
forceps at age 38 weeks, weighing 5.37 lb. Difficulty in moving the arm was noted shortly after birth.
Examination shows no active motion of the shoulder, elbow, or wrist. Active finger flexion and
extension are present. The elbow is mildly swollen, and passive motion shows lack of full extension of
20°, lack of full flexion of 15°, and no restriction of pronation or supination. What is the most likely
diagnosis?

1- Obstetrical brachial plexus palsy


2- Congenital dislocation of the elbow
3- Congenital dislocation of the radial head
4- Arthrogryposis
5- Transphyseal fracture of the distal humerus

DISCUSSION: Fractures involving the entire distal humeral physis may be a


complication of a difficult delivery. Basing the diagnosis on radiographs can be
difficult at this age because the secondary ossification center of the lateral
condyle has not developed. The key to the diagnosis is the constant relationship
of the radius and ulna, with medial and posterior displacement of the forearm
relative to the humerus. An ultrasound can be obtained to confirm the diagnosis
in newborns. Because the fracture is through cartilage, examination may reveal
only mild swelling, and crepitation may be muffled or not apparent. The lack of
apparent active motion of the shoulder, elbow, and wrist is secondary to
pseudoparalysis. Child abuse is a common mechanism of this injury in a child who is age 1 month to
age 3 years.
PREFERRED RESPONSE: 5
128 39
American Academy of Orthopaedic Surgeons 40
2001 Pediatric Orthopaedic Self-Assessment Examination.by Dr.Dhahirortho
80. Examination of a 12-year-old girl with a painful flatfoot deformity reveals tenderness in the
region of the sinus tarsi and no appreciable subtalar motion. Radiographs are shown in Figures 48a
through 48c. Two attempts to relieve her symptoms by cast immobilization fail to relieve the pain.
Management should now consist of

1- triple arthrodesis.
2-manipulation of the foot under general anesthesia.
3-continued nonsurgical management until the
synchondrosis ossifies.
4-resection of the coalition and interposition with the extensor digitorum brevis.
5-a medial closing wedge osteotomy of the calcaneus.

DISCUSSION: Surgical treatment is indicated for a symptomatic tarsal coalition that has failed to
respond to nonsurgical management. In this patient, the radiographs reveal a calcaneonavicular
coalition and no degenerative changes. The patient is symptomatic, and two attempts at use of a short
leg walking cast have failed to provide relief. For calcaneonavicular coalitions, good results have been
reported following resection and interposition of the extensor digitorum brevis. A retrospective study
of this procedure achieved good to excellent results in 58 of 75 feet (77%). Degenerative arthritis or
persistent pain following resection of a coalition is a reasonable indication for a triple arthodesis. A
medial closing wedge osteotomy of the calcaneus may be indicated for a rigid flatfoot with severe
valgus deformity. There are no studies documenting the long-term effectiveness of a manipulation
under general anesthesia for this condition. PREFERRED RESPONSE: 4

81. When counseling a patient with hypophosphatemic rickets, which of the following scenarios
will always result in a child with the same disorder?

1- Female patient who has a female child


2- Female patient who has a male child
3- Male patient who has a female child
4- Male patient who has a male child
5- Disorder not inherited

DISCUSSION: Hypophosphatemic rickets is an inherited disorder that is transmitted by a unique sex-


linked dominant gene. Therefore, if a male patient has a female offspring, his affected X chromosome
will be transmitted and all of his female children will have hypophosphatemic rickets. All male
129 40
American Academy of Orthopaedic Surgeons 41
2001 Pediatric Orthopaedic Self-Assessment Examination.by Dr.Dhahirortho
offspring of a male patient will be unaffected. All offspring of a female patient have a 50% chance of
having the disorder. Understanding the inheritance of hypophosphatemic rickets facilitates early
diagnosis and early treatment. Medical treatment with phosphorus and some types of vitamin D (most
authors recommend calcitriol) improves, but does not fully correct, the mineralization defect in
hypophosphatemic rickets. However, if medical treatment is begun before the child begins walking,
the growth plate is then adequately protected and a bowleg deformity will most likely be prevented.
PREFERRED RESPONSE: 3

82. A 15-year-old boy with Duchenne muscular dystrophy has a progressive scoliosis that now
measures 55°. He is in foster care and is no longer ambulatory. Because posterior spinal fusion with
instrumentation is the recommended treatment, the patient participates in a thorough discussion of the
risks and benefits of the procedure. However, he refuses the surgery. The physician should now

1- obtain a court order to perform the surgery.


2- follow the patient clinically.
3- place the patient in a brace.
4- perform the surgery with permission from the legal guardians.
5- perform the surgery with the consent of two surgeons.

DISCUSSION: Traditionally, patients have been viewed as ignorant about medical matters and ill-
equipped to determine what is in their best interest. This has been especially true for minors.
However, recent informed consent policies are now based on the patient’s right to self-determination.
While most spinal surgeons would agree that spinal fusion improves pulmonary function, sitting
balance, and comfort, they would also agree that this comes at considerable risk in a patient with
compromised pulmonary function and ultimately, a terminal condition. With increasing frequency,
young people older than age 14 years are gaining greater autonomy in decision making about their
health care matters. This includes do not resuscitate orders when young patients are terminally ill, as
well as in less serious situations. Surgery could be performed with the permission of the legal
guardians; however, in this situation it is preferable to follow the patient clinically until he consents to
surgery along with the legal guardians. Bracing is contraindicated. PREFERRED RESPONSE: 2

83. Figure 49 shows the radiograph of a 3-year-old child with progressive bowlegs. Laboratory
studies show a calcium level of 9.5 mg/dL (normal 9.0 to 11.0 mg/dL), a phosphorus level of 4.2
mg/dL (normal 3 to 5.7 mg/dL), and an alkaline phosphatase level of 305 IU/L (normal 104 to 345
IU/L). What is the most likely diagnosis?

1- Blount’s disease
2- Hypophosphatemic rickets
3- Nutritional rickets
4- Schmid metaphyseal dysostosis
5- Jansen metaphyseal dysostosis

DISCUSSION: The patient has bowlegs associated with very wide


physes, particularly noted at the hips. The widening of the growth
plates is a classic sign of rickets; however, the normal levels of
calcium, phosphorus, and alkaline phosphatase rule out both
130 41
American Academy of Orthopaedic Surgeons 42
2001 Pediatric Orthopaedic Self-Assessment Examination.by Dr.Dhahirortho
nutritional and hypophosphatemic rickets. Patients with nutritional rickets or hypophosphatemic
rickets have hypophosphatemia and increased alkaline phosphatase levels. Jansen metaphyseal
dysostosis has very severe radiographic findings that are not found in this patient; however, these
radiographic findings are classic for Schmid metaphyseal dysostosis. This disorder is caused by a
mutation in the gene for type X collagen, which is found only in the growth plates of growing children.
PREFER RESPONSE: 4

84. Figures 50a and 50b show the standing clinical photographs of a 12-year-old boy who has had
increasing pain in the left foot for the past 9 months. He reports that the pain is activity related, aching
in nature, and localized to the medial aspect of the midfoot and hindfoot. History reveals that he
sustained a puncture wound located superior and posterior to the medial malleolus from a plate glass
window 18 months ago. Examination reveals no restriction of ankle or subtalar motion, normal
neurovascular status, no masses, and a well-healed 1.5-cm laceration posterior to the superior aspect of
the medial malleolus. Inversion strength of the foot is decreased to grade 3/5. Radiographs of the foot
show no bony abnormalities. Treatment should consist of

1- application of a UCBL orthoses.


2- application of an ankle-foot orthosis.
3- transfer of the flexor digitorum longus to the posterior tibialis tendon.
4- a lengthening osteotomy of the calcaneus.
5- talocalcaneal arthrodesis.

DISCUSSION: The photographs show a planovalgus posture of the foot. The foot deformity and
decreased inversion strength are secondary to laceration of the posterior tibial tendon 18 months ago.
If the injury had been recognized acutely, optimal treatment would have consisted of repair of the
tendon; however, contracture now precludes that possibility. Therefore, transfer of the flexor digitorum
longus or flexor hallucis longus is the preferred treatment. In adults with posterior tibial dysfunction,
the entire tendon is typically degenerated and the transfer must be anchored through a drill hole in the
navicular. In this patient, the distal end of the posterior tibial tendon is a satisfactory insertion site.
Lengthening osteotomy of the calcaneus could be combined with the tendon transfer if the patient had a
fixed deformity of the foot. UCBL orthoses and an ankle-foot orthosis are not considered good long-
term solutions for a 12-year-old patient. PREFERRED RESPONSE: 3

85. Which of the following clinical scenarios represents an appropriate indication for convex
hemiepiphysiodesis/hemiarthrodesis in the treatment of a child with a congenital spinal deformity?
A 3-year-old child with a hemivertebra opposite a contralateral bar and thoracic scoliosis that measures
53°
2-A 4-year-old child with a fully segmented L1 hemivertebra and scoliosis that measures 80°
3-A 4-year-old child with a fully segmented T10 hemivertebra and scoliosis that measures 50°
131 42
American Academy of Orthopaedic Surgeons 43
2001 Pediatric Orthopaedic Self-Assessment Examination.by Dr.Dhahirortho
4-A 4-year-old child with a posterolateral hemivertebra at the thoracolumbar junction and a
kyphoscoliotic deformity that measures 45°
5-A 10-year-old child with a hemivertebra and scoliosis that measures 50°

DISCUSSION: Convex hemiarthrodesis and hemiepiphysiodesis are procedures designed to gradually


reduce curve magnitude in congenital scoliosis because of hemivertebrae. They are used to surgically
create an anterior and posterior bar to arrest growth on the convexity of the existing deformity. Success
of the technique is predicated on continued growth on the concave side of the deformity. Prerequisites
for this procedure include curves of limited length (less than or equal to five vertebrae), curves of
reasonable magnitude (less than 70°), absence of kyphosis, concave growth potential, and appropriate
age (younger than age 5 years). PREFERRED RESPONSE: 3

86. Figures 51a through 51c show the radiographs of a 7-year-old soccer player who reports a
gradual onset of midfoot pain that began shortly after the start of soccer season. He states that the pain
is worse with activity and is partially alleviated by rest. Examination reveals soft-tissue swelling, and
tenderness and warmth in the region of the talonavicular and navicular cunieform joints. Management
should consist of
1-biopsy.
2-curettage and bone grafting.
3-open reduction and fixation.
4-immobilization with a long leg cast and
no weight bearing.
5-immobilization with a short leg walking
cast or fracture boot.

DISCUSSION: Osteochondrosis of the


tarsal navicular (Kohler disease) is an
infrequent cause of midfoot pain in
children, and the etiology is unknown.
The typical radiographic findings include
flattening and irregular ossification of the tarsal navicular. The medial cunieform and talus maintain
their normal articular contours. The acute process is best treated with rest and immobilization. A short
leg walking cast results in relief of pain and a quicker return to activity compared with orthotics,
although long-term success is similar with either method of treatment. Children may return to activities
when the symptoms subside. The radiographic appearance of the talus begins to normalize by about 8
to 10 months following the onset of symptoms... PREFERRED RESPONSE: 5

87. A 10-year-old child with cerebral palsy undergoes bilateral hamstring lengthening for severe
knee flexion contractures, and knee immobilizers are applied postoperatively. Examination at the
initial postoperative check 2 hours after surgery reveals that she can dorsiflex her toes on the right foot,
but not on the left foot. The physician should now

1- repeat the examination in a few hours.


2- remove the left knee immobilizer and flex the knee.
3- apply long leg casts that include the feet.
4- obtain an electromyogram and nerve conduction velocity studies.
5- perform peroneal nerve exploration.
132 43
American Academy of Orthopaedic Surgeons 44
2001 Pediatric Orthopaedic Self-Assessment Examination.by Dr.Dhahirortho

PREFERRED RESPONSE: 2
DISCUSSION: Children with cerebral palsy are often difficult to examine. However, this patient
clearly has a peroneal nerve deficit, most likely from the acute stretch after the hamstring lengthening.
The nerve has the best chance of recovery if it is relaxed by flexing the knee. Once the nerve has
recovered, gradual knee extension can be accomplished.

88. Figures 52a and 52b show the plain radiographs of a 12-year-old girl who has right distal leg
pain. She reports that symptoms are present with weight-bearing activities and improve with rest.
Examination reveals diffuse tenderness over the distal tibial metaphysis and mild swelling. A
photomicrograph of the biopsy specimen is shown in Figure 52c. What is the most likely diagnosis?

1- Giant cell tumor


2- Aneurysmal bone cyst
3- Unicameral bone cyst
4- Eosinophilic granuloma
5- Ewing’s sarcoma

DISCUSSION: This lytic lesion is in the epiphyseal-metaphyseal region of the distal tibia. The most
common lesion in this area is a giant cell tumor. Although these lesions are most commonly seen in
adults, they can also occur in the skeletally immature patient. The photomicrograph shows a lesion
with multiple giant cells, the nuclei of which are similar to those in the background stroma; this finding
is characteristic of giant cell tumors. Giant cells can be seen in many benign lesions, including
aneurysmal bone cysts, Brown tumors, and eosinophilic granuloma. These lesions usually have fewer
giant cells with less nuclei. The location of this lesion in the epiphyseal-metaphyseal area is not seen in
aneurysmal bone cysts, unicameral bone cysts, Ewing’s sarcoma, or eosinophilic granuloma.
PREFERRED RESPONSE: 1

89. A 15-year-old boy with epilepsy who is treated with phenytoin sustains a vertebral compression
fracture during a breakthrough seizure. Radiographs of the spine reveal generalized osteopenia.
What is the most likely cause of the osteopenia?

1- Induced osteoporosis
2- Acquired osteomalacia
3- Calcitonin effect
4- Hyperparathyroidism
5- Disuse osteoporosis

133 44
American Academy of Orthopaedic Surgeons 45
2001 Pediatric Orthopaedic Self-Assessment Examination.by Dr.Dhahirortho
PREFERRED RESPONSE: 2

DISCUSSION: As a side effect of treatment, phenytoin induces osteomalacia, or rickets, in growing


children, through interference with metabolism of vitamin D. Oral supplementation of vitamin D can
minimize this effect in patients who are undergoing prolonged treatment with phenytoin.

90. Figure 53 shows the pedigree of a family with an unusual type of muscular dystrophy. This
pedigree is most consistent with what type of inheritance pattern?
1- Autosomal-dominant
2- Autosomal-recessive
3- X-linked dominant
4- X-linked recessive
5- Mitochondrial inheritance

DISCUSSION: The pedigree documents involvement of male


offspring only, and it also shows transmission through an
uninvolved female carrier. This inheritance pattern is most
consistent with a x-linked recessive inheritance. It would be
inconsistent with a dominant inheritance pattern unless there
was incomplete penetrance. Autosomal-recessive inheritance would be possible only if the family
member labeled II.F was also a carrier of the same gene; however, this is unlikely. Mitochondrial
inheritance is possible, but as with autosomal patterns, mitochondrial inheritance normally affects both
male and female offspring. It is transmitted only through the maternal line.

PREFERRED RESPONSE: 4

91. Which of the following is considered the most accurate test to determine the amount of limb-
length discrepancy in a patient with a knee flexion contracture of 35°?

1- Standing block test


2- Standing AP radiograph of the pelvis with blocks under the foot of the short leg
3- CT scanogram
4- Scanogram
5- Measurement from the anterior superior iliac spine to the medial malleolus

DISCUSSION: Flexion contractures and angular deformities of a limb cause inaccurate limb-length
measurement results with most clinical methods. A CT scanogram is more accurate than standard
scanograms for determining limb length in patients with knee flexion contractures of 30° or more. The
cost and time necessary to complete the examinations are comparable, but the CT scanogram delivers
only 20% of the radiation needed for standard scanograms.

PREFERRED RESPONSE: 3

134 45
American Academy of Orthopaedic Surgeons 46
2001 Pediatric Orthopaedic Self-Assessment Examination.by Dr.Dhahirortho
92. Figures 54a and 54b show the radiograph and MRI scan of a 7-year-old boy who has a painful
right thoracic scoliosis that measures 35°. Neurologic examination is normal. Management should
consist of

1- repeat radiographs in 6 months.


2- a technetium Tc 99m bone scan.
3- posterior spinal fusion.
4- anterior and posterior spinal fusion.
5- a neurosurgical consultation.

DISCUSSION: Because hydrosyringomyelia, with or without an Arnold-Chiari malformation, is now


being recognized as the etiology of many infantile and juvenile idiopathic scolioses, management
should consist of a neurosurgical consultation. Observation with follow-up radiographs is not an
option in curves of this magnitude. A technitium Tc 99m bone scan is unnecessary because the
etiology of the curve has been identified. Although spinal fusion may be needed in the future, it should
not be undertaken before the neurosurgical problem has been addressed. PREFERRED RESPONSE: 5

93. A 10-month-old girl has the spinal deformity shown in Figures 55a and 55b with no apparent
neurologic finding. The next step in evaluation should be to obtain

1- a genitourinary ultrasound.
2- an MRI scan of the spine.
3- an AP radiograph of the pelvis.
4- an electromyogram and nerve conduction velocity studies.
5- an echocardiogram.

135 46
American Academy of Orthopaedic Surgeons 47
2001 Pediatric Orthopaedic Self-Assessment Examination.by Dr.Dhahirortho
PREFERRED RESPONSE: 1
DISCUSSION: Approximately 60% of patients with a congenital spine abnormality have associated
malformations outside the spinal column. Genitourinary abnormalities are probably the most common,
occurring in up to 37% of patients. These are usually anatomic anomalies, such as renal agenesis,
duplication, fusion, and ectopia. A genitourinary ultrasound is the least invasive screening tool. Other
associated anomalies include cervical vertebral abnormalities, VATER syndrome, and intraspinal
abnormalities such as diastematomyelia. An MRI scan is not recommended as part of the screening
examination; however, if the patient had neurologic signs or symptoms, an MRI scan would be
indicated

94. A 12-year-old girl has scoliosis at T5-T10 that measures 62°. A clinical photograph of the
axilla is shown in Figure 56. Management should consist of

1-a thoracolumbosacral orthosis.


2-in situ posterior spinal fusion.
3-posterior spinal fusion with segmental instrumentation.
4-anterior spinal fusion with instrumentation.
5-anterior and posterior spinal fusion with posterior
segmental instrumentation.

DISCUSSION: Neurofibromatosis type 1 (NF-1) is an


autosomal-dominant disorder affecting about 1 in 4,000
people. NF-1 causes tumors to grow along various types of
nerves and affects the development of non-nervous tissues,
such as bone and skin. The gene for NF-1 is located on the long arm of chromosome 17 and codes the
protein neurofibromin. Research indicates that NF-1 acts as a tumor-suppressor gene and, as such,
plays an important role in the control of cell growth and differentiation. Axillary and inguinal freckling
is considered a good diagnostic marker for NF-1. The hyperpigmented spots that measure from 2 mm
to 4 mm may be congenital, but these typically appear and increase later in life. Scoliosis is the most
common musculoskeletal disorder of NF-1. The curves are frequently dystrophic, kyphotic, and have a
high risk of pseudarthrosis following spinal fusion. Anterior and posterior spinal fusion with rigid
posterior segmental instrumentation is the treatment of choice. PREFERRED RESPONSE: 5

95. A 12-year-old girl has bilateral developmentally dislocated hips. History reveals no previous
treatment, and she reports no discomfort. Good long-term clinical results are most likely to
occur with

1- surgical reduction of both hips and stabilization with a pelvic procedure.


2- no development of false acetabula.
3- performance of surface replacements.
4- abductor strengthening exercises.
5- Chiari osteotomy.

DISCUSSION: The natural history of complete developmental dislocation of the hip is dependent on
two factors: bilaterality and the presence or absence of a false acetabulum. Patients with bilateral
dislocations may have low back pain because of hyperlordosis, but they tend to have less disability than
patients with unilateral dislocations who have secondary problems related to limb-length inequality.
136 47
American Academy of Orthopaedic Surgeons 48
2001 Pediatric Orthopaedic Self-Assessment Examination.by Dr.Dhahirortho
Degenerative joint disease and clinical disability are most likely to develop in patients with completely
dislocated hips and well-developed false acetabula. In a 12-year-old child who has bilateral
developmental hip dislocations, it would be difficult to obtain surgical treatment results that are better
than the natural history of the disorder. Abductor strengthening exercises are unlikely to influence the
long-term outcome in this disorder. Surface replacements are not indicated in young asymptomatic
patients.
PREFERRED RESPONSE: 2

96. Thyroid hormone regulates skeletal growth at the physis by stimulation of

1-proteoglycan and aggrecan synthesis in the zone of proliferation.


2-fibroblast growth factor (FGF) activity.
3-transforming growth factor type beta (TGF-=) and parathyroid hormone-related protein (PTHrP)
activity.
4-chondrocyte proliferation.
5-chondrocyte hypertrophy, type X collagen synthesis, and alkaline phosphatase activity.

DISCUSSION: Children with hypothyroidism have delayed bone age, reduced thickness of the physis,
disorganization of the cartilage columns of the physis, and impaired differentiation of proliferating
chondrocytes into hypertrophic cells. As a result, these children have severe growth retardation, and
slipped capital femoral epiphysis may develop because of mechanical weakening of the physis.
Thyroid hormone regulates terminal differentiation of the growth plate chondrocytes, with a resultant
increase in type X collagen and alkaline phosphatase. These substances are important factors in matrix
mineralization. Insulin-like growth factors and FGF-2 appear to act synergistically to stimulate mitotic
activity of the growth plate chondrocytes. TGF-= and PTHrP stimulate proteoglycan synthesis and
mitotic activity of the chondrocytes and inhibit type X collagen and alkaline phosphatase activity.
PREFERRED RESPONSE: 5

97. A newborn has an anterolateral bow of the tibia and a duplication of the great toe. Which of the
following conditions will develop as the infant grows?

1- Lisch nodules and axillary freckling


2- Fracture of the tibia with pseudarthrosis
3- Increased tibial bowing and varus deformity
4- Decreased tibial bowing and limb-length discrepancy
5- Progressive valgus of the ankle

DISCUSSION: Anterolateral bowing of the tibia is normally associated with congenital pseudarthrosis
of the tibia. This, in turn, is associated with neurofibromatosis. Posterior bowing is more benign and
usually corrects spontaneously. However, anterolateral bowing also corrects spontaneously, and the
limb-length discrepancy may be the only remaining sequela when associated with duplication of the
great toe. Lisch nodules and axillary freckling are pathognomonic findings in neurofibromatosis but
would not be expected in this patient because this type of tibial deformity is not associated with
neurofibromatosis.
PREFERRED RESPONSE: 4

137 48
American Academy of Orthopaedic Surgeons 49
2001 Pediatric Orthopaedic Self-Assessment Examination.by Dr.Dhahirortho
98. During stabilization of a slipped capital femoral epiphysis, the screw penetrates into the joint.
The screw is repositioned so that it is within the femoral head. This transient penetration of the
hip joint will most likely lead to

1- an increased risk of chondrolysis.


2- an increased risk of osteonecrosis.
3- an increased risk of stiffness.
4- an increased time to closure of the physis.
5- no deleterious effect.

DISCUSSION: Chondrolysis may be associated with unrecognized permanent penetration of the joint
space by a pin or screw. However, transient penetration by the guide wire or screw is not associated
with this problem. One study described 11 hips in which there was transient intraoperative penetration
of the joint space by a guide wire or screw. These patients were followed for at least 2 years, with none
showing any clinical or radiographic evidence of chondrolysis. Another retrospective study of 55
slipped epiphyses described 11 hips with transient intraoperative pin penetration, with none showing
development of chondrolysis. There are no studies to suggest that transient pin penetration leads to
osteonecrosis, stiffness, or premature physeal closure. PREFERRED RESPONSE: 5

99. The mother of a 3-month-old infant states that she has difficulty positioning the infant’s legs
during diaper changes. Examination reveals limited abduction of both hips and a negative Ortolani
sign. A radiograph reveals bilaterally dislocated hips. Initial management consists of guided reduction
in a Pavlik harness, with weekly follow-up. Figures 57a and 57b show the radiograph and CT scan
obtained after 6 weeks in the harness. Management should now consist of

1- placement of the hips in wider abduction and continued use of the harness.
2- increased flexion of the hips and continued use of the harness.
3- removal of the harness and application of an Ilfeld splint.
4- removal of the harness and application of a von Rosen splint.
5- removal of the harness, followed by closed or open reduction.

DISCUSSION: In an infant younger than age 6 months with a complete dislocation of the hip that is
not initially reducible, the Pavlik harness may be used for a trial of guided reduction. When the harness
is used in these patients, the infant should be followed at weekly intervals to see if reduction has been
achieved. If the hip does not reduce after 3 to 4 weeks of harness wear, the harness should be
138 49
American Academy of Orthopaedic Surgeons 50
2001 Pediatric Orthopaedic Self-Assessment Examination.by Dr.Dhahirortho
discontinued, and closed or open reduction should be considered to avoid secondary deformation of the
posterolateral acetabulum, also known as Pavlik harness pathology. Changing to other abduction
braces is not indicated.

PREFERRED RESPONSE: 5

100. A 6-year-old boy with severe spastic quadriplegic cerebral palsy is nonambulatory.
Examination reveals 10° of hip abduction on the left and 30° on the right with the hips and knees
extended. The Thomas test shows 20° of flexion bilaterally, and Ely test results are 3+/4 bilaterally.
Radiographs show a center edge angle of 0° on the left and -10° on the right. The neck shaft angles are
170° bilaterally. Which of the following procedures would offer the best results?

1- Proximal femoral resections


2- Bilateral adducter, iliopsoas, and hamstring lengthenings
3- Bilateral varus derotation shortening osteotomies
4- Injection of botulinum toxin into the adducters bilaterally
5- Posterior branch obturator neurectomies bilaterally

DISCUSSION: The patient has bilateral subluxated hips, with nearly vertical neck shaft angles;
therefore, the treatment of choice is varus derotation osteotomy. Shortening of the bone on one or both
sides may be necessary to allow adequate range of motion postoperatively. In patients this age and
with this degree of bony deformity, soft-tissue releases are not likely to lead to hip stability. Botulinum
toxin has been shown to be effective in the treatment of ankle equinus, but its efficacy in other areas
has not been demonstrated as yet. The indications for obturator neurectomy are unclear at present.
Proximal femoral resection is a salvage procedure for long-standing hip dislocations that are
symptomatic and not reconstructable.

PREFERRED RESPONSE: 3

139 50
140

You might also like